Вы находитесь на странице: 1из 108

Pool

Of Issues

GRE Essays

Table of Contents
1.

As people rely more and more on technology to solve problems, the ability of
humans to think for themselves will surely deteriorate.. 8

2. To understand the most important characteristics of a society, one must study its
major cities... 9

3. Educational institutions have a responsibility to dissuade students from pursuing
fields of study in which they are unlikely to succeed10

Educational institutions should actively encourage their students to choose fields of
study that will prepare them for lucrative careers.
Some people believe that college students should consider only their own talents and
interests when choosing a field of study. Others believe that college students should
base their choice of a field of study on the availability of jobs in that field
College students should base their choice of a field of study on the availability of jobs
in that field.
College students should be encouraged to pursue subjects that interest them rather
than the courses that seem most likely to lead to jobs.

4. Scandals are useful because they focus our attention on problems in ways that no
speaker or reformer ever could....... 12

5. Claim: Governments must ensure that their major cities receive the financial
support they need in order to thrive.

Reason: It is primarily in cities that a nation's cultural traditions are preserved and
generated 13

6. A nation should require all of its students to study the same national curriculum
until they enter college..... 14

7. Some people believe that government funding of the arts is necessary to ensure
that the arts can flourish and be available to all people. Others believe that
government funding of the arts threatens the integrity of the arts.............................. 15

8. Claim: In any fieldbusiness, politics, education, governmentthose in power
should step down after five years

Reason: The surest path to success for any enterprise is revitalization through new
leadership... 16

9. In any field of endeavor, it is impossible to make a significant contribution without
first being strongly influenced by past achievements within that field... 18

In any field of inquiry, the beginner is more likely than the expert to make important
contributions.

10. Nations should pass laws to preserve any remaining wilderness areas in their
natural state, even if these areas could be developed for economic gain. 19

There is little justification for society to make extraordinary effortsespecially at a
great cost in money and jobsto save endangered animal or plant species.
Some people believe that society should try to save every plant and animal species,
despite the expense to humans in effort, time, and financial well-being. Others believe
that society need not make extraordinary efforts, especially at a great cost in money
and jobs, to save endangered species

3


11. People's behavior is largely determined by forces not of their own making 20

12. Governments should offer a free university education to any student who has been
admitted to a university but who cannot afford the tuition 21

13. Universities should require every student to take a variety of courses outside the
student's field of study.... 22

14. Some people believe that in order to be effective, political leaders must yield to
public opinion and abandon principle for the sake of compromise. Others believe
that the most essential quality of an effective leader is the ability to remain
consistently committed to particular principles and objectives... 23

15. Formal education tends to restrain our minds and spirits rather than set them free. 24

Some people believe that the purpose of education is to free the mind and the spirit.
Others believe that formal education tends to restrain our minds and spirits rather
than set them free.

16. The well-being of a society is enhanced when many of its people question authority... 26

17. Government officials should rely on their own judgment rather than unquestioningly
carry out the will of the people they serve. 27

Some people believe that government officials must carry out the will of the people
they serve. Others believe that officials should base their decisions on their own judgment.

18. Governments should focus on solving the immediate problems of today rather
than on trying to solve the anticipated problems of the future..... 28

19. Laws should be flexible enough to take account of various circumstances, times& places. 29

20. Claim: The best way to understand the character of a society is to examine the
character of the men and women the society chooses as its heroes or role models.

Reason: Heroes and role models reveal a society's highest ideals.. 30

21. Governments should place few restrictions on scientific research and development.... 31

22. The luxuries and conveniences of contemporary life prevent people from
developing into truly strong and independent individuals..... 33

23. The surest indicator of a great nation is represented not by the achievements
of its rulers, artists, or scientists, but by the general welfare of its people 34

Some people believe that in order to thrive, a society must put its own overall
success before the well-being of its individual citizens. Others believe that the
well-being of a society can only be measured by the general welfare of all its people.

24. The best way to teachwhether as an educator, employer, or parentis to praise
positive actions and ignore negative ones.. 35

25. Teachers' salaries should be based on their students' academic performance... 36

26. Society should make efforts to save endangered species only if the potential
extinction of those species is the result of human activities............................. 38

4
27. As we acquire more knowledge, things do not become more comprehensible,
but more complex and mysterious.. 39
28. Some people believe that scientific discoveries have given us a much better
understanding of the world around us. Others believe that science has revealed
to us that the world is infinitely more complex than we ever realized.... 40

29. In any situation, progress requires discussion among people who have
contrasting points of view.... 41

30. Governments should not fund any scientific research whose consequences are unclear 42

31. Society should identify those children who have special talents and provide training
for them at an early age to develop their talents 44

32. It is primarily through our identification with social groups that we define ourselves. 45

33. Claim: When planning courses, educators should take into account the interests and
suggestions of their students.

Reason: Students are more motivated to learn when they are interested in what
they are studying.... 46

34. The greatness of individuals can be decided only by those who live after them,
not by their contemporaries... 47

35. Students should always question what they are taught instead of accepting it passively... 48

36. The increasingly rapid pace of life today causes more problems than it solves. 49

37. Claim: It is no longer possible for a society to regard any living man or woman as a hero.

Reason: The reputation of anyone who is subjected to media scrutiny will eventually be
diminished.. 50

In this age of intensive media coverage, it is no longer possible for a society to
regard any living man or woman as a hero.

38. Competition for high grades seriously limits the quality of learning at all levels of
education. 51

39. Educators should teach facts only after their students have studied the ideas,
trends, and concepts that help explain those facts.... 53

40. Young people should be encouraged to pursue long-term, realistic goals rather
than seek immediate fame and recognition... 54

41. If a goal is worthy, then any means taken to attain it are justifiable.. 55

42. In order to become well-rounded individuals, all college students should be
required to take courses in which they read poetry, novels, mythology, and
other types of imaginative literature........ 56

43. In order for any work of artfor example, a film, a novel, a poem, or a song
to have merit, it must be understandable to most people 57

44. Many important discoveries or creations are accidental: it is usually while
seeking the answer to one question that we come across the answer to another. 58

5
45. The main benefit of the study of history is to dispel the illusion that people living
now are significantly different from people who lived in earlier times... 60

46. Learning is primarily a matter of personal discipline; students cannot be
motivated by school or college alone 61

47. Scientists and other researchers should focus their research on areas that are
likely to benefit the greatest number of people.. 62

48. Politicians should pursue common ground and reasonable consensus rather
than elusive ideals. 63

49. People should undertake risky action only after they have carefully considered
its consequences... 64

50. Leaders are created by the demands that are placed on them.. 66

51. The human mind will always be superior to machines because machines are only
tools of human minds.. 67

52. Every individual in a society has a responsibility to obey just laws and to disobey
and resist unjust laws.. 68

53. People who are the most deeply committed to an idea or policy are also the most
critical of it.. 69

54. Some people believe it is often necessary, even desirable, for political leaders to
withhold information from the public. Others believe that the public has a right to
be fully informed 70

55. Colleges and universities should require all faculty to spend time working outside
the academic world in professions relevant to the courses they teach 72

56. Knowing about the past cannot help people to make important decisions today. 73

57. We can usually learn much more from people whose views we share than from
people whose views contradict our own. 74

58. The most effective way to understand contemporary culture is to analyze
the trends of its youth. 76

59. People's attitudes are determined more by their immediate situation or
surroundings than by society as a whole 77

60. Claim: The best test of an argument is its ability to convince someone with
an opposing viewpoint.

Reason: Only by being forced to defend an idea against the doubts and contrasting
views of others does one really discover the value of that idea 78

61. All parents should be required to volunteer time to their children's schools.. 80

62. Colleges and universities should require their students to spend at least
one semester studying in a foreign country. 81

6
63. Claim: Any piece of information referred to as a fact should be mistrusted,
since it may well be proven false in the future.

Reason: Much of the information that people assume is factual actually
turns out to be inaccurate. 82

64. Claim: Many problems of modern society cannot be solved by laws and the
legal system.

Reason: Laws cannot change what is in people's hearts or minds.. 83

65. The primary goal of technological advancement should be to increase people's
efficiency so that they have more leisure time 84

66. Educators should base their assessment of students' learning not on students'
grasp of facts but on the ability to explain the ideas, trends, and concepts that
those facts illustrate. 85

67. Unfortunately, in contemporary society, creating an appealing image has
become more important than the reality or truth behind that image... 87

68. Although innovations such as video, computers, and the Internet seem to
offer schools improved methods for instructing students, these technologies
all too often distract from real learning... 88

69. The best ideas arise from a passionate interest in commonplace things 89

70. Claim: Imagination is a more valuable asset than experience.

Reason: People who lack experience are free to imagine what is possible
without the constraints of established habits and attitudes... 90

In most professions and academic fields, imagination is more important than
knowledge

71. To be an effective leader, a public official must maintain the highest ethical
and moral standards 91

72. Critical judgment of work in any given field has little value unless it comes
from someone who is an expert in that field 92

73. Any leader who is quickly and easily influenced by shifts in popular opinion will
accomplish little. 93

74. It is primarily in cities that a nation's cultural traditions are generated and preserved.. 94

75. When old buildings stand on ground that modern planners feel could be
better used for modern purposes, modern development should be given
precedence over the preservation of historic buildings 95

76. The best way for a society to prepare its young people for leadership in
government, industry, or other fields is by instilling in them a sense of cooperation,
not competition... 97

Some people argue that successful leaders in government, industry, or other fields
must be highly competitive. Other people claim that in order to be successful, a leader
must be willing and able to cooperate with others


77. Some people believe that corporations have a responsibility to promote the
well-being of the societies and environments in which they operate. Others believe
that the only responsibility of corporations, provided they operate within the law,
is to make as much money as possible. 98

78. Some people believe that our ever-increasing use of technology significantly
reduces our opportunities for human interaction. Other people believe that
technology provides us with new and better ways to communicate and
connect with one another. 99

79. Claim: Major policy decisions should always be left to politicians and
other government experts.

Reason: Politicians and other government experts are more informed and thus
have better judgment and perspective than do members of the general public. ... 101

80. It is more harmful to compromise one's own beliefs than to adhere to them.. 102

81. Claim: Colleges and universities should specify all required courses and eliminate
elective courses in order to provide clear guidance for students.

Reason: College studentslike people in generalprefer to follow directions rather
than make their own decisions.. 103

82. No field of study can advance significantly unless it incorporates knowledge and
experience from outside that field 104

83. True success can be measured primarily in terms of the goals one sets for oneself. 106

84. Governments should offer a free university education to all students.. 107



1. As people rely more and more on technology to solve problems, the ability
of humans to think for themselves will surely deteriorate.

The statement linking technology negatively with freethinking plays on
recent human experience over the past century. Surely there has been no time in
history where the lives of people have changed more dramatically. A quick
reflection on a typical day reveals how technology has revolutionized the world.
Most people commute to work in an automobile that runs on an internal
combustion engine. During the workday, chances are high that the employee will
interact with a computer that processes information on silicon bridges that are
.09 microns wide. Upon leaving home, family members will be reached through
wireless networks that utilize satellites orbiting the earth. Each of these common
occurrences could have been inconceivable at the turn of the 19th century.
The statement attempts to bridge these dramatic changes to a reduction in the
ability for humans to think for themselves. The assumption is that an increased
reliance on technology negates the need for people to think creatively to solve
previous quandaries. Looking back at the introduction, one could argue that
without a car, computer, or mobile phone, the hypothetical worker would need
to find alternate methods of transport, information processing and
communication. Technology short-circuits this thinking by making the problems
obsolete.
However, this reliance on technology does not necessarily preclude the creativity
that marks the human species. The prior examples reveal that technology allows
for convenience. The car, computer and phone all release additional time for
people to live more efficiently. This efficiency does not preclude the need for
humans to think for themselves. In fact, technology frees humanity to not only
tackle new problems, but may it create new issues that did not exist without
technology. For example, the proliferation of automobiles has introduced a need
for fuel conservation on a global scale. With increasing energy demands from
emerging markets, global warming becomes a concern inconceivable to the
horse-and-buggy generation. Likewise dependence on oil has created nation-
states that are not dependent on taxation, allowing ruling parties to oppress
minority groups such as women. Solutions to these complex problems require
the unfettered imaginations of maverick scientists and politicians.
In contrast to the statement, we can even see how technology frees the human
imagination. Consider how the digital revolution and the advent of the Internet
have allowed for an unprecedented exchange of ideas. WebMD, a popular
Internet portal for medical information, permits patients to self-research
symptoms for a more informed doctor visit. This exercise opens pathways of
thinking that were previously closed off to the medical layman. With increased
interdisciplinary interactions, inspiration can arrive from the most surprising
corners. Jeffrey Sachs, one of the architects of the UN Millenium Development
Goals, based his ideas on emergency care triage techniques. The unlikely

marriage of economics and medicine has healed tense, hyperinflation


environments from South America to Eastern Europe.
This last example provides the most hope in how technology actually provides
hope to the future of humanity. By increasing our reliance on technology,
impossible goals can now be achieved. Consider how the late 20th century
witnessed the complete elimination of smallpox. This disease had ravaged the
human race since pre-historical days, and yet with the technology of vaccines,
freethinking humans dared to imagine a world free of smallpox. Using
technology, battle plans were drawn out, and smallpox was systematically
targeted and eradicated.
Technology will always mark the human experience, from the discovery of fire to
the implementation of nanotechnology. Given the history of the human race,
there will be no limit to the number of problems, both new and old, for us to
tackle. There is no need to retreat to a Luddite attitude to new things, but rather
embrace a hopeful posture to the possibilities that technology provides for new
avenues of human imagination.

2.


To understand the most important characteristics of a society, one must
study its major cities.

The major cities, usually leading the economic development, respect some
aspects of the society characteristics, indeed. However, the society is a
multidimensional and complex system. To understand a society, many other
factors should be known about, such as festival, traditional culture and so on.
To begin with, the major cities can delegate some aspects of the society. The
economic is usually advanced in major cities, and the popular garments and
culture in major cities will thrive in minor cities later. Not only these things, the
modern life style is also leading by the major cities. For instance, the PayPal,
eBay, Amazon and other electronic commerce services are used in the
metropolis first, then with the development of fast express and other mature
system, they are spread to minor cites. It is same to Facebook and twitter, which
are first popular in higher education institute and collar worker. Therefore, it is
somewhat reasonable that the major cities are representative in some areas.
However, it should be noticed that society characteristics contain many other
matters, which are more obvious in small villages than in the metropolis. For
example, the traditional dietary habit is mostly maintained in the countryside.
Due to the rapid life style and high-pressure in metropolis, fast food and identical
set meal are prosperous. To meet the different demands of people, who come
from different areas in the world, the set meals in office buildings are usually
normal and lack characteristics. In contrast, in villages, families usually have
enough time and sufficient materials to prepare and cook the traditional food.
Furthermore, the similar factors contain festivals, which remained significant in

10

villages. Though the modern and popular life styles are lead by major cities, the
traditional and slow life characteristics are remained in small cities. These
factors are also indispensable to understand a society.
From another perspective, the society includes the huge cities and small ones,
which should all be took into consideration. Though the major cities respect
main trend in some aspects of the society, the number of the major cities are
limited in one country. Even assuming that the influence of small villages on
society is minor, the amount of the villages is huge. Because of this area and
population advantage, the life and dressing habit in small cities may be more
representative than in huge cities from the holistic views.
To understand a society comprehensively, miscellaneous matters should be
considered, the knowledge only about few major cities are not enough. Both
huge cities and small villages need to be understood to know a society.


3.

Educational institutions have a responsibility to dissuade students from


pursuing fields of study in which they are unlikely to succeed.

Educational institutions should actively encourage their students to choose
fields of study that will prepare them for lucrative careers.
Some people believe that college students should consider only their own
talents and interests when choosing a field of study. Others believe that
college students should base their choice of a field of study on the availability
of jobs in that field
College students should base their choice of a field of study on the availability
of jobs in that field.
College students should be encouraged to pursue subjects that interest them
rather than the courses that seem most likely to lead to jobs.

Education is a broad topic, one that means career preparation for some, and
means exploration of new ideas and opening of the mind to others. Choosing the
right education is important to success, making the education industry one of the
largest in the country, and also one of the areas that have continuously grown
even throughout times of recent times of recession. It is a given that educational
institutions should provide their students with means to success. They use their
resources to train and to education students to reach their highest potentials. So
if an education institution, through its high level of experience with various types
of students and with its knowledge of professional fields, finds that a student is
unsuited to his persuaded field of study, then it should dissuade the student from
that particular field.
Of course, some might argue that educational institutions have no right to do
persuade or dissuade students about their futures. After all, the idea of success is
different to each person. Success can mean financial stability, a creative life, or

11

the ability to be of service. What right do institutions have to decide success for a
student? And how exactly can educational institutions know how to judge a
student? Michael Jordan's high school coach considered him a horrible
basketball player and failed him during tryouts. Students have hidden potential
and no judge is perfect.
Yet looking at the first argument, though the idea of success is different to each
person, a basic tenet of success is satisfaction of a certain level of financial
freedom and of contributing to society. If an educational institute sees that a
student might have trouble fulfilling this basic need, then it should inform the
student of other choices he has in his career. Also, though not all judges are
perfect in their perceptions of their subjects, if a student has passion and the
drive to succeed in his given field, then no amount of criticism will stop him.
Michael Jordan would have become the number one basketball player whether
or not his coach dissuaded him. In fact, perhaps because of this dissuasion he
might have tried harder. If someone truly wants to do something, then he won't
be affected by dissuasion from others. If, on the other hand, a person is timid
about his future and uncertain, then a little guidance might be of great help.
An important reason why educational institutions should help their students
decide their courses of study is that they have the most knowledge of the real
world of job prospects and cases of successful students. They have access to
research, to past students, and to companies and organizations throughout the
committee. They have a more accurate view of the real world of professional
jobs. The student, on the other hand, though not in all cases, will most likely be
less knowledgeable of the current world than the educational institution. The
student will probably be younger and therefore have less experience with
careers and jobs, more likely to make decisions on faulty assumptions or on
misevaluations of him. Many students prepare for medical school by taking pre-
med during their undergraduate years, their basis for preparation being vague
dreams of being doctors or pressure from parents. However, if a student's
capabilities are not toward the medical sciences, they should be afforded the
resources to decipher this, if not actual counseling as to better career choices.
Educational institutions should also prevent students from spending
unnecessary time and money in a field they cannot succeed in. Costs of college
are increasing every year, with graduate school becoming the norm. If students
are investing so much of their time and money into an educational institute, that
institute has a responsibility not to just provide them with a diploma but also to
prepare them for their careers. There are pre-med students who spend years
trying to get into med school, only to be rejected by the colleges that advised
them during admissions. There are also colleges that are considered diploma
mills by knowledgeable professionals, who accept students based on their ability
to pay tuition, and who roll out diplomas and graduate students without taking
responsibility for their futures.
Because educational institutions have the most resources, connections, and
knowledge, and because their students are investing important time and money
into them, they must take responsibility toward their students by making sure
they enter fields they can succeed in. If a student's characteristics prove he

12

cannot be successful in his chosen field, the institution must provide him with
the realistic reasons why he is not suited and show him a better course of study.


4.

Scandals are useful because they focus our attention on problems in ways
that no speaker or reformer ever could.


The author mentions that scandals are those lightening events that speak
about the weather more prominently than rain or clouds can. Indeed, reformers
and speakers have a diplomatic tone of conveying when they attempt to remind
people about ethical codes and consequences. Scandals remind people of the
limits more strongly, when a person involved in it goes thru the consequences.
For instance when renowned sportsmen loose their sports credibility just
because of a ridiculous extra marital affair, it helps other sportsmen to keep a
check on their private behavior. Similarly, in the field of politics, religion, charity
and social services any event of scandal actually takes a heavy toll on the faith
people have in that system. This is like an alarming situation that no one wants
to be in.

Organizations and institutions keep volunteers and employees up to the mark of
training and education to prevent any scandalous mishaps. However, in the
event of such occurrence the results are more complex than simply reforming or
cleansing. Scandals at many times are allegations that get publicity. These
allegations are often hear say of events and prejudices accumulated. When a
scandal is exposed a can of worm is opened. There are lot of investigations,
litigations and other proceedings that go to prove black and white. This certainly
costs great amount of resources, time and energy for anyone involved in it.

In the events, of the allegations being baseless the person still has to go through
a painful path before one gets exonerated. This causes emotional scars, loose of
respect amongst family members and social seclusion. At the cost of all vital
aspects if a scandal impacts reformation at certain level than one must wonder
about its value.

Recently, in China when the milk powder scandal surfaced there was a lot of
blame game that went about. The bureaucrats blamed the production company
and the company blamed its quality control engineers. Yet at the end of it there
was not much of a system change in the way processed food companies logistics
exist. Hence, one would wonder if scandals really help in performing more
efficiently than reformers do.

A reformer or a speaker acts as a preventive and proactive element, which
motivates the system to keep it healthy rather than a scandal which actually calls
for cure after its occurrence.

13

Thus, a scandals effects no matter how remarkable it may be, possibly leaves
many economical or emotional scars on the people involved in it. So assuming its
thorough usefulness is a little too audacious a belief.


5.


Claim: Governments must ensure that their major cities receive the
financial support they need in order to thrive.

Reason: It is primarily in cities that a nation's cultural traditions are
preserved and generated.

The author concludes that the major cities should get more financial
support because the cultural traditions are preserved and generated in these
metropolises. However, personally, small villages may maintain more traditional
factors. The support to culture protection should be combined in major cities and
small villages. Furthermore, the spirits and other invisible aspects in cultural are
also crucial to culture protection.
To begin with, some culture factors in big cities are hard and expensive to be
maintained. However, in small villages, the cost to culture maintenance will be
considerable lower than in the big cities. For instance, all the hoary architecture
need work in maintenance. In huge cities, the worker employee fees are higher
and the real estate value is far higher than small villages. Except for the economic
considerations, the rare places in major cities may be used for better benefits,
such as hospitals, schools and so on. Taking the large numbers of people in
metropolis into consideration, these infrastructures may be more practical and
exigent for residents. Therefore, when the same traditional buildings exist in
huge cities and small villages, the culture protection in small villages maybe
more economical and advisable.
From another perspective, more cultural factors may be remained in villages
than in cities. The cities are more vulnerable in battles than villages. The complex
politic turmoil, often happened in huge cities, also has negative influences on the
culture protection. In small villages, the calm and steady life style is better for
culture protection. Some old architecture remains only damaged structure, due
to the war. For instance, the temple in Athens, and the old summer palace in
Beijing. However, in small villages, some old structures can avoid the war
damage.
Furthermore, more traditional factors, such as poetry, literature, and ballad, are
inherited invisible, regardless of the area difference. These intangible factors are
maintained in small villages easier than in cities, since the life style in huge cities
are influenced by modern technology productions. For example, the popular
music concert may replace the Shakespeares poetry in cities. The novels, like
Harry Potter, may replace the Bible story in childrens hands. The fast food and
set meal may replace the traditional food in most cases. However, in villages, the
music concert is often few, and traditional stories may be talk more, so that the

14

old value and spirits may be inherited more in villages than in cities. The
protection for these things doesnt need much financial support, but education
and propagation.
To sum up, both cities and villages have some kind of unique factor on cultural
tradition. Small cities may contain more old culture factors in some aspects, and
the invisible things need more attention and should be educated to next
generation. The financial support may not that important comparing with the
invisible propagation. In conclusion, if financial and other methods support are
applied to cultural protection, the excellent tradition will be inherited for long.


6.

A nation should require all of its students to study the same national
curriculum until they enter college.

The children of a nation are its future. Along with becoming independent and
successful, they are the ones to carry the responsibility of running the nation as
well as developing it in the future. Therefore, it is very important to form a
strong base in their formative years so that they are not only successful in the
personal front but also turn out to be good citizens. Education plays an
important role in forming the basis of a child's future. The curriculum should
therefore be carefully chosen to fulfill these requirements. I believe that the
curriculum being followed in a nation should be similar but schools should have
some freedom to introduce or change it to a certain extent. While the larger part
of the curriculum should be decided to remain similar on a national level, it
should not be binding schools completely to follow it strictly.
Having a uniform curriculum is beneficial in many ways. It determines a
standard level, which is required by every student to clear before he is able to
take another academic step. Each level prepares the student for the next level
and this forms a stepwise format of learning. A common curriculum ensures a
stable and universal learning pattern. It allows making the curriculum familiar
throughout the nation and it becomes easier to refer to it. There are many
students who attend more than one school during their education. A common
national curriculum is very important for such students who leave one school
and join another at a different place. For example, consider the people in a
transferable job like the defense. The children of a defense person have to go to
different places along with their families. It is evident that they need to change
their schools as often as their guardians get transferred to a new place. If there
are no guidelines for curriculum given to schools, these children will have to face
new and unfamiliar curriculums everywhere they go. This will make it difficult
for them to cope up with the requirements of a new curriculum and they will be
at a great disadvantage. According to me, a uniform curriculum at all places
makes it easier for such people to change schools and adjust in a new
environment without hampering their growth.

15

Today is the age of competition. In order to find a place anywhere the students
need to compete with each other. Therefore, uniformity is required at some level
so that there is fair competition. Generally, students have to face an entrance test
at the college level to secure a seat. This entrance test is based on the knowledge
acquired by students in their high schools. If there is variation in the curriculum
that they have studied, the entrance test will not be made on similar lines for
everyone. Hence, it would benefit some students who have studied that
curriculum and the other students would be at a loss. There would be no way to
judge their capabilities with the same parameters. Hence, it is very important
that uniformity be maintained in the curriculum at the school level, and students
are able to face fair competition when they leave school.
However, I maintain that along with following the national guidelines for
curriculum, schools should have certain liberty in choosing the format of courses
it offers. Each student is different and so are his abilities. While there are some
students who are fast learners, some are average and some are extremely slow.
Moreover, the interests of students vary. Keeping in mind the interests of
students, the curriculum should be molded so that they retain their interest in
studies altogether. Hence, schools should be allowed a certain amount of
discretion in making certain changes according to the needs only for the
betterment of all the students.
In the end I would I like to conclude that a uniform pattern on the whole is very
important at a national level. However, at the same time, schools should not be
bound to follow it blindly and should be able to make certain changes that help
the students in their studies.



7.

Some people believe that government funding of the arts is necessary to


ensure that the arts can flourish and be available to all people. Others
believe that government funding of the arts threatens the integrity of the
arts.

Since time immemorial art has been an integral part of our society. The varied
creativity of people results in distinctive forms of art. There are various forms of
arts prevalent around the world. Different nations have different forms of arts
and arts depict the culture of a nation. The various forms of art act as a mirror of
the country's values and beliefs. Art is not dependent upon any other source for
its existence. The only thing that an artist ever wishes for is his admirer who is
the true source for his inspiration. In the present scenario government has taken
due interest in protecting various forms of arts of a country. I believe that an
artist does not need any sort of monetary support and hence an artist is capable
of protecting his form of art. Therefore, government's intervention by funding
arts is unnecessary and it would certainly harm the integrity of any art form.

16

An artist only seeks admiration from public and the various forms of art came
into existence without any monetary support but out of creativity. An artist's
creativity does not depend upon any sort of funding and an artist cannot be
forced to create any form of art because government funds that art form. A
dancer does not need funds to support his dance rather he needs admirers and
followers who would take his legacy to another level. Similarly, a sculptor is not
dependent upon government's funds for his sculptures rather he would need
students who would keep alive his art form. There are numerous examples from
history, which depict that the various art forms came into existence without any
sort of funding. These art forms are till date followed by people. The great
painter artist Leonardo de Vinci was not funded by any government for his art
form but his admirers were the true source of his popularity. Art has been
prevalent in our history without the support of funding from government.
Although art has always been there since the time humans have been there in
this world. There are numerous art forms that are in existence without the
support of the government. The government intervention in protecting certain
art forms because those art forms were on the verge of extinction is credible. In
such fields the role of government has shown remarkable success. The support
from government has only helped in saving an art form and also it has aided
artists to keep their art form alive. Government supports the various small-scale
industries and pottery is one of them. The artisans are now provided funds by
government to save their art form.
Various art forms have been in existence on their own and without the support
of the government. It is not important that art needs funding for its existence but
when any form of art needs funds the support from government can always be
asked for. The government should adopt measures to fund arts when it is
needed. The government should also take steps to encourage young aspiring
artists.


8.

Claim: In any fieldbusiness, politics, education, governmentthose in


power should step down after five years.
Reason: The surest path to success for any enterprise is revitalization
through new leadership.

Does a leader lose his charisma and competence after five years? The
answer to this question is debatable. People in power may be competent enough
to handle their responsibilities, but it would be a grave mistake not to give a
chance to the others. It is likely that there may be someone else who is able to
handle the same responsibilities in a better manner. Moreover, since the modern
world now elects its leaders in almost all professions, it would be a wise decision
to change the leadership regularly. It may not necessarily be after every five
years. The duration may be further reduced or increased, but a change is
inevitable for success in any enterprise.

17

There have been monarchs who have successfully ruled their countries for
decades. On the other hand, there are examples of rulers who were overthrown
in a coup because the public found that they were not worthy of the position of
power held by them. If monarchy was indeed so successful, then why is it that
democracy has replaced monarchy in most countries across the world? This is
because there is the requirement of a change in leadership at regular intervals.
This change of leadership serves two purposes. Firstly, the person in power is
aware of the repercussions of carelessly handling his responsibilities. He knows
that he can be removed from his post even before his tenure is over, so he puts in
his best for doing what is expected of him. Secondly, this process ensures that
new talent is given a chance to come and prove their competence in carrying out
the tasks assigned to them. There is always the possibility of someone remaining
in power for the next term also if the organization fails to identify a suitable
replacement for him. Therefore, a change in leadership is bound to bring in long-
term benefits for the enterprise in addition to success in all its ventures.
It is an exhilarating feeling to have the power to make people dance to your
whims and fancies. It is this feeling that is the reason behind the prevalent
corruption amongst the people in power. We have heard of so many instances
wherein the people in power have misused their official position for personal
gains. Stories of embezzlement of accounts, fraud, bribery and nepotism have
generally been found in the front pages of our national dailies. These are all
related to the misuse of power by the people who have been placed in this
position for the betterment of the organization that they work for. A change in
leadership ensures that such people will eventually be removed after their
tenure is over and the new person thus appointed may be much more honest and
cooperative, thereby improving the working conditions in the organization.
There are numerous students who earn professional degrees every year. As
technology keeps changing at short intervals, it would be a wise decision to try
out new talent, as they would be trained to keep instep with the latest
developments. If the head of the computer department of a high school is a
teacher who had graduated 10 years ago, wouldn't it be a better option to
replace him with a teacher who has recently graduated and who is aware of the
latest technological developments in the world of computer science. The school
and the students will benefit from the knowledge of the new teacher who can
give an entirely new dimension to the computer education being imparted in the
school.
In sum, it becomes necessary to change the leadership at regular intervals in
order to ensure that the best available talent in the market is tapped for leading
an organization on the path to success.




9.

18

In any field of endeavor, it is impossible to make a significant contribution


without first being strongly influenced by past achievements within that
field.

In any field of inquiry, the beginner is more likely than the expert to make
important contributions.
Necessity is the mother of invention. History is replete with examples of
inventions that human invented due to their necessity and observations.
However, in recent era, the form of inventions has drastically changed and
improvements have been made through recent experiments. The entire process
of inventions and developments are the result of either self-influence and
observation or encouragement by past achievement. Therefore, the above
statement is arguable and could be true to specific circumstances.
The focused concentration power and observations gave birth to many
inventions. Newton observed the energy being falling of apple, Thomas Edison
invented bulb, and Einstein discovered the law of gravity etc were the results of
self-motivated inventions. At that stage, there was not any history in front of man
to get inspiration and initiating point to meditate on. The nature and
surroundings provided source of inspiration and courage to invent that is in
contrast with the statement.
In past, human commute long distances either by foot or utilizing animal's
strength. There were so many difficulties during journey like weather hardship,
health problems, riot etc. All these problems gave birth the invention of Wheel.
Wheel is considered one of the revolutions in mankind history. The invention of
the wheel was not the achievement due to inspiration from past but rather it was
result of human need that is in converse to the statement.
In recent era, people come across with many inventions in their daily life. People
get inspirations from these inventions and gradually they develop their interest
in some particular thing. The invention of jets, automobiles, medical
advancements, higher machinery for agricultural etc are the recent
developments that were the result of influence from past like Wheel, Law of
gravity etc. All these advancement would not be possible without past influences
and inspirations. In past computers and mobiles were so big and heavy but
humans' contribution to make this entire device tiny and fast is the remarkable
achievement. Human made lot of progress and innovation into invention that
developed in past. Therefore, it is correct to comment that in any field the
significant contribution is impossible without strongly influenced by past
achievements.
Conclusively, it is imperative to have rich history of inventions to inspire with it
and make contribution on it. However, the law of nature provide sufficient
knowledge and information to articulate simple idea into great inventions.

19

10. Nations should pass laws to preserve any remaining wilderness areas in
their natural state, even if these areas could be developed for economic
gain.

There is little justification for society to make extraordinary efforts
especially at a great cost in money and jobsto save endangered animal or
plant species.
Some people believe that society should try to save every plant and animal
species, despite the expense to humans in effort, time, and financial well-
being. Others believe that society need not make extraordinary efforts,
especially at a great cost in money and jobs, to save endangered species

We should be grateful to the bountiful nature for sustaining us, as it had
sustained our ancestors for many generations. But, don't we need to preserve it
for posterity's sake? State of wilderness is a very exigent issue, that should be
concerning every nation in fact every individual. We have already cut down
innumerable trees and cleared forest for our selfish economic gains. Post
industrialization has seen a complete metamorphosis of how vast stretches of
green land and forests had been replaced by factories, industries, commercial
building and township resulting in uncontrollable pollution, global warming,
diseases such as lung cancer, asthma and other respiratory disorders owing to
pollution and lack of fresh air. Hence, I undoubtedly agree that nations should
implement strict laws to curb further deforestation.
Only countries, which are under developed, have fair amount of wilderness area
and where economic development is a prerequisite for survival should consider
about disturbing them within limits. They should learn from mistakes of
developed nations and take thoughtful actions - how to manage their wildlife
resources such that minimum damage happens to environment.
Forest contributes a lot in rain, maintains levels of oxygen, recycles carbon
dioxide and is home to innumerable species of flora and fauna. Unchecked
deforestation has lead to global warming, disturb pattern of rain and climatic
variations.
Due to man's reckless deforestation many animals are on the verge of extinction
or their population have significantly reduced. Siberian tiger, Panda, dodo bird,
American Bison, mountain gorilla are a few to mention. We are left with only 3,
000 wild tigers now whereas a century back their number was around 1, 00, 000.
Well what a shame
Incidents reporting animals attacking people in cities are also increasing these
days because of loss of their habitat and subsequent disruption in their food
cycle. Human beings for their selfish motives brought down these forests but to
keep them from further damage nations should enact stringent laws for
wilderness protection as well as for regulating mining. Furthermore,
afforestation should be encouraged and practiced wherever possible.
As "Necessity is the mother of invention", our scientist and engineers would
surely find some alternatives instead of cutting down trees for fulfilling their

20

needs. Increased green cover will ensure better environmental balance,


improved health for their citizens, will preserve their natural wildlife. Apart from
this scenic beauty of the place can attract tourist and generate revenue. In short,
it will make their country a better place to live - for them and for future
generations to come.
To sum up, it's a high time that whatever wilderness is left is kept intact;
otherwise whole world will have to face dire consequences. There is nothing
wrong in economic development but it should not be on cost of environment, it
may seem profitable right now but then earth won't be able to survive for long.


11. People's behavior is largely determined by forces not of their own making.

The term "behavior" is not quite easy to describe. There exists a plethora of
definitions for it according to individual perspectives. But, in general "behavior"
means how a person acts or reacts according to the external or internal forces.
Human beings being the most intelligent species, have liberty of choice. They
think, decide and execute things. People do behave in a way they are brought up.
Many external factors such as society, environment, school, peers, families may
contribute to their thinking, but it is highly unjustified to say that external factors
are solely responsible for the behavior of people.
We as human learn, hear and absorb things but eventually it is us who decides
what to do. We have free will and we can only account ourselves for our actions.
However, there are outside factors that contribute to human behavior. External
factors always had impact on humans. History shows many a great or flawed
men were a product of these influences. But, eventually it depends on the
individual about how and what to do. There are laws, rules and regulations in the
society but it depends on the individual whether to abide by those rules or not.
For instance, in a particular country there may be a punishment for throwing
garbage on the road. A person traveling to that country might not throw garbage
being extra cautious about the rules there. But, when he goes to the other
country where there are no such rules, he might. In this case, their sense of
individual responsibility will come into play. External forces might push people,
make them think, realize what needs to be done. In the end it is completely
dependent on us to make the decisions. We are our decision-makers and nothing
can make us do what we don't want to. There are rules, laws and outer forces
that continuously remind us against doing bad or illegal. But, there are people
who break laws, which is evident by the crime rate. So, what are the effects of
external forces on such people? They have decided not to let the external forces
determine their behavior. Students are continuously told to study well for their
bright future. There are students who will let the external forces push them and
study well. On the other hand, there are students who will not let the external
forces influence them and act according to their will. A thief will steal in spite of
the laws in the society. Many argue that their act resulted from poverty,

21

circumstances etc, but it is eventually the thief who makes the decision in spite of
the options. A person can only be responsible for his behavior. External forces
can play a part, but human beings are the outcome of their own actions.
However, the above statement is not entirely without support, it runs contrary to
common sense and everyday human actions. So, I firmly believe that our action
springs from our free will accords with common sense and everyday experience.


12. Governments should offer a free university education to any student who
has been admitted to a university but who cannot afford the tuition.
Education is a vital requirement for the progress and development of nations.
The increase in the level of knowledge among the people will be directly
reflected in its development level. Therefore I total agree that governments
should offer a free university education to any student who has been admitted to
a university but who cannot afford the tuition.
Since education is a responsibility of the governments and it is included in the
annual budget. Therefore they have to take into consideration the equality
among all people. They should offer equal opportunities among their people, no
bias to any group other than the other. There should be no differentiation
between races, casts and social levels. Taking the university tuition as criterion
to compare the differentiation among the government practices, we will
conclude that it will be unfair for the poor people, as only the rich people will
afford the high tuition fees of universities and they will be the only people in
society who hold university degrees. For more clarification, only the children of
the doctors, engineers, businessmen will be well educated and join universities
while the majority of average and below average caste in the society will still
have to be working as white collars. This will increase the gap between social
levels in the society.
In addition, the government investing in its people will be the mean for its
development and progress. In order for the government to eliminate and
decrease the level of illiteracy among its citizens, it has to spread education and
help people earn degrees. As to the increase in globalization and technology,
governments have to cope with that spread of technology and knowledge among
countries. If they did not help majority of people earn university degrees, they
will not cope with that vast spread of knowledge. For example, developing
countries where governments do not have a big budget for education, not all
people are well educated and therefore affecting the progress and development
of these countries. I think that governments should focus on education in order
to develop the skills of the people.
To conclude, governments will not be able to cope with the knowledge era and
globalization unless concentrated on the educational level of its people. The
number of well-educated people in the society will measure the progress and
development level of nations. Also they should decrease the gap among social

22

levels and give them all the same opportunities to be educated and receive
university degrees. Therefore I strongly agree that governments should offer a
free university education to any student who has been admitted to a university
but who cannot afford the tuition.


13. Universities should require every student to take a variety of courses
outside the student's field of study.

It is true that one should be aware of various academic disciplines in order to
expand his vision and become truly educated. One's knowledge cannot remain
restricted to one's own field of study or else he will not be able to analyze his
knowledge with respect to the happenings in other related fields. However, the
decision to acquire knowledge of academic disciplines other than their fields of
study should rest with the students themselves. The recommendation that
students should be forced to study subjects other than their fields of study is
extremely unconvincing.
Students are in universities for a short duration of time that amounts to a couple
of years in which they have to acquire maximum possible knowledge about their
fields of study. These students will go on to become professionals in their
respective fields. Is it justified to take out time from this precious period to study
subjects that will have no relevance to the career that the students plan to
pursue? If someone is studying medicine, will it be justified to force him to study
history? It is true that knowledge of history will definitely make him truly
educated as he will be aware of historical facts which are essential for being
well-informed. However, would this knowledge make such a big difference to his
life that it should be made mandatory for him to study this subject that has no
relevance to his field of study?
It is a well-known fact that students have to appear for various tests to prove
their capabilities in a particular subject. They are given admissions to various
programs of study depending upon their performance in such tests. Therefore,
will all students be able to study all subjects of the university level with equal
ease? A student who has an aptitude for Biology may not be in a position to study
Mathematics that easily. If he is forced to take a course in Mathematics, it might
eventually turn out that he spends more time in studying Mathematics rather
than studying Biology, which is his main subject. Making the students study
subjects other than their fields of study cannot be justified without assessing the
aptitude of the students and their liking for the additional subjects.
It is not warranted to force an engineering student to study English literature in
the time that he should have spent studying the concepts of engineering. Such an
action could have serious repercussions like the student not being able to
become a competent engineer as he wasted precious time during his years in the
university memorizing facts that were of no relevance to his choice of career. It
would be a much better option to give ample time to students in their years in
the universities to learn as much as they can about their fields of study. They can

23

choose to take up additional courses pertaining to other disciplines if they feel


that they have spare time for the same. For instance, a student studying History
may have additional time on his hands as he does not have to go for practical
classes and therefore, he may choose to take up a course in some foreign
language.
In conclusion, there is no denying the fact that gaining knowledge about various
disciplines is of a great advantage as it helps you to be more learned and
educated. At the same time, enforcing regulations in universities that force
students to take a variety of courses outside the students' fields of study is
unwarranted as the students should be allowed to take this decision depending
upon their capabilities.


14. Some people believe that in order to be effective, political leaders must
yield to public opinion and abandon principle for the sake of compromise.
Others believe that the most essential quality of an effective leader is the
ability to remain consistently committed to particular principles and
objectives.

Public opinion is the essential element in democracies. However, by taking into
consideration of the size of the population, it is not usually possible to ask
publics opinion about every action that the governments would make. In order
to accelerate decision-making process in democracies, citizens elect leaders who
represent his voters view in the political area. Although it is important for a
leader to behave according to public opinion, a leader should not abandon his
principles for the sake of compromise. A leader who claims that his primary
concern, when deciding actions is the public opinion, usually cannot be trusted
by his citizens and other countries governors in the political area.
To begin with, leaders might manipulate the facts to get the support of public for
the actions they desire. Hence, a leader who asserts his actions are based on
public opinion as often as not distorts the reality and manipulates the facts for
his own sake. For example, Germans elected Hitler in 1930s and he claimed that
he took the support of German nation for his actions. His actions leaded to a
world war, which had devastating results for both parties in the world especially
for Jews. Even though he was an elected leader, it is impossible to claim that the
majority of public supported his governments misbehavior and massacre
towards the Jews. He undermined the scope of his actions and lied to his citizens.
Hence, public support was cultivated not because of realities but due to distorted
facts. As in the case of example, leaders can easily misdirect public so populace
should not trust these types of leaders.
In addition, as well as his citizens, the political leaders in other countries cannot
trust this kind of leader. Hence, behaviors of this kind of leaders undermine the
position of his country in the political area of the world. In order to become an

24

economically powerful country, a state should have strong trade relationships


with its neighbors. Political relations are also essential for creating strong
bounds between countries. However, when a leader acts in way which might
hurt the relation of two country, the other leaders would not trust him and do
not want to take their trade relationships further. For example, petroleum and
natural gas reserves of Iran should make Iran one of the most affluent countries
in the worlds. However, their leaders hostile attitude towards Christian
countries, especially America, leaves Iran alone in political area. As a result, Iran
could not enjoy this economical potential because of the other countries could
not trust actions of their leaders.
Finally, some people might claim a leader should only consider, without
considering problems stated above, public opinion when deciding an action.
Although public opinion is important, democracy is not a regime in which
majority of public oppress the minority. Hence, leaders and governments should
protect the rights of the minorities in their actions. However, taking public
opinion might weaken the position of people who do not agree with public
opinion. For example, in countries where majority of populace believe in Islam,
public opinion would lead to an Islamic regime. However, this type of religious
regimes has devastating effects for atheist and homosexuals. As a result,
governments should consider all parties interest when making decisions not
only majority under the name of public opinion.



15. Formal education tends to restrain our minds and spirits rather than set
them
free.

Some people believe that the purpose of education is to free the mind and the
spirit. Others believe that formal education tends to restrain our minds and
spirits rather than set them free.

A child learns whatever is taught to him by either his parents or his teachers.
The knowledge being imparted to a child conforms to the facts and figures,
which have already been established. It is seldom seen that children are given
the time or the space to explore their inner selves and free their minds. This is
because the pressure of doing well in school in order to earn high grades is so
much that children often having no option but to stick to their schoolbooks.
When the same children grow up into adults and they are well settled in their
careers, the formal education imparted to them plays on their minds all the time
and they are unable to think outside the realms of formal education. Therefore,
formal education that has been acquired over a period of time succeeds in
restraining our minds and spirits instead of setting them free.

25

The importance of learning in a formal classroom environment has assumed a


great significance in the modern world because the grades that you get in your
high school or college can determine how your future will shape up. Children do
not have the freedom to learn aspects that interest them as they are under
constant pressure from their parents to get good grades in school. Moreover, the
children themselves are conscious of the performance of their peers in school
and so they try their best to keep in step with their friends who may be doing
better than them. The fear of failing to score the passing grade makes them
concentrate on their studies.
Even if students are able to secure good grades in school, the story does not end
there. After completing high school, there is stiff competition for securing
admissions to reputed professional courses and only a few students are able to
join a program of study of their choice. This is because there are various
standardized tests which tests your competence in the field in which you wish to
carry out your higher studies. If you want to succeed in a career of your choice,
then it is essential for you to earn degrees that endorse your skills in the fields
related to your career.
Once you are settled in a career of your choice, you are again limited to utilizing
the information gained from your formal education because it is this knowledge
that will help you rise in the hierarchy of your career. Therefore, it is evident that
the life of an individual constantly revolves around his formal education. A
person's thinking ability is largely influenced by the formal education that he has
received and this education restrains him from thinking otherwise, unless he
does so by risking his career and a successful future. A person will have to make
a substantial effort to tear himself away from the norms of formal education if he
wants to free his mind. The rewards for doing well in your formal education are
manifold and these rewards deprive people of the motivation to set their minds
free. The end result is that people do not have the incentive to set their minds
free as their minds are restrained by the formal education that has been
imparted to them.
In view of the above, it is evident that formal education assumes immense
importance in the early years of life. The pressure exerted by the importance of
formal education is such that there is little or no time left for anyone to allow
their minds to be free and to think of something outside the realms of formal
education.








26

16. The well being of a society is enhanced when many of its people question
authority.

We all live in a society where the authority to run the society is given to
some capable hands. It is impossible for all of us to have the authority that is why
some representatives chosen by us are given authority to run the social system
for us. These people help us to create a better environment for our living. We as
citizens have all the rights to question their authority. If we think that certain
things are not happening as they should be happening we can question the
authority. Although it is the prime responsibility of the people who are given the
responsibility to provide us with all the comforts but if we question their
authority then these people become more prompt and it certainly enhances the
well being of the society. If we were denied the right to question then this social
system would be like monarchy where nobody can question the authority.
Therefore, I believe that if we keep exercising our right of questioning the
authority it would make the authorities provide us with better facilities.
If a person is not answerable to anybody then in spite of his sincerity he might be
negligent in performing his duties. On the contrary if a person is aware of the fact
that he is answerable to the public for what ever he does for the society, he
would make sure that everything is done properly. Authority and responsibility
are the different sides of the same coin. They appear to be different but they are
integral parts of each other. Therefore, a person who is given authority has to
take the responsibility. It was in the past when there was authoritarian rule like
the one of Adolph Hitler but soon people realized the importance of being free
and slowly all such practices came to an end. In the present day scenario people
are more educated and thus more aware of their rights and they want to
question if they are deprived of their rights. The right to question has brought
revolution and people have become stronger. Now nobody can make a fool out of
them as they know what their rights are.
Although it is the responsibility of the authorities to be answerable to the public
but that certainly does not mean that public interferes in all the matters. There
are some matters where public interference cannot be allowed. There are some
decisions that are taken in emergency which are for the betterment of the society
and under such circumstances authorities are not answerable to public. If there
is too much interference from the society then there can also be delay in
important decision-making. Such decisions, which need immediate attention, will
take long if there is public intervention.
Therefore, public should question the authorities but public should not prove to
be a hindrance in decision making by the authorities. Public should participate
but not become an obstacle for the government.


27

17. Government officials should rely on their own judgment rather than
unquestioningly carry out the will of the people they serve.

Some people believe that government officials must carry out the will of the
people they serve. Others believe that officials should base their decisions on
their own judgment.

The government of a nation is meant to serve its people and take care of its
present and future well-being. In any country, it is not possible for any
government to survive if it does not satisfy its compatriots. When people are not
happy with their leaders, there is bound to be revolt, agitation and defiance of
the government's decisions. Nonetheless, it is not wise for any government and
its officials to 'unquestioningly' accept the demands of its people. There has to be
contemplation over every decision that a government makes and over every
requirement of the public. I agree with the given statement, mainly owing to the
word 'unquestioningly.'
It is the duty of any government to look after its people. No matter which form of
government a country follows, the government is to put the interests of its
people before any other agenda. Well-being of its people is the sole criteria that a
government should consider while making any decisions. When the government,
its decisions and policies do not satisfy people, they are sure to revolt. The
uprising of Indians against British and the recent revolt of Tibetans against China
and are such examples. Such revolts are even seen at lower levels of organization
of public. If a state is not happy with the way its leaders function, there are
revolts and agitation against them. Though the society has to be satisfied, it is not
wise for any government to listen to its people without questioning their
opinions.
Every society is composed of people from different classes. There are those who
are poor, middle class and rich. Similarly, a society can also be classified
according to the occupations of people. There are businessmen, salaried
employees, entrepreneurs, private and public sector employees in every society.
The requirements of each category of people are different and so are their
expectations from the government. Because of the differences that exist in its
people, it is very rare that unanimous opinions exist. It is then the government's
duty to reach a decision that is acceptable by one and all. For everyone to be
satisfied, it is required that each opinion be given due consideration and be
questioned by the government. If its people make certain claims and demands,
the government has to question the basis of the same. This questioning has to be
done for ensuring that what people want is actually what they require. Never
should a consensus be arrived at without discussions and debates.
At times, there are policies of a government that are made by keeping the future
in mind, rather than a short-lived present discomfort. At other times, there are
decisions that are for the betterment of downtrodden sections of society rather
than for the creamy layer. Such decisions could upset one lot of people and
satisfy others. It is then up to the government to consider different judgments
and take a tough stand. The government is meant to serve its people, but it is a

28

composition of leaders. The leaders should keep the interest of people in the
forefront and take tough decisions about things that they foresee as beneficial,
irrespective of the opposition they face.


18. Governments should focus on solving the immediate problems of today
rather than on trying to solve the anticipated problems of the future.

Government is a body that governs the rules and regulations of a state in the
interest of the people. It sets some targets for the common well-being of all the
citizens and tries to bring the less privileged at par with others. A government's
main motive is to look after the needs of the people, best utilization of resources
and implementation of ways for the development of the nation.
There are a few issues that every government has to deal with immediately, and
some that need long term planning with foresight. It is therefore important for
any government to segregate these issues to avoid chaos and to work efficiently.
If on one hand, the problem of unemployment needs immediate action, then on
the other hand, the economic reforms should be made by anticipating the
upcoming demands and problems. The government must try to foresee the
future prospects of any project undertaken. It must be in a position to frame a
tentative outline to meet the future demands, keeping in view the resources in
hand. The borrowing or sharing of the latest technologies is done with an aim to
have a smooth future. Since, enormous money is involved in adopting a
technology and starting a project at the national level, like constructing a power
plant, building a highway, importing or manufacturing aircrafts and ships,
building a dam or railway line, the government cannot simply base them on the
present demand and supply ratio. In fact, it should have a vision to see the future
need and applications to avoid wastage of time and labor in the near future.
Above all, it would be blocking the money too, if the plans and projects demand
changes every now and then. For example, investing thoughtlessly in highways
and good roads may need drastic changes owing to the increasing traffic. Thus, it
would be wiser to anticipate the future of traffic on roads and accordingly invest
in the different diversions on busy junctions, by-lanes and flyovers wherever
necessary.
Challenges like illiteracy need to be taken care of immediately along with long
term planning. If it is necessary to make the adults of today aware of the
importance of education, then the government needs to anticipate the future and
provide requisite facilities such that sending their children to educational
institutions disappoints no one. Similarly, the law and order problems need to be
tackled at times by an immediate amendment in the law. The provision of
amendments in laws should be made with a futuristic approach such that the
offshoots of similar situations may be handled as well.

29

The issues of poverty, natural calamities, terrorism, need immediate attention. If


only plans for future are drafted to reduce poverty without taking care of
immediate poor and malnourished, then the futuristic goals make no sense. In a
similar fashion, if means are adopted to avert the situations of flood and famine
without looking after the present victims of such natural disasters, then such a
government is a disaster itself. The vice-versa being equally true, it is a sheer
wastage of time, money and effort to wait for a disaster to occur, in order to
provide relief to the victims. Thus, the government has a huge responsibility of
focusing on both immediate problems of today and plan such that the anticipated
problems of future are also efficiently dealt with.



19. Laws should be flexible enough to take account of various circumstances,
times, and places.
Laws are the basic principles of our day-to-day life; hence, behavior of people
should not go beyond the lines confined by laws. Laws are strong weapons to
maintain peace and harmony in a society. However, everyday and every situation
can be different. You cannot generalize the circumstances beyond a certain limit.
It is the same with laws. You cannot apply the same law in each situation. It has
to be flexible. Change and flexibility are required for growth.
In general, there are many laws. Violation of laws can have different forms. There
are different laws for different situations. However, within a particular type of
crime also, the situation might be different. Hence, while taking a decision, one
has to take decisions keeping in mind circumstances of a particular case. For
example, it has to be considered whether a person has made an offence
deliberately, unintentionally, just for fun or for money.
There are many examples in history, which make you think in favor of flexibility
of laws. The latest law is one child rule in China. This law was made to control
population in China. According to this law in China, each couple can have only
one child. Now after the earthquake, the Chinese government is thinking about
revising this law. Hence, as a society is growing and developing, proper changes
in laws are required. Laws have to keep pace with the changing times.
Another example of a law changing with time is the law related to the legal heir.
In ancient times, only the oldest son or only the sons of the family had the right
to the family property. As time has progressed and women have started asking
for their rights, law has changed and now all the offspring have equal right to the
family wealth. Nowadays, another aspect in this context also plays a role. That is
the will of the dying person. Family wealth is distributed according to the will of
the dying person.

30

Similarly, you cannot give the same punishment to two persons who have killed
some other persons for different reasons. While one person might have killed for
some personal gain and the other person could have killed to save his own life.
Both the situations are entirely different. There is no doubt that fixed laws help
the judges in determining whether a person is guilty or not. However, it also
forces people to follow laws, as they are afraid of punishments. It instills the fear
in their minds that if they do some wrong, they will be punished.
Finally, it can be said that there should be a balance between the two aspects.
There should be a fixed law for every crime to guide the judges and the
government to install rules and regulations. It gives the feeling of security and
stability. However, at the same time, there should be flexibility when it comes to
applying the laws in different circumstances. There should be a scope of
modifying the existing and fading laws to give way to the development of the
society and justice to people.


20. Claim: The best way to understand the character of a society is to examine
the character of the men and women that the society chooses as its heroes
or its role models.
Reason: Heroes and role models reveal a society's highest ideals.

The character of the people who form that society determines the character of a
society. A society idolizes certain men and women as its heroes and heroines
depending on what these people have done for the society. These people may be
politicians or they may have brought about a radical change in the way people
live in that society. Irrespective of what they have done for the society, their
characters cannot determine the character of the entire society as these people
will have their own individual characteristics that cannot reflect the characters
of all the people who are a part of that society. This is because each individual
has his own perception about the things around him and this perception shapes
his character. There may be some cases wherein people try to emulate their
idols, but this does not mean that the entire society's character can be
understood by studying the character of its heroes and heroines.
The news channels and the newspapers are always following well-known people
for a peep into their personal lives. Lady Diana was adored and loved by millions
of people across the world. She was the idol of not only her country but also
many other countries around the world. There have been numerous occasions
where people have maligned her name by associating her with scandals even
after her death. If these accusations are true, then do the societies that worship
her have the same character? What about the heroes and heroines who are
respected and loved for the characters that they portray in movies? It is a well-
known fact that the lives of models, actors and artists are laced with scandals
and controversies. Some of them are known drug-addicts and others lead
amorous lives. Does the society that idolizes these people have the same

31

character? It would be wrong to assume that the society has the same character
as it is an accepted fact that these people are liked for their values other than
their characters. Therefore, it is not necessary that the entire society behaves in a
manner similar to the people who are loved by the masses.
Another example is that of the political leaders who have been elected by the
people of a country to lead them. If they have been elected because of their
honesty and their patriotism, then does this mean that the entire society is
honest? On the contrary, what if one of the politicians is corrupt and
unscrupulous? Will this reflect the character of the people who have elected him?
The people who have elected this particular politician may be conservative,
honest and God-fearing, but it is unfortunate that the true character of the
politician whom they have elected as their leader is hidden from them. Even if
they can see his true form after he has already been elected, a close examination
of his character would reveal a character that would be far from that of the
society that has chosen him.
In view of the above, it can easily be concluded that the character of a society and
that of the people idolized by the society are not related in any way whatsoever.
There may be some cases wherein someone has tried to imitate the character of
the person he considers a hero. However, on the whole, the character of a society
is independent of the character of the people chosen as its heroes and heroines.
In other words, the character of the heroes and heroines is not an indication of
the character of the society that has chosen them.


21. Governments should place few, if any, restrictions on scientific research
and development.

We have all heard, "Science is a necessary evil." Why was this saying
coined? The reason is evident as all research work in science has two aspects.
The results of scientific research can be used both for constructive purposes as
well as for destructive purposes. The probability of the result of a scientific
research being misused assumes greater importance when private companies
and organizations are carrying out the research. Therefore, it is imperative that
the government puts some form of restrictions on scientific research and
development.
Research work in the field of nuclear science has always been a controversial
issue. The energy produced by nuclear reactions can be tapped for constructive
purposes like generation of power. At the same time, this nuclear energy can also
be utilized for developing weapons for mass destruction. If a private company
carries out research for fabricating nuclear weapons, then it is bound to make a
lot of profit by selling these weapons to revolutionary outfits and other
countries. This would lead to a volatile situation wherein the power to cause
destruction on a large scale would be available with numerous organizations. Is

32

this acceptable? We all know the aftermath of the atomic explosions at


Hiroshima and Nagasaki. Can we afford to let it happen again? It would be worse
if it happens at the hands of a terrorist outfit. The memory of the terror attack on
the World Trade Center towers in New York is still fresh in our minds. The
widespread destruction as the result of a probable terror attack based on nuclear
weapons confirms the fact that there is a requirement of restricting the scientific
research and development of nuclear energy.
One cannot refute the fact that the greatest contributions to the progress of
mankind have been made by scientists who carried out scientific research on
their own without having to seek permission from their respective governments.
However, the scenario today is entirely different from what existed centuries
ago. In the present times, the quest to earn huge amounts of money and fame has
reached alarming proportions. People are willing to go to all possible extents to
make their lives more comfortable. Surgeons have carried out research work and
developed compounds like Botox that can wipe years off your face by a couple of
injections. This may be useful for people who are associated with the glamour
world and who earn their livelihood because of the youthful appeal of their faces.
However, what about the common people who are now getting addicted to
annual Botox administrations? Won't these have side effects? For how long can
one conquer age? If one does intend to remain youthful all his life, then is
mutating the genes of human embryos justified?
Research is already underway for cloning animals, but there is widespread
speculation about the results of similar experiments on humans. Is it warranted
to have a baby that is the clone of a popular movie star or a clone of his own
brother or sister? What would be the result if such a technology falls into the
wrong hands? People can use this for creating clones that will carry out crimes
by impersonations. All this will be possible if the research work on human DNA
is allowed to be carried out without any restrictions. At the same time, one
cannot deny the fact that extensive research on human DNA has led to the
emergence of the stem cell technology that can cure you from terminal diseases
later on in life.
In view of the above, it is evident that putting stringent regulations on scientific
research work would not be a wise decision. Nevertheless, there should be some
restrictions applied by the government on all forms of scientific research to
ensure that the technology being developed will be used for the betterment of
mankind.





33

22. The luxuries and conveniences of contemporary life prevent people from
developing into truly strong and independent individuals.

As per the author, people's independence is deteriorated by use of
modern facilities. In some aspects, this is true, as people have become more
dependent. However, we cannot neglect the importance of many things in our
lives. Moreover, despite the use of the facilities, humans are still very strong and
independent beings. It is just that whenever people see some crimes happening
around, they blame the luxurious and convenient life for the fecklessness,
irresponsible behavior, and independence of contemporary individuals.
However, it is not the luxuries and conveniences of contemporary life that
hampers the healthy development of individuals; rather it is the attitudes one
has of life and his personality that guides him through moral development.
Before discussing this topic, one should be clear about what luxury is. Luxury is
something inessential and expensive but conducive to pleasure and comfort. In
the modern times, people, especially the urban population, are enjoying the
luxurious and convenient lives. Ironically, people in cities encounter much
difficulty in developing themselves into truly strong and independent
personalities in the modern "harmless" life. Technological advancement has
brought humans many convenient facilities, which are useful if a person does not
depend on them completely. These facilities help humans to overcome physical
difficulties, which were otherwise hard to conquest by mere human power.
Through luxuries, people could overcome difficulties caused by geography,
weather, or time for example, car, cell phone, computer, airplanes etc. Cars and
airplanes take people from one place to another in less time. However, people
start depending on them too much. They get into the habit of using a car for just
short trips to a store at a corner. They avoid walking down even short distances.
People prefer to go to gyms instead of walking or riding a cycle.
Many people spend much of their time with facilities like television and
habitually use mobile phones. These people get so much addicted to these things
that absence of these facilities makes them feel insecure. However, this partial
dependency does not prove that people are less strong or are weakened
mentally or emotionally. A person with a strong personality and positive attitude
toward life is always well prepared for any challenge and competition. While
those with characteristic deficiencies or negative attitudes towards life are
always blaming the luxuries or poverty for the failure without examining the
actual causes that may be lying in themselves.
However, we cannot neglect the effect of the improved life on the developing of
individual traits. Without proper guidance, today's youth may well take it for
granted that there is no necessity for hard work and responsibility since the
convenient life is already at hand. Hence, they should be explained that
improvement of life is the contribution of every individual's hard work, and that
no one deserves it unless he also does his contribution. In order to achieve the
strength and soundness of personality, one should set up healthy attitudes to life
and chasten himself through competitions and failures.

34

23. The surest indicator of a great nation is represented not by the


achievements of its rulers, artists, or scientists, but by the general welfare
of its people.

Some people believe that in order to thrive, a society must put its own overall
success before the well-being of its individual citizens. Others believe that the
well-being of a society can only be measured by the general welfare of all its
people.

How is a nation classified as great? Does this classification depend on the
achievements of the rulers; scientists and artists that the nation has produced or
does it depend on the satisfaction and happiness of the citizens of the nation? It
is evident that a nation cannot be termed as a great nation if there is widespread
discontent amongst its citizens no matter how successful the nation has been in
producing illustrious rulers, scientists and artists. Therefore, the general welfare
of the people of a nation is definitely an indicator of the greatness of a nation.
A nation marches ahead on the path to progress with support from the
contributions made by its people. The citizens need to be employed, literate,
healthy, satisfied and happy in order to assist the nation on its way to progress. A
nation plagued by poverty, famine, slow economic growth, unemployment will
have to deal with several related problems that will make it difficult for the
nation to progress and to be known as a great nation.
People who are below the poverty line will resort to unscrupulous means to
make money. Unemployed youth can easily be lured into carrying out criminal
activities because of the probability of high monetary gains. Corruption would be
rampant and we all know the effect that corruption has on the well-being and
progress of a nation. Living in the country would no longer be safe either for the
citizens or for the tourists visiting the country. The nation may boast of several
well-known scientists, artists and rulers, but will the achievements of these
people help in improving the image of the nation in the eyes of the world? The
answer to this question would be a definite, "No". People judge the greatness of a
nation by the living conditions of the nation and the greatness of some of its
citizens makes little or no contribution to the greatness of the nation.
Breakthroughs in the world of science and technology made by the scientists of a
nation cannot do much for the well-being of the nation unless the technology is
made use of for improving the living conditions of the citizens of the nation. For
instance, if a team of scientists from a developing country makes a path breaking
discovery in the field of generation of power, the nation can progress ahead only
if the technology is applied for generation of power within the nation. The team
of scientists may move to another developed nation for further research work or
the nation may not have the necessary funds to implement the newly discovered
technology for the betterment of its citizens. In either of the cases, the scientists
would not be able to contribute anything towards the progress of their nation.

35

A nation is determined by its people. If the people are looked after well by their
leaders and their welfare is always kept in mind whenever major political
decisions are taken, then it is evident that the nation will progress on the path,
which leads to greatness. This is because the well-being of a nation is directly
related to the well-being and welfare of its people who constitute the nation. The
greatness of a nation cannot be determined by a handful of people who have
gained popularity and fame because of their achievements in the field of science
and technology or politics.



24. The best way to teachwhether as an educator, employer, or parentis to
praise positive actions and ignore negative ones.

Encouragement helps a great deal in bringing up any quality. A positive
feedback can boost the morale to a great extent. Such learning that is backed
with positive feedback and encouragement is effective and faster. However, I
disagree that negative actions should be ignored. Along with praising positive
actions, it is very important to point out the negative ones so that they are not
repeated.
Teaching involves inculcating good habits and eliminating bad ones. It is a
process of refining the personality and imparting correct knowledge. If it does
not fulfill any one of the two things, it is incomplete and ineffective. Education
can be imparted in many ways. However, the point is not just to make the
students learn their lessons anyhow but to make them incorporate what they
learn in their lives. Moreover, education also involves teaching those things that
the child is not able to pick up on his own. This can be done only if they are
taught in a proper way. Children in school are in their growing age. What they
are taught in school goes on for all their lives. Therefore, it is important that
along with appreciating their good actions they are told about the places where
they are wrong. For example, a child may be brilliant in studies and may score
well in all subjects. However, he may not be friendly and polite with other
students. He could be a bully or rude to others. Here it is not only important to
encourage the child to do well academically but it is very important to teach him
how to behave. If this is not done, he will grow up to be an unfriendly person
who is not accepted in the society. This will eventually make him a loner since no
one will want to interact with him.
Similarly, parents have a very important role in bringing up their children. Home
is the first school of a child. A child has no knowledge of what is right or wrong.
He picks up whatever he sees around him. This may be good or bad. It is the duty
of parents to check where their child is going wrong. They should teach him
values and morals. While students are under the observation of educators for
fixed hours, a child is under constant observation of his parents. Therefore, there

36

is increased responsibility of a parent to observe the actions that are not


accepted and correct them. I would like to add here that correcting such an
action does not mean that the child is scolded, rebuked or beaten to put him on
the right tract. In fact, such behavior can make the child more stubborn and
difficult to handle. Negative criticism need not be done in a harsh manner. It can
be done by making the child understand why a certain action is wrong.
The habits as well as knowledge a child picks up in his growing years go a long
way. It goes to his benefit if he inculcates good habits and correct education.
However, this does not mean that he can never go wrong. For example, for an
employer it is very important to check the performance of his employees. The
employees should be appreciated and encouraged to keep up the good work. On
the other hand, it is possible that they make wrong decisions at some point of
time. Instead of ignoring these mistakes, an employer must help them to come to
the right decision. This will help in the professional success of the individual as
well as of the employer he is working for.
In the end, I would like to maintain that correct and the best teaching is where a
child is given the knowledge of right and wrong. He should not only be
appreciated for the right behavior but also be told about the places where he
goes wrong. This is the essence of teaching.



25. Teachers' salaries should be based on their students' academic
performance.

The imparting of knowledge in formal learning institutions are done
through teachers. They, a coterie of qualified professionals trained in the art of
disseminating knowledge, have a major influence on a student's life. A teacher
can make the most boring of subjects interesting through a well-thought out
teaching plan and interesting charisma, so can it be the other way around. Hellen
Keller found her way back into life through a dedicated and caring teacher.
However in view of the statement tying teachers' salaries to their students'
academic performance, I fundamentally tend to disagree with the speaker's
assertions due to reasons stated below.
The first reason of my belief is that there is a lack of a standard academic
performance test in place. The question that will definitely arise is what is the
best method or test to gauge a student's academic performance. If we were to go
by tests carried out by respective schools, we will be certain that the standard
would differ and thus there would not be a standard gauging scheme. The tests
that abound today are mostly aptitude test, for instance the SAT (scholastic
aptitude test), which primarily measures the students' grasp on vocabulary,
reading comprehension and mathematical prowess. It is a foregone conclusion
that academia does also include Geography, History, Science etc. and there are no

37

standardized tests out there for them as of yet. The next problem that would
arise is what about the group of students who are taking a minority subject, say
for example anthropology? The judging scheme would thus be different since it
does not take into account the entire schooling population but rather, involve
only a small minority group. Would the assessment be any different since a
smaller number of sample size is affected? My first point of the lack of standard
gauging tests thus hinges on this argument, which clearly show it would be
unwise and untenable to tie the teacher's salary to the students' performance.
In addition to that, if a teacher's salary were to be based on students' academic
performance, this would bring about a biased way of giving of marks. We have to
be largely aware of the fact that students' test papers are graded by teachers, and
if a teacher receives a higher salary were the student to perform well, needless to
say there would be many unscrupulous individuals who would accord their
students higher marks to their own benefit. Critics might argue that assessment
of a particular class of students can be done by a different teacher, but I foresee a
system that would be plagued with corruption and bribery. What if the class
teacher bribes another teacher, or what if the teacher intentionally gives bad
grades because they know that it would affect the class teacher (who might be
someone they do not favor)? Many what-ifs might take place, which would
ultimately jeopardize and challenge the education system. In light of that, I think
that the statement is not valid and sound.
Furthermore the statement can only hold true assuming that all teachers get
students who have the same level of intelligence, commitment, determination
etc. Teachers who are teaching better performing classes would most definitely
have the upper hand since their students would do better compared to students
of other classes. Even if we were to base it on the percentage of improvement of
a student's grade, we have to similarly take into account that each individual is
different. In a poor performing class, the improvement ratio of a student willing
to learn and another who is not concerned with academia would definitely yield
different percentages. So the validity of the statement essentially falls back on a
student, for he/her through his attitude can affect the teacher's salary, which
sounds outright fallacious.
In a nutshell, based on all the reasons provided, I find myself disagreeing with
the writer's assertions. As of now, the lack of a standardized test and the many
variable factors that are in place would not make this plan an appropriate one.





38

26. Society should make efforts to save endangered species only if the
potential extinction of those species is the result of human activities.

It is the responsibility of humans to maintain the ecological balance of our planet
by virtue of their being the most intelligent creatures on earth. Moreover, Man
has the necessary power to save endangered species to ensure that the
ecological balance of the Earth is not disturbed. However, there is no apparent
reason for society to save endangered species at costs of money and jobs that are
very high. Therefore, it is essential that society makes all possible efforts to save
endangered species, but if the costs involved are sky high, then the society can
choose not to save the species and save the human costs instead.
We no longer see Dinosaurs on the planet whereas they freely roamed on Earth
eons ago. Has their non-existence made any difference to life on Earth? It can be
accepted that their disappearance would have made some difference to the food
chain, as they were one of the most prominent predators. However, in today's
context their absence has no effect on the food chain existing on the planet.
Similarly, it is highly probable that the extinction of a particular species of
animals may be of no consequence in the future. Therefore, it would not be
justified to spend an enormous amount of money in trying to save the species. On
the other hand, if the species can be saved without much investment, then
society should do its bit to preserve the environment and the animals that live
therein.
The same line of reasoning may not hold true for certain types of plants and
animals. This is because apart from providing nutrition, certain species of plants
and animals have been known to have therapeutic advantages including herbal
remedies. For instance, cod-liver oil is well known for its content of Omega 3
fatty acids that help you fight against heart disease, cancer, depression, arthritis,
Alzheimer's disease, ulcers, diabetes and a host of other diseases. Plants
containing Aloe Vera have now been found to have extremely useful healing
properties and all the species containing Aloe Vera are now in high demand.
Therefore, society may have to make an attempt to save any of these species if
they are endangered, but technology has now developed so much that one can
look for other alternatives if the cost of saving these species goes beyond what
society can easily afford.
There are certain natural events that are beyond the control of society. Recurring
floods, storms, forest fires, climatic changes etc. may be playing havoc with the
existence of certain types of rare species. There is little that society can do to
contain such events in such a way that the species are preserved. There will be
the requirement of pulling in maximum resources for making efforts to save
every endangered species. Society will need to go to extreme extents in case of
attempting to save endangered species that exist at high altitudes or at ocean
floors. Therefore, in a majority of cases, society should avoid making an attempt
to save endangered species that would require colossal efforts.

39

The fact remains that in the face of catastrophic chain effects as a result of the
extinction of a particular plant or animal, it might become necessary for society
to save the species. Other than this reason, there is no evident reason for which
society should invest huge amounts of money and other human costs to save
each and every species of plants and animals.


27. As we acquire more knowledge, things do not become more
comprehensible, but more complex and mysterious.

Knowledge is the state of being enlightened. It is acquiring information and
skills, which help you to understand things around you in a better way. It also
helps in bringing perfection in a person's work and life. Knowledge is gained
consciously as well as unconsciously. While one makes an effort to acquire
knowledge through getting educated and becoming learned it is also through
experience that one becomes knowledgeable. It is however questionable as to
why gaining knowledge is important. As discussed earlier, it makes you
understand your surroundings better and bring perfection in your life. Things
become more comprehensible and you gain confidence. However, as a person
gains more and more knowledge, it can lead to details, complexities and
mysteries, but it is only with more efforts that new study comes to light.
When a child comes into this world, he is completely ignorant and therefore is
dependent on his parents. As he grows up, he observes things around him.
Slowly he gets the knowledge about what is good and what is bad for him. It is
only when he gains the knowledge of his surroundings that he is capable of being
on his own. He comes to know that by putting his hand in a fire, he might get
burnt, and by playing with sharp objects he can hurt himself. In case this does
not happen, he would remain dependent upon others all his life. Therefore,
knowledge makes you independent and practical in life.
It is with exercising one's brain and gaining knowledge that we are living in this
age. Without the knowledge of agriculture, we would not be able to produce food
for ourselves, without the knowledge of iron we would not be able to set up
industries. Similarly, the knowledge of human body and medicines helps us to
keep fit and live a healthy life. Such things are basic to our life and life is
incomprehensible without them.
Apart from the basic knowledge of things, one needs to acquire specific
knowledge as well. That makes him survive in the world by making a place for
himself. For example, if a person gains the knowledge of carpentry, he can extend
his knowledge to others who do not have this specific skill and earn his living.
A person goes on acquiring information unless he is satisfied. However, there are
unlimited things unknown to man. There are a number of things that are
unexplainable and accepted as they are. It is only with the efforts of someone
that we can have an explanation of these things. We did not know anything about

40

the universe unless it was explored. The biggest of inventions and discoveries
have been possible only by indulging in them. If we retreat our steps for the fear
of getting jumbled up in complexities, new inventions and discoveries would
never be possible.
However, in the pursuit of acquiring knowledge, one must not lose ground and
get lost in the complexities. While it is important to gain knowledge, it is also
important to keep your balance and not get lost in finding solutions to
mysterious and complex situations.


28. Some people believe that scientific discoveries have given us a much better
understanding of the world around us. Others believe that science has
revealed to us that the world is infinitely more complex than we ever
realized.

It differs from person to person what level of understanding of the world they
are expecting science to give them, which divides them in opinion. If knowing
everything about nature what everything is, why they are, what determine
their behavior and why so is the quest, then science has its limits in answering.
Science can tell you what an electric field does to a charge, but not what it is
exactly, and why it is. These questions and their answers lie beyond the realm of
scientific discoveries.
On the contrary, how nature works, the basic laws governing the phenomenon
we encounter, is the understanding scientific discoveries can provide us with.
Science is the study of nature, what can be observed and measured by us. Thus,
through measurements and observation, scientific discoveries have explained
the phenomenon around the world. For example, in the Southern Hemisphere,
colored curtain-like patches are observed in the night sky; scientific community
has explained that this is due to motion of charged particles in the magnetic field
of earth and ionizing the particles in the earths atmosphere. This very motion of
charges in magnetic field is used as the principle for moving them in cyclotron
and the famous LHC.
There are many more examples showing how our understanding of the world is
affected by the scientific discoveries. The discovery of genes, being the cause of
traits of organism, gave way to genetic engineering, cloning, mix-breeding,
designer baby and what not. This made us understand the basis of some our
human traits. Everyday new researches are carried out, discovering which
hormones play role in occurrence of which emotions and feeling, adding to our
knowledge of human mind and heart.
Science has not just explained physical and psychological phenomenon, but is
also probing the materials available around, that made life possible, in the first
place. NASA has undertaken various missions to search signs of extra-terrestrial
life on our moon, Mars, and now Europa. They study rock samples, availability of

41

water, and the atmosphere, and thus, adding to our vision of the world. Back to
earth, there are various investigations going on to understand the ocean life,
minerals, and the magnetic field of the Earth and the inner layers of it. These are
adding to what we know about the world, and thus we are using them in our
stride.
The direct evidence of our knowledge of the world is the technology we have in
hand. If ever we lacked in understanding, we, particularly the engineers and
scientific community, could not have used scientific discoveries to make our lives
easier. With the knowledge of light being an electromagnetic wave, came the era
of fast communication, Internet. With the discovery of constant speed of light,
through the Special Theory of Relativity, came GPS system in service. Further,
understanding semiconductor brought the digitalization of every gadget and
their shrinking dimensions, followed by nanotechnology. All this is a gift of
understanding the world through scientific knowledge.
Given the restricted span of science, limited to what can be observed, it has given
us much better understanding of the world around us. Every aspect of nature-
phenomenon, organisms and materials, has been exploited by man to make his
life easier, through development of technology, which wouldnt have been
possible without knowing nature.



29. In any situation, progress requires discussion among people who have
contrasting points of view.

It is always a good thing to have a healthy discussion wherein people have
contrasting points of view. Such a discussion enables everyone participating in
the discussion to address the issue from the perspective of the others. However,
can you force someone else to agree to your viewpoint? On the contrary, how
many times have you agreed with the viewpoint of the other person or have you
ever agreed that you are wrong and that the other person is right? We as humans
always try to stick to our viewpoint and rarely admit that we are wrong.
Therefore, can a discussion among people with contrasting points of view
proceed in the right direction? It is true that everyone will learn something new
as he sees the issue from a different angle, but it would be wrong to assume that
such a discussion can actually lead to progress and that this would be the best
way to make progress. On the contrary, it is very rare to make progress in the
right direction when the participants of a discussion do not see eye to eye with
each other on the issue being discussed.
Consider the example of a committee that has been constituted to work out the
modalities for the implementation of a publicity campaign. What do you expect
will happen if the members of the committee have contrasting viewpoints on the
method to be adopted for the publicity campaign? If the members of the

42

committee do not agree to a common method for the launch of the publicity
campaign, how can one expect the details of the campaign to be worked out? The
members of the committee who are of the view that spending money on
advertising in the print media is a total wastage of money will definitely oppose
those who think that the print media is as important as the television media.
Each member will try his best to criticize the viewpoint of the other members in
a bid to defend his own viewpoint. The discussion can progress further only if all
members agree on a common method and then only can the other details be
worked out.
There can be no progress if the participants of the discussion stick to their
viewpoints, which are in stark contrast to those of the others. Such a discussion
would lead to total chaos and wastage of time. The only solution lies in either
forcing some of the participants to agree to the viewpoints of the others or
finding a solution that both parties agree to. Isn't this always the best method to
steer a discussion in a direction that will lead to progress? We have always heard
of cases wherein discussions were fruitful because the participants either agreed
on an issue unanimously or found a common ground where everyone agreed.
Whenever a discussion goes out of hand, the person chairing the discussion opts
for carrying out voting to arrive at the solution, which is agreed to by a majority
of the participants. This is done because it is certain that a discussion cannot
progress in the right direction if the participants have contrasting points of view.


30. Governments should not fund any scientific research whose consequences
are unclear.

The government of a country shoulders a great responsibility of keeping its
populace happy. For the betterment of its people, there are times when the
government has to make decisions that are met with severe criticism. There are
chances that some decisions go against the nation in the long run and the
government is later apologetic about it. In some cases, the impact of a decision of
one country is felt by other nations as well. It is, thus, very essential that a
government keeps in mind the goodwill of its people, and the world as a whole,
before any decisions are taken. This is exceptionally true for a government in
matters related with scientific research. Where I do not completely agree with
the views of the author, I cannot disagree with him either. In my view, deep
contemplation should precede any decision-making regarding research and
regular checks during the research should ensure that medical and ethical
consequences are foreseen.
It is only through research that we can make progress. Scientific research is like
a chain reaction and one invention or discovery leads to another. In most cases,
discoveries are accidental but inventions are a result of directed efforts. A
discovery in itself is not sufficient, it is only after research is carried out that the
true potential of a thing is realized and exploited. One cannot limit oneself while

43

researching unless there are set objectives that one is trying to achieve. Scientific
research is not only for the purpose of inventing something new; it is, in fact, a
means of increasing one's knowledge about the various advancements taking
place all around the world. How else would any country be able to make use of
any development-taking place anywhere in the world?
Since the progress of any nation depends on its government, it becomes
obligatory for the government to fund research work. Where funding projects is
important, it is equally vital for the government to keep a track about whatever
research is happening. There are some medical and ethical questions that are
always associated with research work, but mostly we have to overlook such
issues. For instance, one cannot do away with experimenting on rats for research
purpose until an equally good substitute is found. Besides, a government has to
decide pragmatically what gains priority.
In most cases, the results of research work are unclear. It is only when
experimentation is carried out and observations are made, that the true value of
a research is brought out. It is nearly impossible to work upon a subject if the
minds of researchers are restricted and their funds are limited. In my view,
research should be funded, but the medical and ethical consequences should be
weighed from time to time. It is not wise to freeze funds before any headway is
made and consequences are apparent. Take for instance cloning. We all know
that cloning of humans would lead to unimaginable tribulations. There is no
point in a country's investing in such research. Nonetheless, cloning endangered
species shall help conserve them. For this purpose, it becomes necessary for a
country to research in this direction.
In today's world, it is essential that every nation be independent in terms of
scientific research and technological advancement. Countries that have
progressed in science and technology are the ones that are the most developed
and powerful. Some developing and underdeveloped nations lack the
infrastructure and funds to support research while others like U.S.A. are leaders
in scientific development. At the same time, the governing body of a country
should be abreast with what the scientific fraternity of the country is doing and
should impose restrictions wherever necessary.







44

31. Society should identify those children who have special talents and provide
training for them at an early age to develop their talents.

This statement rightly states that if children with special talents get training at
an early stage, then they can do wonders in their respective fields. It is necessary
to identify these children to give them the right direction. Otherwise, if these
children do not get the chance and the facilities, they might be lost and it will be a
loss to the society.
Sometimes, gifted children do not have means to develop their talents. Hence, it
is the society or government's duty to nurture their talents. It can be access to
musical instruments or providing education. If a child has a special talent in
scientific facts and researches, then he needs access to science labs. In the realms
of science, especially in physics, it is important for society to pay more attention
on genius kids so that they can eventually excel in research in the future. If one
gets the facilities to grow then he might prove a prodigy for the society.
Similarly, students with outstanding talents for sports should be selected and
trained separately. It will for make sure that they show their special talents to
the maximum extent. Special training can polish a child and can produce an
extraordinary athlete. The children need to be trained at an early stage and they
need special coaching to win medals. They need special coaches. If they get the
right guidance, they can smash all the previous records.
There should be special schools where students at the same level should be
admitted and special training should be provided to them. In a free academic
atmosphere, students will feel motivated, a sense of competition will be there,
and it will constantly stimulate him or her to advance in the specific areas of his
interest. This facility cannot be provided by common schools.
However, giving attention to the children with special abilities should not mean
neglecting other kids. Providing facilities to specially gifted kids should not be at
the expense of other children. Giving attention to some children might divide the
children into fractions, which can cause ill effects on the delicate minds of the
children. It would be undemocratic to give preference to some kids over others.
The un-chosen children might feel neglected and inferior. That would lead to
childhood trouble.
The children who will get more attention and facilities might have the feeling of
superiority and they might misuse their talent and facilities to harm the society.
Sometimes, parents also push their children with special talents and affect their
childhood and other times, parents simply refuse to acknowledge their kids'
special talents.
Hence, considering all these factors, it is important for the society to identify
children with special talents and to nurture them. However, they have to take
care not to neglect other children. It should not lead to social and economic
disparity. A well-planned training or a talent program will systematically train
the talented child on his way to reach the full potential at an early age.


32.

45

It is primarily through our identification with social groups that we define


ourselves.
The identity of a person is his characteristics, features or beliefs that help in
distinguishing him from others. It gives the answer to a stranger who is
unknown to you and thus helps in defining your persona. However, one wonders
what the best definition of a person would be. I believe the definition of a person
depends upon the people he is interacting with. While superficially we can
identify ourselves with social groups in order to define ourselves, a deeper
definition becomes explicit, giving importance to individual identity within a
social group. Holding this view, I agree with the statement only partially, and I
would like to add that a person does not only define himself by identifying with a
social group but also by his personal characteristics that differ from others'.
People are generally a part of some social group or the other. This social group
can be identified for its distinct characteristics from another and these
characteristics help the person define his personality. This is the definition of an
individual as taken in the superficial sense. For example, if a student visits a
foreign country for an exchange program, he refers to himself according to his
nationality, which is the social group he belongs to. In an international beauty
contest, we have different contestants from various parts of the world and they
are referred to as Miss Africa, Miss Venezuela etc. Sometimes an individual is a
part of more than two social groups at the same time. That means that a person
can be a Christian as well as an Indian. Belonging to a social group helps to
identify peculiar characteristics of a person with the members of the same group
and distinguish from those of another. It is not only a matter of convenience to
refer to the broader social groups that a person belongs to but also the only thing
that significantly distinguishes a person from others.
On the other hand, I hold that the true definition of a person lies in what he is
when he stands as a single unit, without attaching himself to any social group. It
is this definition of the self which is more important and detailed when it comes
to the description of a person. For example, if a person goes for an interview for a
job and is asked to define himself, he would have to bring out his personal
definition by introducing himself by his name. Further, the employer would like
to know his personal achievements, interests, experiences and skills. It would
not matter to him if the candidate belongs to any social group. Similarly, the
personal achievements of people like Newton and Graham Bell is what helps to
define them.
Further, the view expressed in the statement also does not help to define a
person who does not belong to any social group. There could be many people
who are not socially active and thus do not identify themselves with any group in
particular. The statement does not help in defining these people who however
may establish their personal identities.
In the end I would like to conclude that defining a person by identifying him with
a social group only defines him on a superficial scale. However, on a detailed
scale, it is the personal characteristics, interests, sex, achievements, skills and
ideas that make a person different from the other.

46

33. Claim: When planning courses, educators should take into account the
interests and suggestions of their students.
Reason: Students are more motivated to learn when they are interested in
what they are studying.

There is nothing more beautiful than the sight of an inquisitive young mind. Such
a mind is engrossed in such an academic pursuit that education no longer
becomes a burden but a passion. Course planning which take into account
suggestions from students are bound to result in effective curricula.
Interest is the main motivation to education. If education to a student is merely
an exercise of burden then such education would make no lasting impact on the
life of the student. However, an education that catches the interest of a student
becomes not just an education where it is the duty of the teacher to input
knowledge to the student, but the student makes his responsibility to see that he
is well learned. Education becomes a hobby for the student. The only way to
achieve this is to utilize curricula, which instigate such interest in students.
However, knowledge of such interest inspiring techniques can only come from
student inputs. Student may suggest a game/sport approach to learning.
Educators may incorporate academic materials with video games to gain student
attention. Course materials may be organized into puzzles, which may attract
student attention. Students may also suggest their liking for stories and movies.
Educators could relate course material through story telling (like interesting
history behind a concept) or through documentaries. Also students may also
mention procedures, which they find difficult to cope with perhaps too many
assignments. Educators in this way are able to have a feedback on their teaching
style and objectively judge what means to better their curricula.
However, the downside to taking student input is that sometimes student
suggestions may be borne out of laziness. Elementary and high school students
in particular are usually not bothered with the quality of education they receive
and are may want academic courses which are not effective. Such suggestions do
not have to be taken by educators but these suggestions help notify educators of
the need to find ways of gaining student attention. Also, since humans vary, the
suggestion of various students may be conflicting and adopting all student
contribution may prove impossible for educators to accomplish.





47

34. The greatness of individuals can be decided only by those who live after
them, not by their contemporaries.

How does one decide the greatness of a person? It would be unjust to claim that
that the people who live after that person are the only ones who can pass a
judgment on his greatness. The people who have lived with him are equally
responsible for elevating the person's status to that of a great personality.
Therefore, a person's greatness can be evaluated not only by the people who live
after him, but also by the people who belong to the same generation and are his
contemporaries.
The effects of a person's actions may be seen years after his death. The history of
man is full of the works of such persons who were scorned by their
contemporaries for putting forth ideas that were opposed by the societies of
those times. For instance, the society could not accept the theory of evolution put
forth by Charles Darwin as he related the evolution of man to that of the apes.
However, years after his death, people have elevated the status of Charles
Darwin from a common scientist to that of a great man. This is because he had
been successful in putting forth the theory that has been scientifically proven to
be true years after his death. Galileo Galilei, Copernicus and Kepler all stood by
the theory that the Earth is not static and it is not at the center of the universe
and their contemporaries did not accept their theory. Galileo was even charged
with blasphemy by the Pope. However, today we all know that the earth revolves
around the sun and these scientists have found their way into our history books
as great men.
On the contrary, if one considers the works of artists and writers, it is seen that
most of these people were given due credit for their work by their
contemporaries. Writers like Shakespeare, Mark Twain, Robert Frost etc., were
held in high regard by their contemporaries and these good reviews helped in
making these writers great in their field of work. Painters and musicians like
Leonardo DaVinci, Mozart and Beethoven were all admired by their
contemporaries for their immeasurable talent. Even today, these writers and
artists are accepted as great people because of the appreciation that they had
earned centuries ago.
The efforts of freedom fighters can be appreciated by their contemporaries to a
great extent because they have lived to see the effects of being under foreign rule
and they have also seen the effort and dedication put in by the freedom fighters
to free their nation. Hence, they will be in a better position to decide the
greatness of freedom fighters as compared to the future generations who do not
know the ill effects of being under a foreign rule. Therefore, the people who live
after the freedom fighters have long gone will probably not be able to appreciate
their greatness as they will not be able to identify themselves with the situation
that existed long before they were born.
A careful study of the examples quoted above clearly establishes the fact that it is
not necessary that the greatness of men can be decided by the people who live
after them. There are numerous instances when their contemporaries have
established the greatness of men during their lifetimes. Therefore, great men

48

have been given their due by both their contemporaries and by the people who
lived after them.


35. Students should always question what they are taught instead of accepting
it passively.
Knowledge flows from a learned to a learner. A school or college is an
educational institute where teachers are the learned and students are the
learners. Although, teaching and learning is a two way process between a teacher
and a student, it is the teacher with whom lies the responsibility of imparting
education.
Students are naive and ignorant about the ways and means of gaining
knowledge. There are endless topics and subjects completely new to them. It is
not possible for them to choose and decide what they need to study. Only the
one, who are thorough with these topics, can decide how to make them easy and
interesting for the students at different levels. They are better at selecting or
rejecting a certain portion of the topics, depending on its relevance and
importance.
For instance, in lower classes the learning of formulae in mathematics and
science and rules of grammar in languages may not be comprehended well by
students. It is not easy to explain to them the reasons for those rules, at such an
early age. Even derivation of mathematical formulae for young children will
prove to be a waste of effort. To ease the burden of students, trained
professionals frame the curriculum for different levels. The difficulty level, the
selection of topics, division of lessons along with a link with previous and next
standards is looked after by the experts who frame the curriculum. Students who
do not even have complete knowledge of any topic are not at all in a position to
make a decision.
Instead of being skeptic about the topics being taught, they should welcome all
the new things and clear their doubts wherever necessary. There are times when
the queries of the students cannot be answered due to time constraints or their
inability to understand the real details. It is quite natural that due to an overly
inquisitive and restless mind, some students come up with endless doubts and
seek detailed explanation thereby distracting the whole class from the topic
being discussed. Manipulating or ignoring them gently will be beneficial in
enhancing the concentration on the topic and will also help in completing the
syllabus on time. If for the sake of satisfying the students, teachers do not draw a
limit, then no topic will ever be completed and understood fully. Therefore, along
with the framed curriculum, it is the responsibility of the experienced lot
(teachers) to proceed in the interest of the ignorant students.

49

Moreover, encouraging a skeptic attitude in students would not only develop a


negative attitude in them, but also build arrogance and indiscipline in them. It
may also promote a rebellious nature on the slightest provocation due to their
limited knowledge and emotional behavior. This will harm studies and put
teachers in an awkward position.
Although, at much higher levels of education, students may be given a right to
question over some issues, it cannot be generalized. Thus, students as learners
should have faith in the system and regard their teachers, as they have not
earned the right to question the merits of either of them.



36. The increasingly rapid pace of life today causes more problems than it
solves.

The pace of life is increasing manifold since the past two decades. As our lives
become more comfortable, they become more complicated as well. I agree with
the author's statement that our pace of life today is the reason for many
problems, even though this pace has made us more comfortable.
Contrast the lives we are leading to those of our ancestors. The complete way of
living has undergone a transformation. Not only are our lives more dependent on
gadgets, but also our daily routines, eating habits, priorities, aspirations,
expectations, values, everything is changing at a very swift pace. If we take a
minute to ponder over it, we ourselves would be amazed at how our lives have
secretly changed. When we were younger, there were snail-paced means of
communication, limited sources of knowledge, traveling was more time
consuming, lesser financial security, fewer gadgets, lesser planning etc.
Alternatively, our childhood was more carefree, our parents had more time for
us, our education system was less complicated, the competition was less in every
sphere etc.
The increase in pace is due to many factors that are interrelated. Development in
technology is the core factor responsible for this. It has led to improvement in
communication and hence globalization; the world is smaller and closer than it
ever was. Nonetheless, can globalization be the only reason why our lives are so
different? Certainly not. The other key factors responsible for the present
situation are increased sense of insecurity and competition among people. We
are insecure in terms of finance, health, property, employment, relationships etc.
Couple with this insecurity, our sense of competition. We are constantly trying to
outdo others and gain an edge over them. Be it our friends, colleagues, family
members, neighbors, anyone. We are constantly racing against time to rise
higher, earn more, have a better standard of living, and have more of everything.
In fact, the more we gain, the more we bother about securing it. This is the
vicious circle of our present day fast-paced lives.

50

The aspect of our lives that is the most affected because of this is our own health.
The changes in our lifestyles leave no room for exercise, recreation and
indulgence. There is seldom any time to relax and even if there is any, thanks to
mobile phones and laptops, we are rarely out of reach and out of occupation.
How many times have you socialized with friends without receiving any
unexpected call to interrupt conversations? Sadly, our bodies cannot keep pace
with these changes. Besides an alarming increase in chronic diseases, there is a
lot of stress that people face in their lives. Though the average life expectancy
has increased, it is basically medicines on which our populations are surviving.
Along with physical ailments, our minds are getting increasingly corrupted.
Humility, cooperation, patience, tolerance, truthfulness, selflessness; such
qualities are hardly innate now. People have to consciously make efforts to
inculcate such traits, where they were an integral part of our values in the past.
When it comes to children, parents are happier to see their child competing to be
the best rather than becoming a good human being. Since it is money that
matters now, people are directing their energies in one direction only. It is work,
competition and success all around us with our lives more comfortable yet more
intricate.



37. Claim: It is no longer possible for a society to regard any living man or
woman as a hero.
Reason: The reputation of anyone who is subjected to media scrutiny will
eventually be diminished.

In this age of intensive media coverage, it is no longer possible for a society


to regard any living man or woman as a hero.

Every person has an ideal in his or her life and we all make endeavors to be like
him or her. Most of us have leaders or actors as our ideals. In this present age of
satellite communication it has become possible for us to know about the life
styles of our ideals. We are aware of everything related to their lives rather we
feel that they have become a part of our lives. This all has become possible
because of media and we are able to see our ideals in happiness and in grief. I
believe that in this age of wide media coverage it is possible for us to be well
aware about our celebrities and because of media coverage only we are able to
know them better. Although their reputation is subjected to media scrutiny but
these personalities are more known to common people and more people regard
them as their hero.

51

It was very difficult for people to get to know about their heroes when media
was not introduced. This technological innovation has revolutionized the social
structure. People can follow their heroes all the time. These people are heroes
because we have made them so and without our support they would not be as
famous as they are. Media scrutiny does not affect those social figures that are
indeed heroes in their respective fields. Mahatma Gandhi was one such example
as media scrutinized all his acts but he was such an able man that it did not affect
him at all. Gandhi not only got independence for India but also made a place in
the hearts of people, which will keep him alive in their deeds. He set an example
that non-violence can achieve great results and till date people follow his
principles. There were rumors of Gandhi having rifts with his son but that did
not diminish his image. He was a human being after all and like every father and
son they also had their arguments, which certainly do not make Gandhi a bad
man. In the same manner the actors and leaders are also looked up to by people
as their heroes and if they commit some mistake that certainly do not make them
bad human beings.
Although, it would not be correct on our part to regard our heroes as good or bad
only based on media scrutiny but this is certainly happening these days. It is
entirely up to the media how to portray a situation and if an incident in a
celebrity's life is portrayed in a wrong way then that can prove to be fatal for that
celebrity's fan following. Princess Diana was one such example. She was literally
worshipped by her fans and one incident of wrong media coverage shattered her
image.
Media coverage does plays an important role in making or breaking a person's
image but it is very difficult to misguide public these days. Public also
understands that media is either exaggerating or showing wrong image of
popular personalities. At the same time if the media scrutiny is justified then
certainly the true picture should come in front of public.



38. Competition for high grades seriously limits the quality of learning at all
levels of education.

Today's world is full of competition in every field. There is so much of talent
around, that this competition is inevitable. Whichever field one chooses, whether
its academics, sports, business, politics, entertainment or media, there is a race
that everyone participates in to win.
Although, this is healthy and is a sign of progress, the quality of work suffers at
time. The same is the case with competitions for high grades. Students, at every
level of education are so much pressurized to excel that they start compromising
on the quality of learning.

52

The parents and the education system pressurize students, to excel in each and
every subject in terms of their scores. Instead of giving a choice to pursue for
more knowledge and develop understanding in the subjects of their choice, they
are taught in a time-bound and syllabus-bound system since their early school
days. The evaluation pattern is such that those seeking deeper understanding of
a certain topic/subject are de-motivated. They are judged merely on the basis of
their score, which generally calls for rote learning. The quality of learning is
bound to deteriorate when preference is given to those who can cram and
reproduce, at least till the secondary level of school.
In high school and colleges, in some of the disciplines more emphasis is laid upon
application based education. Although, this is a healthy practice, the learning is
still exam oriented rather than concept forming. Availability of more and more
tutorials that boast of preparing the students for competitive exams emphasize
on short cuts and tricks to attempt the paper in lesser time. The giving away of
such techniques is increasing the number of aspirants who are grilled to get
through without possessing sound knowledge of the subject matter. This further
increases the competition thereby neglecting the actual comprehension and
application of the concepts on a student's behalf. Mostly these competitive exams
end up judging the speedy applications of practiced techniques and not real
learning.
Besides, in a classroom both pupil and teacher are constrained so much by time
and syllabus that the topics covered cannot be explored beyond a point. The
education and evaluation systems are such that pupils, teachers, parents and
even school administrations have to target the exams to excel in the competition.
Less emphasis is laid upon concept building and more on attempting the exams.
The very system of education, evaluation and selection are so flawed that even at
the highest levels of education, more weight age is given to scores when
compared to quality learning.
Though, scholars excelling with quality education can not be ignored, a majority
of students end up being rote learners good at merely reproducing the learned
facts. The deterioration in the quality of education is due to the growing
competition for higher grades. Reforming the education system starting from the
primary level itself can check this. Moreover, methodologies should be adopted
to evaluate the understanding and deeper knowledge of students along with
their abilities to apply the acquired knowledge.






53

39. Educators should teach facts only after their students have studied the
ideas, trends, and concepts that help explain those facts.

Why is understanding a particular concept always given preference over
mugging it up? This is because once you have understood a concept; it is unlikely
that you will forget it that easily. On the other hand, something that has been
memorized by cramming it up will last in your memory for a couple of days only.
The education system stresses a lot on the explanation of concepts and teaching
the ideas and trends that are behind the facts, which have to be learnt. If
understanding the basic concepts was not all that important, then there would
be no requirement of teaching in schools. Students could be given a long list of
facts and they could be told to just memorize them. However, this is not the case.
This is because students will not be able to learn anything by merely memorizing
the facts. They should study the concepts that explain the facts before they
attempt to memorize those facts.
Acquiring knowledge and the application of that knowledge are two different
aspects of learning. It is easy to acquire knowledge by reading and memorizing
facts. This way you cannot become learned as you would be unaware about the
concepts that have led to the establishment of these facts and you will not be
able to apply the knowledge that you have acquired by memorizing. Moreover,
you will not be able to analyze other related aspects on your own as you would
have learnt very little by memorizing facts. For example, simply memorizing the
speeds of light and sound are not as important as understanding the difference
between these two speeds and how they are related to each other. Unless you
are clear about this concept, you will not be able to either understand or explain
the reason behind the lightening phenomenon wherein one sees the lightening
first and hears the thunder later; whereas both these events have occurred
simultaneously. Being able to explain this phenomenon would mean that you
have actually learnt something rather than just being able to rattle out the
figures related to the speeds of light and sound.
Children studying in kindergarten are taught the shapes and sounds of the
alphabets before they are taught the spellings of common words. There is no
school in the world, which would advocate memorizing the spellings of words
before understanding the phonetics related to the alphabets. Even if a child does
memorize the spellings of words without understanding the phonetics involved,
how long will he be able to retain those spellings? Even if he does remember
those spellings for an unusually long time, will he be able to work out the
spellings of more difficult words on his own? The answer to this question would
be a definite 'no'. This means that the child would have learnt almost nothing by
memorizing the spellings without understanding the concepts of alphabets and
phonetics. On the other hand, if he has understood the sounds and shapes of the
alphabets, then he will have no difficulty either in memorizing the spellings of
common words or in reading and writing difficult words later on in life. Hence, it
is obvious that memorizing facts after studying the explanations to those facts
will help the students to learn in a better way.

54

Understanding the system that sustains life and allows us to grow and develop in
this technically savvy world is not very easy. However, you have to make all
efforts to understand these concepts if you want to be well educated in life. It
cannot be said that you have learnt something if you have merely memorized the
facts as you are likely to forget these facts when you are no longer in touch with
your studies. Therefore, the better option would be to gain an in-depth
understanding of whatever is being taught and you are bound to retain the
knowledge gained for the rest of your life.



40. Young people should be encouraged to pursue long-term, realistic goals
rather than seek immediate fame and recognition.

We all aim to achieve great heights in our professional front. Nothing comes easy
in this world. We all have to strive hard to achieve our aim in life. It is very well
said that timely hard work leads to sweet fruit of success. All those who work
hard in their lives taste the sweet fruit of success. The present generation is
forgetting the true meaning of success. The younger generation is aiming at
getting overnight success in their careers. In the present scenario nobody wants
to work hard for success but they want success on the very first step of their
careers. I believe that the younger generation should be shown the true path of
hard work and success as that would lead to their true success. They should be
encouraged to get their final goal in life rather than running after momentarily
success.
We all fight in our lives to prove ourselves in the rat race of earning our
livelihood. We all are in a hurry to prove ourselves in our respective fields. In
this run the true essence of success is getting lost. The present generation is
running after only fame and recognition. This is resulting in adoption of wrong
means to do work to obtain success. The need for success is making people blind
and they are instigated to adopt all the wrong means to fulfill their aim. We have
numerous examples of people from our past generation who struggled in their
lives and came out shining bright in their respective fields. There is no denying
that sheer hard work surely pays. There is a very famous saying that slow and
steady wins the race. It is correct in context to all fields of life. When in life we
are deliberate in making our decisions then the outcome is remarkable as the
decision was well thought over. Therefore, the chances of doing something
wrong are minimized to a great extend. The various technological developments,
which are the result of many years of research and hard work, have made our
day to day life easy and simple. The scientists have put in so much effort and
hard work in inventing all such innovations. Therefore hard work has always
resulted in remarkable achievements.

55


Although the outcome of hard work is always credible but people who get
instant fame and recognition in life are hardworking too. It is the creativity of
those persons that has shown them the doors of success in life. In spite of the fact
that few people are successful early in their life it certainly does not mean that
the way to success is always short. Those who strive hard always get blessed
with success in life. Therefore, younger generation should be encouraged to
achieve their long-term goals rather than running after short-term success.
Short-term success is short lived and long term goals yield true success in life.



41. If a goal is worthy, then any means taken to attain it are justifiable.


There is no doubt that goals are necessary for the growth of an individual and
the development of the society. Every individual dreams to achieve his goal.
However, to say that any means taken to attain it is justifiable is not correct.
There are both positive and negative aspects of this statement. It depends on the
individual whether he/she takes the positive side or the negative side.
Sometimes, based upon the distinct function of the worthy goals, it appears that
any means taken to achieve the purpose is justifiable. However, a balance is
required considering the morality factor.
It is a common accepted fact that goals provide directions to people's actions and
make their actions more purposeful. It helps in improving the conditions of
human beings and the world. Sometimes it is essential to take extreme steps in
some sense; it becomes justifiable to use some extreme tackles to gain the
meaningful goal. For example, independence war of India against British Empire.
Usually wars are considered a serious loss to nations, however, this war was to
eliminate the slavery in India and for setting true democracy in India. It is also
true that the benefits of attaining the goal of this war cannot reverse the effects it
had on the nation's heart and mind. However, it cannot be ignored that it gave
birth to the largest democracy of the world.
However, it cannot be said that one can use any means to achieve his goal. A very
general example is that if a person has a goal to become a millionaire then it does
not mean that he can attain his goal by any means. He cannot be allowed to rob a
bank and say that his goal is now achieved and it is justifiable. On the other hand,
if the goal for an individual is to study in the best university and if he/she works
very hard to get through it and studies day and night without any concern about
what people say, then this can be called justifiable. Hence, it is difficult to predict
whether any goal, which a person is seeking, is worth. At times, maybe some
measures would lead to a side effect and even worsen the situation.

56


People have to take some basic moral standards into account before taking
extreme measures to fulfill their goals. In the absence of logical steps, a valuable
goal loses its meaning. A worthy goal is a purpose that would benefit the entire
humanity and the society. A personal goal should not be above the national or
global cause. For example if you take the instance of cloning, it can achieve a
peerless level in the biology science. However, all the nations are very cautious
to develop this technology as cloning may lead to a chaos in ethic. Moreover,
terrorists can misuse it. Thus, you cannot justify all means of attaining a worthy
goal.
Hence, an individual should consider the means, which help to achieve a worthy
goal. The result should be materialistically as well as morally correct.



42. In order to become well-rounded individuals, all college students should be
required to take courses in which they read poetry, novels, mythology, and
other types of imaginative literature.

The statement claims that students should be required to take courses in which
they can read various types of literature in order to become well-rounded
individuals. It is true that reading poetry, novels or other kinds of literature is
beneficial to students mind and imagination, which might cultivate well-
rounded individuals. However, from my perspective, there is no need to make all
college students attend these courses. Forcing college students to take part in
such courses is likely to have adverse effects since not all students are interested
in reading these literature works.
Reading imaginative literature has some positive influence on students minds.
With imaginative literature, students can obtain some knowledge and thoughts
they can hardly get from reality. They can also cultivate their minds as well as
letting their imagination soar. For instance, by reading Walden, students are
likely to appreciate the beauty of nature and share the tranquility and pleasure
the author once had; by reading Nineteen Eighty-Four, students would
understand freedom is of very great importance and thus be willing to fight for
democracy; by reading Greek mythology, students could experience different
lives of heroes. Therefore, through attending courses in which they can touch
diverse books, college students are more likely to have a positive attitude toward
life and develop right values. Also, reading books not only helps them think
deeply but also inspires their imagination. All of these desirable qualities are
essential for a well-rounded individual.

57

However, letting all college students participate in such courses is a little bit
unrealistic and unnecessary. Not every student likes reading imaginative
literature. Demanding students to take this course might cause adverse results.
After all, interest is the best teacher for every student. Students who are
interested in other fields may become very bored during class and pay little
attention to what they read and what the teacher says. Due to this fact, the aim of
making students become well rounded could hardly be achieved.
In addition, for the sake of cultivating well-rounded individuals, taking courses in
which students read various books is not the only way since other means can
also help students improve imagination, set right values or hone critical thinking
ability. For instance, students attending courses in which they can learn dance,
painting or sculpture are also expected to become well-rounded students since
these courses are very similar to literature courses in some aspects. Therefore,
universities need not require all college students to take literature courses.
In conclusion, if students are very interested in reading, attending these courses
may benefit them since reading can teach students a lot, helping them have a
positive attitude toward life. In contrast, students who are not fond of reading
can hardly become well rounded, even though they attend these courses.
Actually, the recommendation mentioned above is not the only way to develop
all-rounded individuals. Other courses have the same effects on students who
like them.



43. In order for any work of artfor example, a film, a novel, a poem, or a
songto have merit, it must be understandable to most people.

The speaker claims that any work of art-whether film, literature, sculpture, or a
song has merits only if it is understandable to most people. What the author fails
to realize is that popularity and merit are two different things. This statement is
controversial in this time. While the value of art is not related to the accessibility
to people, the acceptability and understandability is important to enlighten.
However, we should agree with the writer that most of the folk art is
understandable to the common people and benefit them in many ways. It is
human nature that they learn from what they understand like fairy tales, films,
music etc. Few people have interest in abstract or obscure art works. Common
people do not understand classical music but when the same is presented in the
form of songs or film music, it attracts them and gets famous. The
comprehensible art works often arouse readers' or audiences' interest or
enthusiasm. Therefore, the essence whether the creators' feeling, ideas, or values
are likely imparted to receivers. A series of Harry Potter films are an example.
These movies have attracted children and adults all over the world alike due to
its recreation and understandability factor. The more understandable the work

58

is, the more popular it is with people and hence more is its value. There is also an
example of fairy tales by a Denmark novelist Andersen. He has used
understandable and vivid interpretation, which has influenced generations of
young children in the world and even shed light on adult world.
However, at the same time, the artwork should not be only for monetary gain
and lucrative and entertaining purpose. If it is so then the writer is correct in
saying that these merits lie in popularity and understandability of art.
Sometimes, the understandable artwork does well also. For example, due to
popularity of Harry Potter films, numerous audiences were attracted to cinemas.
This popularity has also benefited the author of the novel and now the audience
and readers highly value the author as one of the most valuable novelists.
However, there are many instances where several artistic works with great value
cannot be understood and recognized by contemporary people. Sometimes, even
the most sensitive critics are not able to work out and evaluate the true and
actual value of the art. Therefore, it is simply not possible for the general public
to understand its value, who do not receive particular trainings and specialized
knowledge.
There are many examples where the artwork of many artists was not given due
respect and reorganization when the artist was alive. People just could not
recognize their value. However, later on after their death, their work got place in
the history of art. Van Gogh was a famous painter whose work was not
acknowledged by people during his lifetime. He could sell only one of his
paintings at a low price all over his career. However, decades later, people began
to recognize the value of his paintings. Recently, two of his paintings have broken
all records as they were sold at a price of 50 million dollars in an exhibition.
In the end, it can be said that people's opinion may or may not influence the price
and popularity of a certain artwork but it cannot change its merit.



44. Many important discoveries or creations are accidental: it is usually while
seeking the answer to one question that we come across the answer to
another.

Although there are many incidents in history where discoveries occurred by
accident or it happened while searching answers to some other problems,
however, there are many other discoveries, which were sought out purposely.
Hence, we cannot generalize that most discoveries are accidental.
The author has given this statement based on some examples from the history,
but this statement is overstated. In this statement, the author has given more
importance to serendipity, which is not correct.

59


However, you cannot ignore many accidental discoveries in the world. There are
many examples in history to support the author. For instance, Columbus
discovered a new continent, America, while he was on his route to find West
Indies for trade. Similarly, the discovery of very important drug, Penicillin that is
one of the earliest antibiotics, by Fleming was also accidental. There are many
other anthropological examples in the history where geologists stumbled upon
important artifacts in quest to understand earth's structure. These discoveries
tell them about the previous civilizations in those places.
However, these were the cases when technology was not very advanced. Hence,
there were not many inventions, creations, and discoveries on purpose in those
times. Nevertheless, that is not the case now. These days astronomers use more
powerful telescopes to discover new planets and to know more about space.
Because of these discoveries and new technology, we know so much about our
atmosphere and planet. It also helps us to know about the earth's weather in
advance.
There are many planned researches by biologists to discover new medicines and
new therapies for various diseases. Most of the times, scientific discoveries are
planned. Scientists research lot for the existing facts and data before applying the
information to create a new thing. Most of these discoveries are made through a
scientific approach. Drug companies have the pressure of their customers and
shareholders to avoid random pursuits. At the time of its discovery, even
importance and use of penicillin was not known. It was in 1939 that a group of
scientists did intensive research and were able to demonstrate penicillin's ability
to kill infectious bacteria. Hence, this discovery was also used after planned
research.
Similarly, creations are also not baseless and instant. They are more engineered
and based on the existing knowledge. Semiconductors, internal combustion
engines cannot be treated as accidental discoveries. These are based upon the
existing scientific principles. However, there might be some discoveries whose
later effects are not foreseen. For example, ARPANET was started on a small
scale as a data communication system, which was used to share data among
engineers at different universities. However, later on it got famous as Internet.
In these modern days, many companies are working in the business of creating
or discovering new things to make our lives comfortable. They hire many
engineers and scholars to work on these projects. Hence, not all the discoveries
can be known as accidental. We must not give more credit to accidental
discoveries than the actual work done by scholars and researchers.



60

45. The main benefit of the study of history is to dispel the illusion that people
living now are significantly different from people who lived in earlier
times.

This statement is difficult to discuss. The significance of the study of history
cannot be discussed in few words. It would be naive to say that the chief benefit
of the study of history is to break down the illusion that people in one period of
time are significantly different from people who lived at any other time in
history. However, during the process of studying history, people do realize that
they actually share much in common with their ancestors, in spite of various
superficial differences.
During the study of history, people find that the life styles of people are variable
from one period to the other. However, some basic human nature always
remains the same. In the earlier times, people mainly depended on pigeons and
then letters to correspond with others, while in modern times with the advanced
technology, Internet, telephone, television, automobile are the main methods for
us to communicate with others. They used to travel from one place to other on
horses and camels. It is but natural that as the technology changes, there come
changes in the lifestyles of people.
However, no matter in which period one lives, some values and merits always
remain the same and get appreciated by people of that period. No matter how
many changes our society has experienced, some characteristics such as bravery,
self-discipline, righteousness, fidelity to friends, family and nations, spirit of
never giving up, altruism are still praised by people. Also, people's pursuit for
truth and beauty remains the same. The spirit behind gaining the knowledge and
searching for new things also remains the same. The main purpose to do
research and the methods used might be different but they are all aimed to come
closer to truth. For example, the doctrines of Socrates, Plato, and Aristotle are
different and incompatible to each other but the spirit of pursuing the truth has
passed down from one to another.
Similarly, people use different art forms to express their feelings. The form of art
might be different in different periods. However, the essence behind all the art
forms is the same, which is longing for beauty. More or less, modern theories,
technologies, or conceptions cannot avoid being influenced by their
predecessors, and, consequently, they inherit merits and demerits of the past
and establish themselves finally.
The study of history benefits human beings in many ways like it can help us
avoid the same mistakes as our predecessors did. Some mistakes of our
ancestors were disastrous for humanity and societies. In order to ensure the
smooth movement of our society it is must to avoid those mistakes. For instance,
the Second World War was the worst thing that happened to our society and it is
necessary to ensure that this kind of mistake never happens again.

61


Hence, from the perspective of life styles of different people are different from
each other. However, there are many other aspects in which people in any period
of time are quite the same. Therefore, the chief benefit of studying history is not
to break any illusion but to learn from the past and to avoid the mistakes done by
people in the past.



46. Learning is primarily a matter of personal discipline; students cannot be
motivated by school or college alone.


The statement given implies that in any kind of learning whether it is sports,
education or something else, the person who wants to learn must have some
strong will, persistence and other personal discipline abilities and learner cannot
be motivated by motivations by his or her school or colleges alone. As the
statement is a factual truth itself and there are plenty of real life examples that
can be seen in every field of learning, so i would prefer to agree with it.
Firstly, as a matter of fact in any kind of field things like hard work, persistence is
mandatory to enthrall the success. For instance, in field of education every
student is not the same and not everyone gets pure erudite. There are some
students have great hunger of knowledge and they just give their hundred
percent to learning and show persistence in their learning, but other reluctantly
learn who definitely don't get success. So such a simple example shows that self-
discipline is must in learning fields.
Similarly, some students do have the ability to persist learning, but they havent
given proper motivation and they fail to achieve what they want. For example,
some students are neglected in schools by teachers for their outside physical
impression like a student who is interested much in sports and not in studies,
but such student might have great potential in studies also. If their parents or
some other people motivate these students properly then they can perform great
in both sports and studies also. Therefore, motivation plays an important role in
learning process and some students really need properly motivated.
On the other hand, the schools or colleges sometimes also play major role in
helping and motivating a student for learning. A better example would be of
students who are neglected by their parents. Some parents don't care much their
childs school or college and they just blindly believe on the schools and colleges
to take care of students studies. In such cases the schools and colleges do have
great responsibility to motivate the student and lead him to good learning habits.
So sometimes depending on type of student the schools and colleges also play
important role in motivation.

62


On conclusion, the learning is surely the matter of self-discipline in most of the
cases. The schools and colleges do play their role in motivation depending on the
situation of the student, but even with the greatest motivations it is on the
student that how much he or she gets influenced by motivation for learning.



47. Scientists and other researchers should focus their research on areas that
are likely to benefit the greatest number of people.


Research work in any area is carried out to look for answers to existing
questions or to dig for information that was hitherto unknown. How can then
one assess whether the results of the research work would be successful or not?
Even if there is a strong possibility of the results being successful, how can one
ascertain how many people's lives would be improved by these results because
no one knows what the results would be like. The need to provide a high priority
to the question, "How many people's lives will be improved if the results are
successful?" is therefore, highly unjustified.
Before starting research work on any subject, the priorities of the research team
would involve planning the method in which they have to carry out their
research work, the time constraints, the ideal end result that they are looking for
etc. It is true that the effect of the results on the lives of people will also be an
important aspect that has to be considered, but this question cannot be accorded
a high priority, as we would be incapable of accurately judging the number of
people whose lives would be improved. Even if one does arrive at an estimate of
the number of people whose lives would be improved, one will have to take into
account the side effects of the results as well. All the results of any form of
research work can be used for both positive as well as negative purposes. If we
start taking the negative aspects of the results of research work into
consideration, then it is unlikely that we will be able to justify the conduct of a
majority of research works in all fields.
When the research work on atomic energy was being carried out, could there
have been an estimate of the number of people whose lives would be improved
by the results of the research work? On the contrary, millions of people lost their
lives and even more people are living with the aftermath of the atomic
explosions at Hiroshima and Nagasaki. Had the researchers been able to foresee
this effect that the results of their research work had on mankind? There is no
denying the fact that a number of people have benefited from the results of the
said research work as it has led to the generation of power as well. Even if the
negative impact of the research work had been foreseen and it had been given a

63

high priority, then stopping the research work could have led to a scenario
wherein the generation of nuclear power would not have been possible.
The controversial research work on genetic mutations may affect the lives of
millions of people by finding the cures for various terminal diseases. However,
the results of such a research work can also be used for altering the genetic
makeup of humans and animals to create monsters. Hence, it is evident that the
effect of the results of a research work on the lives of people should not be given
a high priority as one cannot predict the turn of events that might take place
once the results are available for common use.
Therefore, it is in the interest of the society that all forms of research work
should be allowed to progress without paying much importance to the number
of people who are likely to see improvements in their lives as a result of the
consequences of the research work.



48. Politicians should pursue common ground and reasonable consensus
rather than elusive ideals.

What is politics? Is politics restricted to the government and the political
leadership that implements policies, which affect the lives of millions of citizens?
Most people consider politics as something that is represented by corruption,
competition and a race to gather votes. However, the truth is that politics is a
game of power. Certain individuals are granted the power and authority to
provide solutions to the existing conflicts in a society or an organization.
Therefore, it is not necessary that politics is limited to the government only. One
can come across politics in an organization or institution that has given its
employees the empowerment to elect a group of people to govern their
functioning and also resolve their conflicts. In either case, the goal of politics has
to be the pursuit of an ideal situation or else the people in power may find it very
difficult to identify the restrictions and policies that are essential for the smooth
functioning of the country or organization.
Searching for common ground and reasonable consensus may lead to issues that
remain unresolved especially since none of the conflicting parties would like to
make an adjustment to their demands. One can see the example of an
organization where the power to make undisputed decisions lies with the boss.
Even in the past, all tribes, empires and countries had leaders and rulers who
made the decisions to resolve conflicts for maintaining peace and harmony in
their reign. There was no need for them to search for common grounds and
reasonable consensus. This was because such a decision could have been cited by
some party at some other time to demand justice.

64

In present times, the elected governments that exist in most countries make the
task of politics much easier. It is the people who have elected the political leaders
to power and they are the ones who can change the leadership at the center if it
does not come up to their expectations. The same holds true for organizations
wherein the employees of the company have elected the top management.
Therefore, the leadership has to carefully set an ideal goal for itself in the form of
rules, regulations and policies. They should not deviate from their ideal even if it
means losing their power. This is a major stumbling block for a majority of
leaders. Who would not like to always remain in power? Therefore, the need to
arrive at a common ground or reasonable consensus arises because all
politicians try their best not to disappoint their vote bank while resolving an
issue. However, in the long run this can prove to be detrimental for the well
being of a nation or organization as a whole, because the society would be living
in conditions that are far from ideal and this is definitely not what was in mind
while electing the leadership at the center.
If there are two parties fighting over an environmental issue or an educational
issue, how should the matter be resolved? Should the leadership favor the
solution that has been arrived at by studying common ground or should the
leadership look for what the ideal solution is in such a scenario and implement
that irrespective of the damage it does to the face-value and vote bank of the
political party? The first solution will effectively lead to the end of the
disagreement between the two parties, but the solution would be far from the
ideal situation that would have been the best for the society. The second solution
would definitely disappoint one of the parties, but the nation would gain from
such a decision as it would lead the society towards an ideal situation, but
unfortunately, it might jolt the leadership out of power. Therefore, despite the
heavy risks involved, the best option for politics is to maintain a goal that is the
pursuit of an ideal no matter how expensive it might prove to be. This is because
attaining such a goal will result in the betterment of the entire society as a whole.



49. People should undertake risky action only after they have carefully
considered its consequences.

The statement claims that people should undertake risky action only after they
have carefully considered its consequences. Under some circumstances, this
recommendation is favorable and beneficial since careful consideration of
consequences would help people steel themselves to manage any likely results.
However, in some cases, people do not have enough time to think carefully. Also,
cautious consideration may prevent people from doing long shots, causing them
to lose great opportunities.

65

It goes without saying that having careful consideration before undertaking risky
action is helpful to people. On the one hand, people are more likely to make right
decisions after enough thinking. On the other hand, considering all anticipated
consequences could assist people to prepare for any outcome. Take warfare for
example. Before war, each side must consider carefully. One side must think
what the other sides action is likely to be and what consequences it might lead
to. With the help of this reflection, commanders are likely to make right decisions
under different consequences. Therefore, cautious consideration is helpful to
people since risky action may cause serious consequences and people should
have enough preparation for the sake of avoiding being at a loss what to do.
However, in some cases, there is not enough time for people to make careful
consideration and thus this recommendation, though commendable, is
sometimes unrealistic and infeasible. Any delay means opportunities are
irredeemably lost. Take battlefield for example again. It is true that before battle
each side should consider carefully its outcomes of every action. However,
because the situation is changing fast and haphazardly, no one can have a
thorough consideration of all consequences. In this case, when the war begins,
some events occurring on a battlefield might require an army respond quickly
and decisively. Otherwise, the opportunities to win the battle are likely to
disappear. In addition, if the house is on fire and someone is trapped in the
burning house, there is no time for a firefighter to think what consequence may
be if he or she goes into the house to rescue this person simply because any
delay would lead to death. Thus, in times of emergency, the statement mentioned
above is unrealistic and impractical.
Moreover, careful consideration may have a negative influence on people,
preventing them from doing any risky action. For instance, if people think most
consequences of this action are desirable except a seriously bad one, they may
become too timid to undertake this action even though the likelihood of success
is great. Being afraid of doing any actions sometimes means losing a great
number of opportunities.
In conclusion, in some cases, the recommendation is desirable for it can help
people prepare for likely consequences. Nevertheless, sometimes, the
recommendation becomes unrealistic. On the one hand, people have not enough
time to consider carefully before doing something risky. On the other hand,
careful consideration may prevent people from undertaking any risky action.






66

50. Leaders are created by the demands that are placed on them.

Who is a leader? A leader is a person who is followed by others due to his
charisma and his ability to lead people in the right direction. A leader must exude
qualities like confidence, integrity, dedication, honesty and competence in
carrying out the tasks related to leadership. A leader has to cater to the demands
of the people who are led by him. However, the contention that the demands of
people can create a leader is not justified as not everyone can inculcate the
qualities that are required of a leader. His people can elect a person as a leader
but he will not be able to succeed in becoming an effective leader unless he has
the inborn traits of leadership. Hence, it is true that leaders cater to the demands
of the people who are led by them, but leaders can never be created by these
demands alone.
A leader's prime responsibility is to provide a solution for the development of
the people who follow him. Take the example of political leaders. The general
population elects these leaders. During the election process, political leaders are
pitted against each other and the one who is able to garner the highest number
of votes wins the election. What do these leaders have to do for winning
elections? They have to consistently prove to the populace that they will cater to
their demands and provide them with facilities that they are in need of. The
leader who is ultimately voted into power is expected to carry out his promises.
A leader who is able to meet the promises made by him during the campaigning
process will probably be elected into power for the second term as well. His
competence in carrying out the promises made by him cannot be developed
overnight. People may have elected a person and placed demands on him, but he
cannot win the confidence of his people and be respected as a leader unless he
demonstrates leadership qualities.
The demands that are placed upon a person are a big challenge for him to
become a leader. In a business enterprise, a manager is expected to lead the
people working under him in such a way that the productivity of the company
increases and the company reaps high profits. Good qualifications and
experience may have assisted in placing the manager in this position. However,
he will need leadership skills in addition to his capabilities in order to meet the
demands placed upon him if he has to succeed as an effective leader. Inability of
the manager to handle the people under him in the most cost-effective manner
may lead to a scenario wherein a more competent manager replaces him.
Therefore, it is absolutely necessary for a person to prove his competence after
being nominated as a leader because merely placing demands is not sufficient for
creating leaders.
One can look up history to find examples of numerous leaders who rose to lead
their people because there were demands placed upon them. Mahatma Gandhi
led the people of India to independence, but could the people of India have
created a Mahatma Gandhi out of any other person by placing demands on him?
It was the qualities, ideology and principles of Mahatma Gandhi which were
revered by the people who followed him. The freedom struggle of India placed
crucial demands on all its leaders, however only a few of them were actually able

67

to lead the people successfully. Therefore, effective leaders can be created by


external demands placed on them along with the support of the inherent
leadership qualities that are required to be present in the person who is
nominated as a leader.




51. The human mind will always be superior to machines because machines
are only tools of human minds.

Admittedly, machines are nearly the synonym of human civilization. General
speaking, among human history, machine is a great invention, which enhance
human productivities, upgrade the efficiency, and then accelerates the
development of human society all the time. Machines are the most loyal friend of
human. They require nothing but electricity or oil. They never take the emotion
when doing their work. They can be as huge as a dam. They can be as fast as a
rocket. They can be as smart as a computer. It is machines make us human
beings the most advanced residents here on the earth.
However, things changed, since people became aware that we are not the owner
of machines, but just a part of machines world. In modern times, machines do
not really serve men but men have to keep an even pace with the machines.
Along the assemble lines, workers take the components and repeat the same
procedure. There is nothing different between the workers and machines, except
the beating hearts. Even the white-collars in the office building, they are just
components of the companies, the big machines built to do the business.
Meanwhile, with the leap of technology, the machines become more and more
clever. The calculating ability of computers has already surpassed human beings
decades ago. And the fast developing of artificial Intelligence seems to show us
that there is no doubt that sooner or later the machines will be cleverer than us.
Going further about the issue, what is quite sad to us is that the more we develop
and evaluate, the more we conscious about the reality that we ourselves are just
a kind of machine. With the development of Gene technology, we can know the
exact sequence of the genes, which code us. The anatomy has already told us we
consists of various organs that work together just like machines. So we cannot
claim that we are superior to the machines, as they are just the extension of
ourselves.


68

52. Every individual in a society has a responsibility to obey just laws and to
disobey and resist unjust laws.

The author of the statement has divided laws into two broad categories: just and
unjust. This division is not correct. Any one individual cannot decide whether
laws are just or unjust. It is subjective to personal interests. Hence, it is wrong to
say that one should obey just laws and disobey unjust laws.
It is difficult to identify just and unjust laws. Many people have tried to define
them. For example, Martin Luther King said that a law is unjust if it is inflicted on
a minority. While as per Thoreau, policies of the State should never be put above
the individual's needs. It is true that one is accountable to obey just laws;
similarly, one has the moral responsibility to speak against unjust laws.
However, it would be an extreme statement that one should disobey unjust laws.
Anyways, laws are based on majority vote and cannot always suit everyone in
different situations. Unjust laws do exist, but there is a proper way to reform
them.
This issue is not a straightforward issue. The fairness of any law depends on
one's personal value system. This is especially true when it comes to personal
freedoms. Consider, for example, the controversial issue of abortion. Individuals
with particular religious beliefs tend to view laws allowing mothers an abortion
choice as unjust, while individuals with other value systems might view such
laws as just. Similarly, some people might find some laws unjust because of their
personal interests. For example, law that limits the emission of harmful
substances from the factories protects people from the harmful hazards of these
things. However, a factory might find this law unjust, as he has to install
expensive filters to dispose of the harmful gases and chemicals from its waste. In
this case, laws protecting the residents` health are more important than the
profit of the local factory.
Laws are made to keep societies in order. We should obey the laws if they are
just, in order to ensure the stability of the society. However, if the law conflicts
with people's moral values or beliefs, people should take direct action against it
and make it amended through legislature. Every country has a certain system to
amend laws. Hence, people should approach the proper channel to reform the
laws, which they think are unjust. However, there are certain events in history,
which involve civil disobedience against certain laws that were not acceptable to
the general public. For example, Martin Luther King fought for American African
rights against color discrimination. He was jailed for his civil disobedience
movement, but today American people are thankful for his efforts. Similarly, in
India, Mahatma Gandhi used civil disobedience against the British Empire for the
rights of Indians and got successful in getting freedom for India. However, these
are the examples where people fought for the nation's interest.

69

Finally, we can say that laws are to maintain peace and harmony in the society.
Justice can be achieved more effectively by helping people to reach a mutual
understanding of justice instead of revolts. Merely ones subjective viewpoint or
personal interests rarely justify radical actions such as resistance or
disobedience.



53. People who are the most deeply committed to an idea or policy are also the
most critical of it.

Good criticism requires thorough and deep knowledge of the subject. Most of the
ideas and policies have a flip side to them, which invites criticism of the policy.
People who are deeply committed to a cause or a policy are the ones who are
aware of this flip side and therefore they not only become the advocates of the
policy but also critics of this flip side. However, I also believe that this is not the
case with all ideas and policies. There have been examples where an idea is
supported fully by some people without being critical of it.
First let us discuss the writer's stand that it is those committed to a cause who is
most critical of it. As I have mentioned earlier, genuine criticism is possible only
with thorough knowledge of the subject. When a person is committed to a cause,
he becomes basic to its functioning and policies. This not only makes him
understand the cause that he supports but also exposes the disadvantages and
harms of the policy as well. For example, a doctor who administers drugs to his
patients to save their lives and make them healthy also knows the ill effects of
their misuse by a healthy person. The doctors become the first persons to
criticize the use of certain drugs by healthy people. Similarly, nuclear explosions
have helped countries to gain power and technology; however, these countries
have also showed their concern towards the misuse of this power. These
countries are strong critics of destructive use of the nuclear power.
However, I maintain that every policy does not have a harmful side to it. For
example, the supports of humanitarian efforts only devote themselves to social
service and humanitarian work. They would not find anything wrong with
helping the poor or needy. As we can see, Gandhi was a supporter of nonviolence
and Mother Teresa stood for the cause of the destitute. They supported their
cause with their heart and soul, without being critical of it.
Even if there is a critical way of looking at an idea or policy, it need not be
criticized by its supporters. Take for example a company building hotels and
malls in otherwise scenic and serene surroundings. An engineer who is involved
in construction of buildings need not be critical of the construction of these
buildings. He only devotes his sincere efforts to his work without thinking about
the result of constructing the multistory buildings and its implications. Another
example of this is that of anti-social elements like terrorists and revolutionaries.

70

These people are also fully committed to their ideas. Their efforts are solely
derived towards attaining their goal. This may involve causing damage to
property, harming people, or even taking lives. This surely brings out that they
are not critical of their activities. It comes as a sharp contrast to the stand taken
by the writer according to which people committed to a cause are most critical of
it.
In the end I would like to sum up by saying that I agree only partially with the
writer. Without refuting his views that the supporters of an idea are the ones
most critical of it, I would like to add that this can't be accepted as a general
truth. While the writer can be true in many cases, there are exceptions where
those who are committed to a cause only devote themselves to it completely,
without being critical of it.



54. Some people believe it is often necessary, even desirable, for political
leaders to withhold information from the public. Others believe that the
public has a right to be fully informed.

Freedom of information to the population is a right that is enshrined in
international instruments such as the Universal Declaration of Human Rights and
is believed to be the cornerstone of every democracy. Freedom of citizens to seek
and access information has wide merits and except for cases where withholding
information from the public is in the larger interest of the nation, any
curtailment of this freedom would result in a demagogic state of affairs.
At the outset, exceptions to this fundamental right must be enlisted.
International charter of human rights defines these unequivocally as instances
where there is a threat to national security, law and order, rights and reputations
of others and public health and morals. In the event of war, classified
information, which would endanger national security if access were available to
egregious elements, must be curtailed from reaching the laity. This is
necessitated, as it is impossible to delineate anti national elements from amongst
the populace. Secrecy in matters is also prudent when unsubstantiated
information is likely to cause panic and pell-mell in the nation.
Barring the above instances, free flow of information is essential for the citizens
to participate actively in the governance of the society. It is impossible for the
government or legislation to guarantee the security of each and every individual
and in instances of possible terrorist attacks, the denizens cannot be vigilant
unless they are disclosed information about possible terror attacks that the
intelligence agencies have intercepted. A complete disclosure of facts,
endangered locations and modus operand of the terrorists is crucial if such
events are to be thwarted.

71

Effective governance is a misnomer in absence of accountability of public


officials who work on behalf of the citizens. Corruption and red-tapeism is
rampant in countries where the populace does not have the right to demand
information about how the money they pay as taxes is being utilized. Statistics
prove that countries where the right to information act has been enacted have
been successful in reducing corruption and mismanagement. Statistics also
reveal that 9 out of 10 countries that rated high on government transparency
had an effective legislation whereby people had the freedom to peruse
government documents. In India, the right to information act has been
instrumental in implicating many high-ranking public officials who had
plundered the taxpayers money.
Transparency in government working is also essential for citizens to make a
more informed decision about their elected representatives. In absence of such
information, the decision would be based purely on rhetoric and in certain cases
vote buying. In less developed countries politicians often rig up data and false
facts during election time to convince the electorates about the impact they have
made. An uninformed citizenry would have no way of adjudging the veracity of
these claims and would be hoodwinked into voting for those who have the
political and mercenary muscle to influence opinion. Even the government
officials are impelled to work for the good of the people under the watchful eye
of an informed citizenry.
In closing, when the larger interest of the nation is taken into consideration there
is no justification of information being suppressed from the public. Free access to
government policies and decisions is imperative for effective governance,
enhanced transparency and informed decision making which are pillars that
strengthen a nation. With the preclusion of matters where disclosure of
information is likely to impede the functioning of the nation, there is no cogent
argument for such censorship.










72

55. Colleges and universities should require all faculty to spend time working
outside the academic world in professions relevant to the courses they
teach.
Education prepares us for life by enlightening us with knowledge. Though
knowledge is present all around us, it is an educator who imparts knowledge to
us. Educational institutes serve as platforms for the interaction between the
learners and learned and their faculty enables one to independently face the
world. In other words, the quality of education at university and college levels
plays a vital role in giving economic independence to an individual.
The faculty of an educational institute primarily consists of lecturers, professors,
lab attendants and activity in-charges. In spite of plenty of books and internet
information available, an educator plays a significant role in giving instructions
such that a complex topic is comprehended easily by his students.
To train an individual such that he does well in a chosen profession, it is not
necessary that the faculties should practice that particular profession. The
strength of an educator lies in his communication skills and ability to transform
topics from complex to easy. For instance, a language teacher should be able to
develop an understanding of his subject. He should be capable of explaining
difficult comprehensions and poems with ease and develop the skill of critical
analysis in the students along with a liking for his subject. For all this he need not
be an author or poet himself. Likewise, a mathematics professor need not be a
mathematician who develops different techniques to approach a problem,
invents new formulae and theories, unless compelled by his own choice. He is to
explain different topics in an easy way, and develop problem-solving skills and
prepare a sound base for the future mathematicians. Even in science subjects, the
faculty need not transform into scientists. Good explanation of scientific terms
and theories along with the relevant practical work in the laboratories would
suffice for the ones preparing to go into research work.
In addition to this, the trainers and passed out trainees pursuing the same
profession may create awkwardness, as working on the same platform may
create ego hassles and diminish the much deserved respect for the faculty amidst
their ex-students. Further, frequent shifting from educational institutions to the
outside workplace may disturb the curriculum and concentration of the students
as well as the teachers.
However, there are a few areas where the quality of education can improve
drastically if the faculty members have some experience of working in outside
world in professions relevant to the courses they teach. For example, the
research guides at the university level can give more useful input to their
students aspiring for research work in scientific laboratories. Similarly, all
faculties of professional institutions like business management, teachers
training, training for doctors, etc. can enhance the quality of teaching by
supplementing it with their first hand experience in the respective professions.
Apart from theoretical knowledge, it creates awareness about the actual
environment and challenges to be faced in future.

73

Thus, the role of faculties in colleges and universities is to prepare, guide and
motivate the students in recognizing their aptitude so that they choose an
appropriate profession. Barring few courses, the faculties can conveniently do
justice to their teaching without working outside in professions related to their
subjects.



56. Knowing about the past cannot help people to make important decisions
today.

History is a collection of past events. It includes the trends and patterns of
people of earlier civilizations, their faith and achievements, along with the details
of the lives of great men who created a niche for themselves in social, political or
religious fields. All in all, history is the study that teaches us not only about the
past events, but also about the cause and impact of actions taken in the past such
that they serve as a source of guidance for the present and the future as well.
The achievements and developments of the past assist us in the present. For
instance, the knowledge of early man in creation of weapons and various
equipments has been the foundation for the tools developed today. The
knowledge of making houses, planning of towns, architectural skills, helps in
further enhancing these traits without wasting much time and in developing a
completely new idea.
History reveals about the achievements in the areas of science, medicine,
literature, mathematics, etc. To quote an example, implementations of Ayurveda,
homeopathy and yoga in today's life are inspired by the developments and
successes in the past. Similarly, the knowledge of mathematical calculations,
astronomy, astrology, literature and famous legends are meaningfully extracted
today because of the mediating role played by history.
Further, the leaders and other eminent personalities of the past act as a source of
inspiration to the present generation. The biographies of Nehru, Ghandhi,
Lincon, Napolean and Hitler are not merely amusements today but they
definitely make one aware of certain actions and their consequences, which are
relevant even today. To cite an example, the use of atom bombs in Japan and its
aftermaths curtailed different countries from using the nuclear weapons till date.
The horrors of the world wars reflected in history warn nations against another
such grave mistake. It is history that teaches leaders to restrain from
dictatorship and empowers common men to retaliate it. The revolts and
revolutions held in the past have set examples for the future.
The study of ancient civilizations and their origin, their belief, their actions and
culture gives a deeper understanding of different religions and social norms. This
in turn enables us to accept people of different backgrounds by inculcating

74

religious tolerance, which is the need of the hour in most parts of the world
today.
In addition to this, there have been many gradual climatic changes since time
immemorial. Historical records of such changing weather patterns make us more
aware and well equipped in dealing with the climatic and ecological changes. For
instance, the recent floods in Bihar (India) caused due to the Kosi river could
have been better dealt with (if not averted), if the historical records of this
notorious river and its highly unpredictable behavior of changing routes was
given sufficient thought to.
Conclusively, history is not merely a study of past for information sake, but it
gives us a foundation to progress in the fields of science, literature, law and
order, politics and social customs. It empowers us with the understanding of
various happenings, patterns and likely consequences of different actions in the
present and in the future as well.



57. We can usually learn much more from people whose views we share than
from people whose views contradict our own.

Any kind of learning needs a favorable environment. Where there is conflict
amongst people it only results in commotion. I believe that although there is a lot
that we can learn from every situation and people around us, it is a general
attitude of people to dismiss the ideas of someone who has a contrary view.
According to the given statement, we can learn a lot more from people with
whom we share our views rather than from those who hold contradicting views.
I, however, feel that there is a great scope of learning from people who hold
different views as well. Nonetheless, we are not ready for this type of learning
because our ego comes in the way. Disagreement causes stress, tension and
commotion, but we should try to rise above this and benefit from the
experiences of others who we disagree with.
Birds of a feather flock together. We join the company of people who share the
same views as ours. Thus, there are increased chances that we listen to what
others have to say instead of getting involved in an argument. This makes a
perfect environment for positive learning and the ideas of others influence us.
There is no inhibition among such people and a mutual rapport is established.
However, the scope of this type of learning is limited. It ends where the views are
exchanged. When one person is at the receiving end and agrees to all the views of
the learned person, he does not question his knowledge. On the other hand, there
is a greater scope of learning from someone who has different ideas than you.

75

When people holding different views come together, they argue over a certain
issue. We cannot take it for granted that we are always right. What seems to be
correct to us may be wrong for someone else. Therefore, in an argument we
come across different views and interpretations of the same issue. There could
be sides of looking at the issue, which do not strike to one person, but these can
come out only in an argument where people present views that are different
from those of yours.
Another example of a great social worker is that of Mother Teresa. She is known
for her concern towards the poor, sick and underprivileged. She has given a
definition to humanity, charity and compassion. Even though there are millions
who claim to appreciate her, there aren't many Mother Teresas around. So much
so that we even ignore our basic social responsibility of charity. 'Charity begins
at home,' is a dictum for everyone but ourselves; we do not even bother about
the financial well being of our domestic helps. We are so self-centered and
engrossed in our own lives that we seldom practice what we know should be
correct. We celebrate the birth and death anniversaries of great men, celebrate
the official holidays on such significant days, attend chat shows, hold
discussions...we do it all but only for the moment. This shows that the influence
of great men on our minds and actions is merely superficial. We 'claim' to admire
them but do not prove the same in our actions.
Let us take an example of a simple classroom. If the students learn everything
that the teacher teaches them without asking their doubts or questions, they
become passive learners. On the other hand, if the students try to figure out the
problem on their own, they will come up with doubts and questions. They might
not agree with their teacher on a certain topic for which they can initiate a
discussion. It is only in such a discussion that a real clarification of the topic
comes out. This type of learning is detailed, effective and lasting.
A new discovery or invention comes to light only when someone disagrees with
the norms and tries to prove his point. It is only when someone refused to
believe that earth is stationary and that Sun revolves around the earth that
efforts were made to discover the truth.
Disagreement surely causes stress if the criticism is taken in a negative way.
Instead we should welcome criticism positively and try to learn from the ideas of
others. It is a matter of attitude that can change the way people think. If you are
ready to welcome contradicting views and accept different ideas, it will increase
your knowledge manifold.





76

58. The most effective way to understand contemporary culture is to analyze


the trends of its youth.

Our culture is our customs, beliefs and attitudes. It is the way of life that the
people of a particular country, class or society follow at a particular time. The
culture of a society or country keeps changing and new trends and beliefs keep
replacing the older ones. It is the young who exhibit their culture most typically.
However, it is also true that the older generation also has its ways of following its
culture and beliefs and the old should not be ignored. I believe, while the old are
moderate in following their culture, which might be different from that of the
youth, they are equally a part of the society. Hence, a proper understanding of
the contemporary culture of any place can be done only by a comprehensive
study of the trends of the youth as well as the way of life of the old.
Since culture keeps on changing, what was seen as a prominent trend in the
olden times may not be so today. This means that culture does not remain fixed.
However, it is possible that in a society at a particular time more than one
culture co-exist. When it comes to understanding the contemporary culture of a
society, it is the young who are the most expressive. By nature itself, the young
are flamboyant. Therefore, the trends and beliefs they follow are noticed the
most. They are the ones to accept the latest changes first of all and make it their
trend. What they wear, think, do, and eat reflects the attitude and trends
followed in the contemporary society. A superficial study would indicate that the
popular culture followed by the youth is the culture of that society.
On the other hand, a proper understanding of the real contemporary culture is
done only when it is studied in totality. That means the whole population should
be considered and the trends followed by all the members of the society are
studied. It may be true that the youth are most expressive and the contemporary
culture is generally seen in their behaviors and beliefs, it is also true that the old
form a large part of the society. They have their own culture, customs and beliefs.
Most of these people prefer to follow their culture without expressing it. They
are not flashy about their ways of lives. This is the reason that the culture they
follow is not observed as the culture of the society. However, ignoring this
section of the society and considering the culture of the youth, as the
contemporary culture would be shutting our eyes to half of the truth. A proper
understanding of the society is done only when the cultures, customs, beliefs and
attitudes of all the sections of a society are taken together.
As mentioned earlier, at one point of time there is more than one culture that co-
exists. It is easy to read and observe the culture followed by the young as
compared to that followed by the old. Although it is the beliefs and trends of the
youth that come out when a society is observed for its culture, it only reflects a
part of its composite culture. A society is made up of different types of people
and thus different cultures are intermingled in it. Therefore, a study of the
culture of a society will be complete and justified only when all the people and
their trends are studied.

77

59. People's attitudes are determined more by their immediate situation or


surroundings than by society as a whole.
It is true that immediate situations do affect people's attitudes; however, you
cannot ignore the effect of internal characteristics. A person can have different
attitudes in different situations. People live in societies and their actions are
influenced by the happenings in their immediate societies. Sometimes people do
not make decisions only by their immediate surroundings. Their intrinsic
characteristic also is a very important determinant. For example, if a person has
a strong sense of sympathy for homeless people, he may propose to set up a
home for these destitute people. Many people make charitable hospitals without
any selfish thinking. Hence, people's altitudes are also decided by people's
internal characteristic like the way they think about the world, dispositions, and
their tempers. We cannot give more importance to any one. Both the things play
a role in determining people's attitudes.
Immediate surroundings are also very important in determining people's
altitudes. When thinking of an issue, whether a social problem or a phenomena
or a personal issue, a person's attitude is greatly affected by his positions in the
society. People usually do no take decisions without thinking about their
positions and status in the society. For example, when we choose our occupation,
we probably will do a research on the different aspects like salary level and
working environment among the companies. These are the outer factors that
affect our decision. What we consider most is whether we are able to do the job
asked by the respective company and we are very likely to choose the job of the
company accordingly to our ability to do that job that is the inner ability.
Internal characteristic and surroundings together affect our attitudes. When a
child comes in this world, he inherits some genetic characteristics from his
parents. These are nothing but inner characteristic of that person. Through the
child's activities, we can predict that he is like his father or mother since his
parents also possess habits similar to the child. However, the environment
where the child grows up also affects his attitude towards life. If a child grows up
in surroundings of criminals then he might also develop attitude of a criminal
since after doing a crime he will get appreciation from his parents rather than
reprimand. Such environment will promote him to become a criminal or a
gangster. The behavior of a person is definitely determined by good or bad
surroundings.
For example, Mother Teresa is considered an angel. Mother Teresa's family was a
devoted catholic family. It was her family's generosity, care for the poor and the
less fortunate that made a great impact on young Mother Teresa's life. She made
up her mind to help poor and needy people when she was just a child of 12 years.
She dedicated every day of her adult life caring for the dying, the cripple, the
mentally ill, and the unwanted people.
As we grow, our good and bad experiences in life also affect our behavior and
attitudes towards others. However, our internal characteristics remain the same.
A good person cannot behave badly with anybody after a certain level. The
internal characteristics contribute to develop the attitude of a person.
Additionally, in the long run, as the situations he faces and the surroundings he

78

interacts with change, his attitude becomes an outcome of that environment. In


sum, people's attitudes are determined by internal characteristics and external
environment almost equally well.



60. Claim: The best test of an argument is its ability to convince someone with
an opposing viewpoint.
Reason: Only by being forced to defend an idea against the doubts and
contrasting views of others does one really discover the value of that idea.

The author opines that the merit of an argument resides in its ability to change
the opinions even of those with dissenting views. I however believe, that the
strength of an argument is ascertained not only in convincing others to agree but
also in an ability to listen to and reconcile opposing perspectives. Socrates
emphasized the importance of seeking the truth of the matter as opposed to the
ability to win an argument. This view characterizes an openness of thought and
the willingness to acknowledge that ones initial ideas could be expanded and
developed or in fact be completely wrong. This process of seeking the truth is
fundamentally based on a dialectical process seeking reconciliation amongst
different points of view and has in its ethos the larger goal of enriching the
outcome.
Admittedly, whether the merit of an argument can be accepted as an idea of
great value and importance depends on whether its points of view are
convincing and persuasive enough. Component parts of arguments, like their
inner logic and supporting evidences, must be factually and logically reliable in
order to deal with doubts and skeptic challenges from various perspectives. In
the realm of science, for instance, theory innovations though having its genesis in
disagreeing with the existing principles would not be made solid and sound, in
order to be accepted as newly prevailing standards, by a few talents proposing
ostensibly and allegedly imaginative and attractive perspectives, but instead the
reliable outcomes of careful and thorough calculations, or at least logically
reasonable extrapolations, are necessitated for the arguments to have the ability
to convince challengers and defend the core values of the innovations. A
government policy, to be popular with and accepted by the denizens, also need
to be capable of persuading at least the majority of representatives of the citizens
to authorize their approval. Otherwise, any argument or idea that the
government possesses could be of little use than some wasteful nonsense
proposed by hypocritical demagogues.
However, compared with an arguments convincing power, its ability to
accommodate and reconcile opposing perspectives is the most powerful part
lying in the core competitiveness of this argument. Although it has been
mentioned above that for science reliable and solid evidence and logical

79

reasoning are of great scientific importance, the truly great and game changing
innovations, such as the principle of relativity created by Albert Einstein, are
indeed some elegant combinations of the knowledge acquired by predecessors
and the newer perspectives of more recent ways of thinking. In the process of
creating this theory, Einstein himself was engaged in years of discussion through
many exchanges of letters and ideas with others who held different or even
totally opposing points of view. It was his belief that a better, more unified
theory, which can reconcile all the existing theories of physics, helped him in this
epic discovery. Preclusion of one view over another would not have created a
synergy with the total exceeding the sum of its parts.
Any single view is also likely to reflect the sentiments only of the puissant and
tends to be limited and even distorted. If socialists and leftists do not question
the government; vested interests of the capitalists and the mercenary gain the
government can derive from them would become the sole considerations for
promulgating policies, which would be detrimental to the interest of the
proletariat. Examples of such scenarios abound in real life and a latest incident in
India where a raging media debate between the government, industrialists and
the laity concluded in the industrial house relocating to another location in order
to safeguard the interests of the farmers, is testimony to the fact that the ability
of an argument to reconcile dissenting ideas better serves the majority. In
situations like these, trying to convince all stakeholders towards a common
viewpoint could result in an impasse which can be avoided through a discussion
where each party tries to see the perspective of the other and willingly arrive at
a common decision.
A rich and lively debate amongst people of dissenting opinions also stimulates
the progress of all concerned stakeholders. Through complete discussion, the
participants develop critical thinking skills and empathy that enables them to
view the situation from different points of view of the others. By seeking to
transcend differences, they can develop their own beliefs more thoroughly and
by critically evaluating various views of others they can nurture the cognitive
skills to identify fallacies and weaknesses in the ideas of both theirs and their
opponents. Additionally, listening to counter arguments is the fastest way to
promote learning and indeed many educational institutions worldwide use
debates as means to pique the interests of students to explore, research and get
into the depth of the topic. All these skills are of life importance and enable
debate participants to make huge contribution in great width and depth.
In conclusion, it has been highlighted that the merits and values of an argument
lie not only in its ability to convince others with solid evidence and logical
reasoning, but also in whether it harbor enough room for reconciliation of
dissenting and opposing perspectives, and thorough discussion with depth and
width is the most effective way to ameliorate and improve an argument.


80

61. All parents should be required to volunteer time to their children's schools.

Learning begins at home. From the time a child is born, he learns from his
environment including his parents and the community he lives in. There is no
denying the fact that a child's behavior and knowledge are largely dependent on
how his parents groom him at home. A child picks up the language that is spoken
by his parents and learns various other facts of life as he constantly questions the
people around him with whatever doubts he may have. How can then one
assume that the moment a child is sent to school, parents and communities
should detach themselves from his process of learning? It is essential that
parents, communities and teachers make a combined effort to impart maximum
possible knowledge to schoolchildren.
Education systems around the world may or may not emphasize on assignments
to be carried out at home depending on the education system being followed in
the countries. If a school child gets assignments to be done at home, the parents
might help him in completing the work. However, if the education system that is
being followed does not emphasize on home assignments, should the parents not
be bothered about what the child has done in school? Can the responsibility of
imparting education be left to the teachers alone? It is not necessary that the
child spend his entire day in school reading and writing. He may be spending a
lot of time in other co-curricular activities like sports and music. When is he
supposed to review whatever has been done in the class? Involvement of parents
in the activities being carried out in the school will benefit both the parents as
well as the teachers in imparting high standards of education.
There may be instances when a child is an introvert and he does not raise
questions or doubts in the class. His questions will always remain unanswered if
his parents do not take the initiative of spending time with their child to clarify
his queries. On the other hand, there may be a case wherein the child is
inattentive in class and is easily distracted by the happenings around him. It is
not necessary that a teacher will always be able to give undivided attention to
such a child. One cannot ignore the fact that in a classroom, a teacher has to give
equal importance to all the students in the class. The traits of a child are best
known to his parents and the people who live with him. Can the child's family
choose to ignore his performance in school and blame the teachers for failing to
teach him properly? The parents will have to take extra pains to ensure that their
child is groomed and taught to be attentive in class. For this they may have to
interact with the teachers and choose the most appropriate method for
educating the child. At the same time, it is not necessary that all schools provide
the best mode of instructions and good quality teaching standards. The inputs
from the parents of the school children will help the schools in modifying their
teaching methods to suit the children studying at their schools.
Education is too important an aspect of life to be handled carelessly. It will not be
possible to reverse time and go in the past to undo the errors made by neglecting
the education of a child when he was in school. We all have heard of the saying,
'A stitch in time saves nine.' Leaving the education of a school going child at the
discretion of his teachers may have serious repercussions in the future. If the

81

parents of the child and the people around him have not been showing an
interest in what he does at school, the child might be led astray and it might have
an adverse effect on his future career prospects. Therefore, it is necessary that all
local schools involve the opinions and contributions of parents in the process of
imparting high quality education, as the teachers alone cannot provide it.



62. Colleges and universities should require their students to spend at least
one semester studying in a foreign country.

The author is of the view that all college and university students would benefit
from spending at least one semester studying in a foreign country. While many of
us may agree with this view, there are various aspects that need to analyze
thoroughly before coming to a solid conclusion.
Admittedly, spending a few months in a foreign university would provide
students an opportunity to work in a culturally and intellectually different
setting from the one in their home country. Staying away from home would
prepare students to survive independently and also learn to adjust in this rapidly
shifting world. For example, students would require to take crucial decisions as
well as do your personal work yourself, which is otherwise neglected and left on
to the parents when one stays at home. Furthermore, in a foreign university
students could avail better research facilities and opportunities that would not
have been possible in their home institution. For example, in Germany, a student
could be provided with world-class infrastructure and work with the best
researchers in automobile industry, which is not possible in India. One must
admit as well that witnessing the rich culture, diversity and exchanging ideas
from people with different civilizations and backgrounds imbues a broad outlook
and tolerance. Appreciation of foreign language and life-style would be pivotal in
understanding that there is a common thread that binds all mankind.
One the other hand, it is also true that making it necessary for students to study
in a foreign country could have its own negative effects. Experience would tell us
that all students do not find it suitable to settle down in a foreign country
initially, especially the ones coming from a comparably conservative
background. The different food, language, culture and an all together different
environment could make it difficult for a student to focus on his studies and
research. It could preclude the primary objective of introducing this program
and instead, inhibit interaction and thus learning. The local problems of the
foreign country and working environment could also deter students and have a
negative effect on their morale.
Although I believe that university students would benefit from spending some
time studying in a foreign country, the policy should be implemented effectively
and the students should be given a choice whether they wish to avail this

82

opportunity. In my opinion, a flexible scheme if brought into action would not


only foster stronger relations between countries but also work in the favor of
research and thus benefit mankind in the long run.



63. Claim: Any piece of information referred to as a fact should be mistrusted,
since it may well be proven false in the future.
Reason: Much of the information that people assume is factual actually
turns out to be inaccurate.

In all fields of study the outcome is termed as facts. We all rely on facts, which
are presented before us with relevant explanations. If a piece of information is
presented before us and there are enough proofs to make it viable then we are
bound to accept it as a fact. I believe that if a study is a proved fact then there is
certainly no doubt about its authenticity. It is quite likely that a fact, which is
proved correct today, might yield some different results after a few years but
that does not make the present facts inaccurate.
The term fact is generally associated with scientific studies and discoveries.
Science is a very vast world and there is so much to explore and scientists keep
making new assumptions every now and then. The assumptions are then studied
and confirmed into facts. The outcome of one scientific study becomes the base
of another scientific study. It was a known fact that cow's milk was better than
buffalo's milk for infant consumption. A more recent study proved that infants
who were on cow's milk only, acquired iron deficiency. The deficiency was due to
the fact that cow's milk disabled iron absorption by the body. Although the older
study could not bring this point into light but that study was also correct. The
new fact certainly does not make the old study false but it aids the old study and
thus it came into notice that it is important to feed an infant other fluids along
with cow's milk. Therefore, whatever studies are done the facts which are
proved have certain amount of authenticity in them and they are not absolutely
false.
Although most of the factual information is accurate but the facts are results of
studies made by humans so they are bound to be incorrect sometimes. This
generally happens in the field of medicines. Firstly, scientists would recommend
a medicine for an ailment and in the next study it would be proved that the
medicine has ill effects on human body.
Therefore, it is important for people to make their research with utmost care as
if researches go wrong it can result in drastic consequences. We all are
dependent on our scientists for new studies and discoveries and if they adopt a
negligent approach then eventually we would suffer. Human life is precious and
any study concerning it should be deliberate so that chances of getting facts

83

wrong results are minimized. Although we are sufferers of such failed studies but
at the same time it can not be forgotten that these scientists are humans after all
and they are only doing their work. New and better technology aids their studies
and thus they come with new and better results. It is not necessary that facts are
always inaccurate but their viability can certainly be questioned.



64. Claim: Many problems of modern society cannot be solved by laws and the
legal system.
Reason: Laws cannot change what is in people's hearts or minds

The author claims that legislation cannot affect a change in the opinions and
attitude of people and laws are not the panacea to all ills that plague our society
today. While I concur with the author when he opines that laws cannot be the
solution for all problems, I disagree with the statement that laws cannot effect a
change in peoples attitudes and beliefs and propose that given sufficient time
laws have the ability to change the ethos of a society.
Laws are only structural guidelines that delineate behavior and guarantee
people their freedom while ensuring that the rights and freedoms of others are
not encroached upon. In absence of legislation, we would be ruled by a state of
anarchy and chaos with each individual plundering and pulverizing at will and
the law of the jungle where the mighty thrive and the weaker perish, would be
true for the human race as well. However while laws provide a framework
within which to operate, by themselves they do not guarantee an equitable and
just society. Laws need to be supported by a substantial deterrence for people to
take them seriously. They also require ardent supporters who would act like
harbingers of change and propel others to adopt them as well. However all these
still believe that human behavior can be predicted and thus controlled. In truth,
human behavior is as much rational as irrational, as thought-out as impulsive
and as cognitive as emotional. The fact, that human behavior cannot be restricted
is evidenced from Middle Eastern countries where severe laws and corporeal
punishments are also incapable of casting human behavior into a mold.
Laws, over a sufficient period of time can however engineer a social change. The
initial indifference metamorphoses over a length of time into acceptable
behavior and deviations are viewed as social stigmas which replace enforcement
as the deterrence to aberrant behavior. From abolition of slavery in the US, to
stigmatization of the sati practice in India, laws have seeped into the social
conscience of nations with a majority of denizens accepting them as part of their
moral conduct.

84

Laws have managed to reform not only how people behave but also how they
think. Laws governing science have transformed how people view their world an
from an almost exclusively religious outlook of the functions of nature, an
objective and scientific rationale of phenomena is considered. The universal law
of gravitation for instance has given an objective grounding and changed the way
we cogitate.
In summary, laws have both the power to transform behavior and thinking.
Laws, though, do not work in silos and strong mechanisms. Both promoting
desired behavior and curtailing undesired behavior need to be implemented.
However a complete transformation of human behavior is not possible even
under the best of legislation and enforcement as the vagaries of human mind
cannot be ascertained completely.



65. The primary goal of technological advancement should be to increase
people's efficiency so that they have more leisure time.

Starting from the Stone Age, man has been exploring different ways and means
to make his life comfortable. There have been numerous technological
discoveries and developments, which have aided in making the world what it is
today. The modern man has access to various technological gadgets which have
made his life much more comfortable as compared to the life led by his
forefathers. Moreover, man has become more efficient as he now has the
assistance of technology to carry out his routine work. There is no doubt that the
primary goal of technological advancement should be to increase people's
efficiency, but creating more leisure time cannot be related to either the
technological developments or the increased efficiency of man.
The advent of technology has definitely made life better by improving various
aspects like communicating and traveling across the globe. People can now
access information on any subject from the Internet at the click of a button
rather than having to search through books and libraries. Has the time that has
been saved, utilized by the Internet surfer for leisure? On the contrary, he would
be busy searching for other information on the Internet which perhaps is not all
that important for him. Moreover, he may start looking for some opportunity on
the net to work part-time and earn some money in the additional time that he
now has. This indicates that the technological development in the form of
internet services has made people more efficient, but this increased efficiency of
people has not led to the creation of leisure time for them.
People can fly to any part of the modern world in a matter of hours. Compare this
with the people in the past who took several weeks or months to sail from one
country to the other. Have these technological advancements created more
leisure time for people in the present world? On the contrary, people are much

85

busier than their ancestors were in the past. People can fly to their destinations
easily and this has led to an increase in the number of conferences and corporate
meetings that are held in some other part of the country and employees are
expected to do extensive traveling to meet the company's requirements. People
spend hours traveling to their offices due to the traffic congestion on roads
because of the numerous vehicles, which are a result of technological
advancements. Therefore, it can be seen that technological development has
made man efficient, but its goal could never be to provide leisure time as that
would lead to the dilution of the standards of the technology being developed.
If technology had been developed with the aim of giving leisure time to people,
then we could not have reached where we are today. People would have leisure
time if they could not travel to distant places easily and so the companies would
be restricted to carrying out their conferences locally, thus giving their
employees leisure time which they would have otherwise spent in traveling. If
there were no Internet, people would not have been hooked on to the internet till
late in the night. They would have utilized the time thus spent for relaxing or
going out with their family. In other words, people would have got leisure time
only if these technological advancements were either non-existent or they were
of a lesser magnitude. If we wish to see man more efficient in the future as
compared to what he is today, then the technological developments will have to
be planned keeping in mind the efficiency of man with total disregard to his
leisure time.



66. Educators should base their assessment of students' learning not on
students' grasp of facts but on the ability to explain the ideas, trends, and
concepts that those facts illustrate.

The proposal of the author that learning or knowledge of concepts should form
the basis of assessment instead of the ability to memorize and reproduce facts is
one that I am in complete accord with. In the subsequent paragraphs I would try
to bolster this conclusion by providing supporting arguments with an objective
of unequivocally proving the merit of the position.
It would be useful to assess the argument by evaluating why an unjustifiably high
dependence on rote learning has come to be. The reasons can be traced back to
the facts that: such competencies are the easiest to measure and grade, teachers
are not competent or have little or no incentive to promote understanding of a
particular topic and lastly, there is a demand for such abilities by higher schools
of learning and employers.
The foremost reason why factual knowledge and by extension grades, have come
to occupy a high pedestal in pedantic instruction is because these are easy to
evaluate. Knowledge of a fact can be either black or white, there is no grey area

86

confounding assessors. With the advent of standardized assessment tools that


help ascertain learning levels on a common platform, it is also interesting to note
that these lower levels of information are mostly easily rendered into items.
According to Blooms taxonomy, learning levels range from remembering to
creating. It is then unfortunate to note that convenience has become a primary
reason is restricting the perimeter of our vision to the bottom two learning
levels. Creativity is not encouraged, instead is severely browbeaten overlooking
the critical ability of the student to apply his instruction in being able to think
afresh to solve problems that would confront him.
The second alarming reality is that a large number of educators today are ill
equipped to explain concepts and ideas, linking them to real life scenarios that
would foster learning. This is a large concern facing policy makers all over the
world, and especially in developing countries, where teaching is viewed as a fall
back career option for those who fail to secure employment in any other field.
Such teachers are not only unqualified, but also have no motivation to aspire for
or transmit a love for learning to their pupils. Students graduating from such
institutions would flounder in the global arena. They would be unable to
compete with their counterparts in other countries and would lack any real
analytical and logical abilities, impeding the progress of themselves and their
countries.
Another reason often cited in defense of the current educational system hemmed
by rote learning, is that grades provide an objective platform enabling anyone
interested to quickly interpret the competence of a particular candidate against
hundreds of other competing ones, whereas subjective parameters are less
qualified to provide such comparisons. Graduate schools and employers both
place a high importance on a pedigree of intellect. A prospective candidate or
employee is considered to be a safe investment if she has displayed excellence all
throughout her academic career. Proponents of the fact based or grade based (as
can be interchangeably used) system would argue that factual knowledge and
learning are not mutually exclusive sets. In fact, research has explicitly
highlighted that this is indeed the case. Because the current assessment system
primarily tests for factual knowledge a child is forced to choose where to apply
his cognitive functions. As a result deciding on the candidature of an individual
based on his scores is somewhat counterproductive. Real life examples of school
and college dropouts like Mark Zuckerberg, Steve Jobs, Bill Gates would hold a
testimony to this argument.
In conclusion, it has been highlighted that all justifications provided in defense of
a factual approach to education are baseless and severely impede excellence. The
effects of this are felt not by the individual alone, but also by educational
institutes, employers and the nation as a whole. It is therefore necessary to
understand the real purpose of education and reorient us to make understanding
and not memorization the crux of education.


87

67. Unfortunately, in contemporary society, creating an appealing image has


become more important than the reality or truth behind that image.

Beauty lies in the eyes of the beholder. This saying holds true in the modern
world where you can consider yourself to have an appealing personality only if it
appears the same to the people who are looking at you. Everyone likes to be in
the company of people who are entertaining, jovial, witty, successful, wealthy etc.
Therefore, it is obvious that creating an appealing image has become very
important in contemporary society. The reality behind the image is not of any
consequence as people can now hide their actual selves from the eyes of the
public by effectively utilizing their right to privacy.
The importance of your image in the eyes of others assumes alarming
proportions if you are a person whose career depends on his image. Politicians
are an example of people who fall in this category. It is imperative that such
people create an appealing image for themselves or else they may not get the
kind of support that they require from the society. They have to show to the
world that they are honest, trustworthy and responsible. The actual truth may be
far from what is being projected, but it is necessary to present an appealing
image or else the politicians are unlikely to be voted into power. This is the
reason why whenever elections are around the corner; there is a huge drive to
dig out the past of the leaders of the opposition parties so that their images can
be tarnished in some way or the other. All this happens because it is a well-
known fact that having an annoying or disagreeable image will adversely affect
the vote bank of the politicians.
It is true that the actual reality behind the images is more important, but the
bitter truth is that the society no longer wishes to see the reality. Who would be
interested in dining with a host who is formal and has a disagreeable
disposition? What if such a host is the owner of a business establishment? He will
not be able to get the best out of his employees if they do not like him at all. They
will probably switch companies the moment they get a chance. This is because
no matter what the boss is like in reality, all employees would like to work for a
boss who appears to be pleasing and this can be done only if the boss presents an
image of himself that is appealing in the eyes of the others. Therefore, the owner
of the business will have to forcibly present himself as an agreeable person who
is open to discussions with his employees and is willing to make adjustments for
the comfort of the people who work under him. He may be an entirely different
person in reality, but it will be more important for him to create an appealing
image for himself if he wants his business to be successful.
The present society is easily swayed by the pleasing attitude of others.
Everybody wants to be with those people who are good to get along with and are
pleasing as well. Therefore, it is evident that it is far more important to create an
appealing image rather than concentrating on the reality behind that image,
especially if you are keen on succeeding in life.

88

68. Although innovations such as video, computers, and the Internet seem to
offer schools improved methods for instructing students, these
technologies all too often distract from real learning.

Innovations are the greatest strength of mankind as they make us think more
and thus create. Innovations of any kind have always been helpful to mankind.
The huge amount of credit for our development goes to innovations that the
humans have made in all stages. Innovations of all kinds are proofs that humans
have been progressing. The metamorphosis of a human from a prehistoric cave
man to today's modern man is all because of innovations. Innovations in all fields
are helpful and useful for us and education is one such field where innovations
like video, computers and the Internet have proved to be of remarkable
importance. I believe that these all means of education in schools have
revolutionized the whole education system. I also recommend that all the
schools, which are not able to make use of these innovations in their education
techniques, must use them for their numerous benefits.
Children are the future of any country and education plays an indispensable role
in making them responsible citizens of a country. The teachers in all education
institutes try their level best to educate children. The students in school can be
taught a great deal with the use of videos. If the service of an experienced teacher
is unavailable in a school then recorded lectures can be helpful to the students.
In the same way with the use of this innovation children can be shown the facts,
which were earlier, told to them verbally. This way the students will retain more
knowledge.
Similarly with the use of slide shows through computers the students can be told
numerous facts in a lecture which otherwise would require a couple of lectures
of the teachers. This way the teacher would be able to impart more knowledge to
the students. Computers are also helpful to students as they can learn and write
more promptly and accurately in lesser time.
The use of Internet has large benefits for the education system. The large
number of schools across the world can share their education through the use of
Internet. Vast knowledge on all the subjects is available on the net, which can be
beneficial for the students. Distant education has become accessible through
Internet. Learning to use Internet helps a student in long run when the student
goes to a workplace.
There are numerous benefits which innovations provide us with. The help in
school education is one of the benefits derived from it. I believe that an
innovation empowers people to do what they desire to do. It helps people in
becoming more efficient, creative and productive. It also helps students to learn
things, which they could not learn before.
Although these innovations have been very beneficial for our education system
but the medium of video, computer and Internet lack the personal touch, which
only a teacher can personally give to a student. The relation of a teacher and
student is divine which is lost in innovative methods of learning.

89

Thus innovation in the field of education should be used to help the teachers in
giving education to the students but it can certainly not take the place of a
teacher.



69. The best ideas arise from a passionate interest in commonplace things.
At the first glance, the world consists of simple things. But all you need is just to
magnify them or to look at them from a different angle to see how complicated
they actually are. I truly believe that careful and persistent investigation of even
most commonplace objects has allowed mankind to create its most significant
ideas.
People are always looking around them. It is a kind of a natural instinct - to look
around and to notice new details in old things. A lot of technological innovations
are derived from the prototypes created by nature. Throughout the time the
flight of birds has inspired humans to make a flying machine, which is heavier
than air. A lot of prominent thinkers have tried to solve this problem until the
law of uplifting force was discovered. This is an excellent example of how the
pure inspiration by such common live creatures as birds has allowed humans to
overcome the earth's grasp. Modern insect-like robots, used for exploration of
space, may serve as another instance of usage of usual, but elaborated natural
forms for the benefit of technology. However, engineering is not the only sphere
of human knowledge where the use of common objects allow to make a
significant breakthrough.
Contemporary Biology is another perfect example. While being nearly purely
descriptive science in the XIX and early XX century, nowadays the life science is
mostly experimental. Now, it becomes clear that the most significant
accomplishments of molecular biology were reached through systematical and
careful examination of the most common life creatures, the so-called model
objects. The persistent investigation of a single celled bacteria Escherichia coli
allow mankind to create artificial insulin, to solve the question of a genetic code,
to learn how the species evolve etc. One famous biological proverb states:
"Everything that is true for E. coli will be true for elephant as well". And that is
generally true.
The art has also become somehow dependent from the common things. The
forms of nature always inspired the painters. Even one single landscape may
allow several artists to see the new facets of a common picture. Some divisions of
the modern art even use common objects in their unchanged form. The simple
commonplace objects become masterpieces if we learn how to look at it from a
different angle. "Even a simple bathtub can be beautiful" - think some artists.
In conclusion, it becomes evident that the mankind throughout it's history have
used the commonplace things as a source of inspiration and new ideas. The ideas
borrowed from nature find it's application in all spheres of technology. Use of

90

common life organisms as model objects have serves as a basis of the rise of
biological science. Art has also found it's inspiration in common forms.




70. Claim: Imagination is a more valuable asset than experience.
Reason: People who lack experience are free to imagine what is possible
without the constraints of established habits and attitudes.

In most professions and academic fields, imagination is more important than


knowledge

Experience is valued in every field and imagination is a valuable asset to
endeavor a fresh approach towards things. Experience warns against actions
that are likely to prove disastrous. It inculcates the tact of handling difficult
situations along with taking success and failure in one's stride. The significance
of experience is confirmed when one applies for a job in any field. Experience of
work is always preferred, as it proves to be a bonus to the organization.
Imagination on the other hand, helps to develop a fresh outlook for every issue.
The freshness and novelty of ideas fills the person with such great amount of zeal
that it is most likely to spread around and affect the associates too.
If experience makes one better with each passing day, it may limit the ability to
break past established attitudes. For example, if being indifferent to the personal
problems of his employees had helped someone to get the work done on time, a
change in attitude may develop more healthy relations with the employees,
resulting in more loyalty and quality work. However, such an attitude would be
difficult to develop in a person who has not realized the need of such a change in
his long work experience, or in someone who has had the experience of being
taken for granted by his juniors in the past.
Similarly, an individual as a salaried employee may have had made unsuccessful
attempts to start his own business. Though, it could be due to his personal
limitations or some unfavorable circumstances prevailing in the market, his
experience would always come in between the desire to start afresh. On the
other hand, his inexperienced kids, with novel imagination would be undeterred
and in a better position to achieve the same goals. In such a case, it is definitely
better to be inexperienced and sustain zeal to fulfill one's fancies and
imaginations.
There goes a famous saying, "If you can dream it, you can do it." Imaginations are
production of mind, which run wild, and knows no limits. If imaginations are not
influenced by the social or economic constraints, there are a number of ways of
tackling the present problems. Not to mention, inventions have always been a
production of an imaginative mind. People, who do not let their imaginations get

91

imprisoned in well known and set patterns, are the ones who have the aptitude
to invent something new and implement novel ideas.
There is no doubt in the truth that experience is counted upon in every aspect of
life, but at times it leads to inhibition in trying something risky. Though,
experience can help in playing safe, it is risk that must be taken to endeavor into
a new venture. The ones who are not set back by failures and preconceived
notions are the ones who trail new paths for others to follow.
Hence, experience and imagination, both hold their respective significance in
achieving success in different aspects of life. Experience of past can be
instrumental in strengthening the positives, if it does not hold one back from
trying out new ideas based on imagination. In other words, though a rare
combination, experience and imagination together would be an ideal pair to
achieve success.



71. To be an effective leader, a public official must maintain the highest ethical
and moral standards.

It is not by mere appointment that a person becomes a leader but by the virtue of
his deeds that he casts an influence over people. Besides fulfilling his duties
towards people, a leader should be able to inspire them and set the utmost
standards of dedication, sincerity, honesty, ethics and morals. I agree with the
statement of the author. A public official must maintain the highest possible
ethical and moral standards.
The role of a public official is to serve the people who have given him the right to
make decisions on their behalf. He is a representative of a large number of
people belonging to different sections of the society. They come from varying
socio-economic categories and unanimously decide who is to lead them. A public
official is thus responsible for safeguarding the interests of all people alike.
Moreover, he is under public scrutiny at every moment. Since people choose him
as their representative, he is expected to behave in accordance with their
intensions. He cannot easily get away with wrong decisions or unacceptable
behavior. There are bound to be discussions and allegations if a public official
behaves in an unacceptable manner.
An unscrupulous public official might manage to retain his position but he shall
lose the respect people have for him. He will be overthrown once people awaken
to his misconduct. Once the image of a leader is tarnished, not only is it difficult
for him to come out clean the next time around, it is nearly impossible for him to
get rid of the stigma. People might never forgive him. Take for instance Bill
Clinton; one immoral act of his is not likely to be forgiven for long.

92

Moral and ethical conduct of a public official is given utmost value when people
judge him. In today's world, where corruption is rampant, people look for a true
leader who is sincere towards their interests. If a person is known to be
dishonest, people shall never trust him with decisions about their welfare. They
would rather choose a person with lesser experience and exposure.
It has been seen that people forgive great leaders their errors of decision but
there is no respite for those who err morally. When a leader misbehaves morally,
it destroys the trust people have in him. Such damage to a person's demeanor is
irreparable. Even if he is forgiven, there shall continue to be doubts in people's
minds about the sincerity of such a person.
Where a public official has to mind his conduct, it is equally vital for the public to
understand him. The moral standards of each person are different. Where telling
a lie for the betterment of another is forgivable for some, it could classify as a
scandalous act for another person. There are many things that leaders have to
balance. Thus, there could be measures that some public officials have to take to
avoid a grave problem in the future. This could lead to some misunderstandings
among people. In such times, people have to be wise enough to support their
leader. They have to realize that the same yardstick cannot judge every situation.
Thus, where a public official has to maintain the highest moral and ethical
standards, people should be discrete while judging their leaders.




72. Critical judgment of work in any given field has little value unless it comes
from someone who is an expert in that field.

Can a television channel appoint a professional doctor as the judge for one of its
reality shows that is based on a salsa dance competition? The answer is obvious.
There is no way that the doctor will be able to ascertain the winner of the
competition unless he himself is aware of the nuances of salsa dancing. The
modern world consists of numerous self-proclaimed critics who judge work in
any given field irrespective of their own knowledge in that field. Such judgment
is utterly useless, unless the person making the judgment is an expert in the field
to which the work being judged belongs.
If you browse the Internet for reviews of the latest released movies, you will find
numerous web sites where common people have posted reviews of the movie.
Do we give importance to the comments posted on such blogs? We probably do
not give much importance to what these people say about a particular movie as
such observations are largely personal opinions and they may have to do nothing
with the actual value of the movie. However, we give due credence to the opinion
of a renowned film critic who may appear on a television program that is based
on film reviews. The observations made by the film critic may change our

93

perspective on the movie much before we have actually seen it. Why do we have
this difference in accepting the views presented by random bloggers and those
presented by well-known film critics? The reason behind this difference in
acceptance of judgment from different types of people is that we attach a higher
value to the judgment passed by people who are proficient in the same field.
If a person has to select suitable furniture for home decor that falls within
his financial budget, where would he look for sound advice? Would he just ask
his friends randomly, or would he rather ask an acquaintance who is a furniture
designer himself? This designer would be able to give him an idea about how
much the furniture should ideally cost and he would also be aware of the names
of commonly known furniture dealers and the amount they charge for the
furniture marketed by them. People who have no idea about furniture designs,
quality of furniture and the costs involved will not be able to point out the pros
and cons of the furniture being selected by this person. They will only go by the
external appearance of the furniture and their judgment will probably not have
any value. Contrast this with a person who is a seasoned furniture expert. He
would be able to identify the type of wood used, its durability, termite resistance
and other related aspects, which have to be considered before buying the
furniture. His judgment would hold much more value, as he will be able to
identity the actual worth of the furniture with respect to its cost.
The same holds true for critical judgment in all fields whether they are related to
the sciences or to arts. Only the science teachers of the school can do the
judgment for the best school science project. On the other hand, the same science
teachers will not be able to pass valued judgment in a painting competition. They
may be able to judge a painting by its face value, but only the painting teachers
can judge the types of colors and brushes that should have ideally been used, as
they are experts in that particular field. Hence, it is obvious that you need to be
an expert in a particular field to be able to pass valued judgment on work
pertaining to that field


73. Any leader who is quickly and easily influenced by shifts in popular
opinion will accomplish little.

The most important characteristic for a leader is that he should be consistent in
his thinking. That thinking can be in any field be it decision making,
policymaking or even his principles and objectives. If a leader is not consistent in
his own thinking then how can he ever think about other's well being? It is very
important for a leader to be sure and consistent in whatever he does because if
others decisions quickly influence him then he can never make a stand for
himself. Leaders are looked upon as a source of inspiration and if they are not
principled then they would not be able to run the social system for us. Therefore,
it is essential for an effective leader to remain consistently committed to
particular principles and objectives, as the leader who is quickly influenced by
shifts in popular opinion will not accomplish anything.

94

We have the example of great Gandhi who had a clear vision in his mind from the
very beginning. He wanted to make India free from the clutches of British rule.
His principles of non-violence were deep rooted in his thoughts. Everyone
discouraged him thinking how he could ever get his country free from East India
Company but his belief was not shattered and he got India free from British. The
leaders with clear vision can not only achieve their aim but also inspire others to
follow their path. Gandhi is an ideal for many people and Indian history is
incomplete without his description. Similarly, German Nazi dictator Adolph
Hitler is another example. He also had his set objectives and principles and he
has made a niche for himself in history.
Although it is very important for a leader to be firm with his principles and
objectives but that certainly does not mean that he becomes fanatic. If a situation
so demands then he should be ready to stake his principles. That does not mean
that he should leave them forever. Man makes principles and principles do not
make a man therefore, it is very important that principles should be flexible.
Sometimes experiences or suggestions of others should also be given importance
because every principle is not applicable everywhere. At certain times even
leaders can make a wrong decision and to avoid such circumstances they should
allow people to put across their views. Having flexibility in ones principles does
not mean changing or abandoning the principles. Too much dependence on
principles can also make a leader accomplish little because rigidity also makes a
person less progressive. It is very important that the leader's objectives and
principles are integrated in the correct direction.
Therefore, it is important for a leader to be clear in his vision about his principles
and objectives, as then only will he be able to do justice to his work. At the same
time a leader should not be forgetting the importance of suggestions from others.



74. Governments must ensure that their major cities receive the financial
support they need in order to thrive, because it is primarily in cities that a
nation's cultural traditions are preserved and generated.

The author claims that governments should ensure that their major cities receive
the financial support they need to preserve the nation's cultural traditions.
However, preserving cultures should not be the single reason to develop major
cities. This is not only for preservation of nation's culture but also for the reason
of developing the whole society.

There is no doubt that major cities are the principal force when constructing a
healthy country and their development directly reflects the development of the
country. Major cities primarily generate a nations cultural traditions and make it
famous in the world. However, it is actually towns and villages where cultural
traditions originate and are preserved and any government that pays attention
to its cultural traditions should neither neglect nor ignore those places. Hence,
these small places should receive equal if not more financial support than major

95

cities. The culture and tradition maintained in these places might be much more
integrated and purer than in counterpart cities. Therefore, in order to protect
their cultural traditions, governments should support tradition-based towns and
villages rather than invest a lot to the cities, which have little traditional
foundation.

For example, Hong Kong used to be a small fishing village 150 years ago. After it
became the colony of the Great Britain, Hong Kong has developed gradually and
has established as a financial center in Asia. Therefore, from its present, nobody
can assert that Chinese cultural traditions are generated in Hong Kong. Hong
Kong developed completely according to the style of the capitalism country,
during the reign of Britain, although the majority of residents were Chinese, who
are known for preserving their culture and traditions. The Chinese culture is not
preserved in Hong Kong; hence there is no point in providing financial support to
Hong Kong just for the culture's sake. However, as it is a major financial and
shopping hub, it attracts people from all over the world and it is a great source of
revenue for the government. Hence, the government of China has to patronize it
to maintain its importance.

While major cities are the resource of one country's culture, the major part of
population in many countries lives in villages. In cities, most people are doing
salary-based jobs and in villages, most of the people are farmers. The
government of a country has to take care of both the sides to keep balance in
society. To construct major cities, the rural places should not be ignored. The
cultural traditions are the backbone of a nation and they should be preserved.
However, major cities and rural areas should get equal attention from the
governments.




75. When old buildings stand on ground that modern planners feel could be
better used for modern purposes, modern development should be given
precedence over the preservation of historic buildings.

How have we educated ourselves about the history of mankind? It is through the
historical buildings we see around us that we get an idea of how our forefathers
lived. Heritage buildings and other forms of old constructions not only give us an
insight into the living conditions of the past, but they are also eye-openers as far
as the architecture in the past is concerned. However, what happens when these
old buildings occupy land that can be effectively utilized for some modern
construction? Instead of hastily deciding to demolish the old buildings or
obstinately sticking to not letting a scratch come to the old buildings, the best
solution would be to strike a balance between the preservation of the old
architecture and the construction of buildings designed to serve modern needs.
We have always been curious to unearth the secrets behind the constructions
carried out in the past, which still continue to baffle all of us. Stonehenge in
Britain is one the world's most astounding piece of architecture. The

96

construction of the Stonehenge is shrouded in mystery and man has only been
able to speculate as to how it may have been constructed in the age when there
were hardly any construction tools available. Take the example of the Sphinx and
the Pyramids of Egypt, which are yet another architectural marvel. These pieces
of architecture have served the purpose of throwing light on the lifestyles of
ancient civilizations. Other old buildings or constructions may not be as famous
or as baffling as the examples quoted above, but each old building provides us
with immense information related to our forefathers. Not only do we get
priceless information about the construction techniques in the past, but we also
get access to various resources that help us understand our past.
It is obvious that demolishing old buildings can never be adjudged as the correct
decision no matter how crucial the new construction is for the present society
because demolishing an old building would be something like erasing a part of
our history. There are numerous examples of old palaces, forts and heritage
buildings that have been converted into luxury hotels that house shopping malls
as well. The architecture, look and feels of the original buildings have been
retained along with the new modern constructions. This is the best solution
wherein an old building has not been completely damaged and it has been
successfully modified into a building that serves the needs of the present society.
It is not necessary that it will be easy to strike such a balance in the case of every
such controversy where a modern construction and an old building are
concerned. For instance, consider a case wherein oil has been struck in the land,
which is a part of a heritage building. Should the old building be demolished to
make way for an oilrig, which is in the interest of the nation, or should the
government preserve the heritage building and ignore the oil well that lies below
it? It would be a tough decision, but the government will have to find a solution
wherein the oil can be pumped out without any damage to the old building.
The past, present and future are all very important in the development of a
society. No society can afford to ignore its past in order to move into the future.
The society should carefully strike a balance wherein it is able to preserve its
past while being able to attend to the common man's needs in the face of
urbanization. Therefore, the best solution would be to find a solution by
somehow preserving the old buildings and at the same time, allowing the new
constructions also to come up.





97

76. The best way for a society to prepare its young people for leadership in
government, industry, or other fields is by instilling in them a sense of
cooperation, not competition.

Some people argue that successful leaders in government, industry, or
other fields must be highly competitive. Other people claim that in order to
be successful, a leader must be willing and able to cooperate with others

When a young child is brought up, there are numerous things he has to be taught,
many of which are conflicting. While one has to learn how to be polite, he has to
even be taught how to be adamant when it is so required. Where compassion for
all human beings is to be taught, it is also required that the child be taught how
to stay away from cons. Bringing up a child and preparing him to be a good
leader is, thus, very challenging. There are numerous debates and discussions on
whether a spirit of competition is more beneficial or a sense of cooperation is
better. In my view, both these aspects are important for a well-balanced
personality to develop. Where competition is required, it is equally important to
learn how things can be done through cooperation.
In today's competitive world, it is not possible for all of us to make our mark. It is
only those few who are motivated and capable enough that make a place for
themselves in society. Such people finally become leaders in their own right and
inspire millions. For such people, the motivation can come from any source. It
could be an aspect of one's personality such as his sense of competition,
compassion, altruism, self-confidence, charity, leadership, intelligence, passion,
greed, jealousy, pride or anything else that could be the reason for his success. A
true leader is the one who inspires people to become like him. For instance, if we
were to choose a leader from the past, we would prefer Mahatma Gandhi to
Hitler. Gandhi was compassionate where Hitler was one of the cruelest people
ever. His pride and determination led to his success, though he could never win
the hearts of people. Gandhi, on the other hand, was compassionate towards
people. He spearheaded the Indian independence struggle while cooperating
with people of different sections of society and became a true leader of masses
and classes alike. He continues to inspire millions.
Cooperation among different nations is a major reason why humankind is
surviving. Without exchanging our raw materials, human resources, food,
medicines, technology, armies etc., it would not be possible for any country to
sustain itself. Similarly, political parties can achieve a lot if they cooperate with
each other for the betterment of the nation as a whole, though this is very
difficult to achieve.
At the same time, it is equally important for a spirit of competition to remain
alive. Great sportsmen insist on a compelling sense of competition for success. In
fact, many sportsmen provoke themselves by some means or the other to
develop a feeling of vengeance towards their competitors. Children perform
much better if there is healthy competition in class. A bright student among
mediocre classmates might never strive enough because of lack of competition
and challenges. A student among those who are at par with him will strive to

98

beat them and excel. This does not mean that such competitive students should
not cooperate with each other. The challenge here is to teach children where the
line is to be drawn. In the name of cooperation, one should not be cheated and
for the sake of competition, one should not fill one's heart with contempt and
vengeance.
It is not the sense of competition or cooperation that leaders require, but the
judicious use of these aspects. Though it is difficult for these two qualities to co-
exist, it is vital nonetheless. Healthy competition, wherever it exists, leads to
success and improvement. Unhealthy competition will always result in
degradation, either of quality or of morals. We should thus teach our young
people how to respect others, cooperate with them and compete for
improvement of their own selves.



77. Some people believe that corporations have a responsibility to promote the
well-being of the societies and environments in which they operate. Others
believe that the only responsibility of corporations, provided they operate
within the law, is to make as much money as possible.

Corporations, world over are entitled the same rights as a person and therefore
undeniably are a part of society and have fundamental accountability towards
the society they operate in. However, corporations are created and organized for
the purpose of maximizing returns for stockholders. It is my assertion that
corporations should single-mindedly pursue this goal, operating within legal
limits, and if contributing to the society has a direct impact on their top line they
should take up the opportunity.
The CSR debate emanated as early as the early 1900s with Dodge vs Ford Motor
company where shareholders filed a case against the promoter Henry Ford for
reducing car prices in order provide an affordable Ford for everyone as against
distributing the profit as dividends. The court upheld the view of the promoters
and noted that the power of the executive or directors cannot be used to
redistribute the shareholder funds or profits for other purposes. This ruling
supports my opinion with regards to this argument under debate, primarily due
to a very simple and strong logic of conflicting objectives.
This argument essentially highlights the dichotomy a corporation functions in
the shareholder theory vs. the stakeholder contention. Being an entity that
operates within the throes of the society, an organization has multiple
stakeholders with variegated objectives the shareholders are interested in
maximizing their wealth, employees require higher wages, customers demand
excellent products and services at reasonable prices & communities demand that
the company work for the larger interest of the society. It highly untenable for an
organization to pursue multiple objectives, especially those representing
conflicting interests such as the ones highlighted above. Even if social good is an

99

agreed objective amongst all stakeholders (including shareholders) there would


be a conflict with stakeholders competing for areas in which funds or resources
need to be directed. The enterprise would wastefully invest time in resolving this
conflict and trying to reach a consensus, when this time would be more
efficiently utilized in taking action that would maximize profits.
Social responsibility has become a buzzword in corporate circles and certain
countries mandate the business community to invest part of their profits in
taking social action. Given this scenario, my view mirrors what is actually
happening. Corporations are using this forced investment to identify
opportunities for themselves that would add to their revenues. The act is not an
altruistic one, but purely mercenary in nature. When a corporation supports the
development of a backward community by building schools and infrastructure, it
is in effect mining a new market and developing a new customer base for itself.
In the case of Dodge vs. Ford highlighted above, if Henry Ford could prove that
the reduced price would lead to an increase in sales volume, substantiating with
numbers, I believe the shareholders would have been in consonant with his view.
In summary, businesses only have the responsibility of being scrupulous and
operating within the confines of law. It is unnecessary and unwise to spend
shareholder money for unprofitable social causes. The shareholders have made
an investment and are dependent on the firm to provide them with a return.
Critics of CSR who I concur with claim, Shareholders do not hire CEOs to be the
U.N., to act like a government or to be a charity. The business of business is to
make money. Only if CSR initiatives increase the bottom line, is implementing
such initiatives is recommended.



78. Some people believe that our ever-increasing use of technology
significantly reduces our opportunities for human interaction. Other
people believe that technology provides us with new and better ways to
communicate and connect with one another.

Philosophers such as John Stuart Mill and Karl Marx believed that the primary
objective of technology should be to increase the leisure time of the people. This
increased leisure time could be invested in social, creative and intellectual
pursuits all of which have an ability to galvanize our lives. While there is no
doubt that means of communications have proliferated, in my view the advent of
technology has resulted in a reduction both in the time and quality of our
interactions. Productivity has become the yardstick to measure all our activities
leaving fewer opportunities to develop or sustain social bonds.
According to ancient philosophers leisure meant a freedom from the routine and
mechanical lifestyle and the belief was that technology would help transform this
and provide people with free time for more creative and intellectual pursuits. As

100

per them this free time was best utilized in cogitating and discussing about the
purpose of life and seeking by ourselves and through our interaction with others
to determine our own individual desires from life, unfettering us from the
established norms and adding a richness in our activities and personalities.
While technological innovations such as the automobile and computer have
helped us speed up our activities considerably, we are much further from the
ideal notion of leisure than our ancestors. Leisure has become a pejorative and is
only justified if it is adding to the amount of work one is doing. Working holidays
have replaced holidays, and thanks to smart phones and connectivity people are
expected to be available at all times. Families are spending far lesser time
communicating with each other with each member of the household being
occupied with one or another form of technology from computers, to laptop, to
smart phones and TV. Family values are falling apart and divorce rates are
significantly going up.
While in medieval times too people worked from morning to evening, work was
more causal and people took off time for social chats and interactions. However
in todays age, sophisticated technology like access cards and employee logs are
employed to monitor employee activities and to ensure that time is productively
invested. The bonds in earlier times were more personal with face-to-face
conversations being the norm instead of communication largely through gadgets
and software. These new mediums have lead to dehumanizing of
communication. A renowned scientist and philosopher noted that information
technology supplies communication canned or frozen and can in no way match
the art of dialogue characterized by both words as well as silence. Advent of
platforms like Facebook, LinkedIn, Skype and instant messaging services like
Blackberry messenger, G-talk have undoubtedly given as an opportunity to stay
in touch with friends and family residing miles away from us or reconnect with
those we had lost touch with, these interactions can at best be described as
superficial and people feeling more lonely and vacuumed than before.
To take a slightly tangential view, proliferation of media technologies has also
impacted the time people take out for themselves and their families. Stories of
the rich and successful having 20 hour days with each minute being invested in
work related activities, mails and texts from colleagues late into the night or in
the wee hours of the morning, global alliances which require people from all
parts of the world to work according to the time of the client; have all created a
sense of insecurity amongst people. Today we are not only expected to be
available, but ourselves wish to be available at all times in order not to be left
behind. This has further put pressure on the time one spends in interacting with
others.
To summarize, while the function of technology should no doubt be an increase
in leisure time helping us enrich our social bonds, in truth technology has lead us
further away from this end.

101

79. Claim: Major policy decisions should always be left to politicians and other
government experts.
Reason: Politicians and other government experts are more informed and
thus have better judgment and perspective than do members of the general
public.

People of a country represent the worth of the nation. Different nations across
the world have different policies for running their government. Where in some
nations democracy is prevalent on the other hand there are nations where
autocracy is still in existence. No matter, which forms of government, is
prevalent in a country; the interest of the people of that country cannot be
overlooked. The will of the people in a democracy is very importance and I
believe they have all the right also to take part in all the decisions, which would
affect their interest as they are eventually going to be affected by it. Therefore
the major policy decisions of a country cannot be left to the politicians or
government experts. The say of the members of the general public should have
equal weight age in this case.
Political leaders are public representatives and they are given the authority and
responsibility on the part of the public to take decisions. A political leader can
take a decision to construct a flyover bridge for a place in a city to solve the
problem of merging traffic in a particular area. The construction of the flyover
would not only result in large expenditure but it can also leave some people
homeless or jobless in the area where buildings have to be demolished for
building the flyover. It could be quite possible that the city must be in more need
of a school rather than a flyover and moreover diverting certain traffic from that
area can also solve the problem of merging traffic. These issues can only be
brought to the notice of the political or government authorities through
representatives of general public. These members would not only convey the
public viewpoint they would also they would be able to convince public in regard
to the decision made by the government. They would prove to be an effective
link between public and government and would be able to remove major
conflicts or misunderstandings from both the fronts.
There is no doubt about the advantages of having members representing general
public as it is in the favor of both, the public and government. There are certain
matters at times, which can not be discussed with public in open as it could lead
to panic. Such situations arise during wars or in emergencies, during such times
we as citizens should have full faith in the decisions of our political and
government officers as they have been elected by us and are our true well
wishers. They are the ones who have the responsibility to run our country and
we have to help them in doing so and not prove to be a hindrance in their way.
However, it should also be the responsibility on the part of the government and
political leaders to involve the general public in making decisions related to them
whenever it is possible and required.

102

It is difficult for both the government and the public to work and perform solely.
They are different sides of the same coin and hence inseparable. They are
incomplete without each other therefore they should work in harmony.



80. It is more harmful to compromise one's own beliefs than to adhere to them.

There may have been many instances in our lives when we have faced the
dilemma to either compromise our beliefs or to adhere to them. How does one
make this choice? It all depends on your principles in life. You can choose to
select the easier way out, which would be achieved by compromising on your
beliefs. Adhering to your beliefs in the face of opposition is a more difficult task.
Nevertheless, irrespective of the situation you are in, it always pays out in the
end to adhere to your beliefs. At the first glance it does look as though a
compromise would result in benefits, but in the longer run this compromise may
prove to be harmful not only to your image but also to your subconscious.
However, this line of thought holds true if your beliefs conform to the norms laid
down by society, otherwise the opposite would be true, that is, adhering to an
immoral belief will be harmful for you.
We all have our religious beliefs ingrained in us since childhood. Everyone is
taught by his parents and his community to carry out his religious practices and
rituals. If you belong to a religion where you are not permitted to have non-
vegetarian food, would your sub-conscious allow you to have a chicken burger if
a person who does not belong to your community offers you? Even if you do have
it, what will be the repercussions? The effects of such a compromise may not be
immediate, but they are bound to follow up as you move ahead in life. Your
community may ostracize you, your parents will admonish you and you may
even regret it later on in life. Therefore, the better option would be to politely
refuse the offer and adhere to your religious beliefs.
Adhering to your beliefs will rarely lead to any harm unless your belief does not
conform to the norms of society. If you are of the belief that there is nothing
wrong in taking bribes and you choose to adhere to your beliefs, then it is
obvious that this will be extremely harmful for you in the long run. However, if
you are of the strong opinion that one should never accept bribery and you have
chosen to be honest in your profession, then compromising your belief will
definitely land you into trouble. Patriots who stuck to their beliefs that their
country should be free from the clutches of an imperialistic power succeeded in
freeing their country. The freedom struggle of India is one such example. Could
this have been possible if the freedom fighters had compromised on their
beliefs? This compromise would have weakened their principles and they would
not have had the strength to fight against all odds for the freedom of their
country.

103

Your beliefs define who you are. Your identity and image are shaped by the
beliefs that you follow in life. It is necessary to be flexible also in life, but
wherever a choice has to be made between right and wrong; one should always
follow the right path irrespective of what your beliefs are. If your beliefs are
morally correct, then adhering to your beliefs cannot harm you, but if your
beliefs are in contrast to those of the society, then adhering to your beliefs may
land you in big trouble. Therefore, it is imperative that you choose your beliefs
carefully and adhere to them if you want to keep out of harm's way.




81. Claim: Colleges and universities should specify all required courses and
eliminate elective courses in order to provide clear guidance for students.
Reason: College studentslike people in generalprefer to follow
directions rather than make their own decisions.

Research indicates that the primary reason for burnout in the work place is
neither monetary nor stress related, but a feeling of impuissance. The same is
true for people across all age groups and in all contexts. It then is incorrect to say
that people prefer to follow directions and use that as the foundation to base the
decision of dictating the course to the students, instead of giving them the
freedom to choose.
Studies across a cross section of settings have proven that an individuals
physical and emotional wellbeing is directly proportional to the sense of control
they feel over their situations. Patients in nursing homes are shown to recover
faster, cases of depression and anxiety are seen to be far lesser and positive
behavior such as a desire to learn and improve oneself is reinforced in a person
who has the autonomy to take his own decisions. Given this insight, it would be
counter-productive to instruct students on what they should study. It would kill
the students drive and cause them to become dull and listless people who are
simply complying with guidelines given to them.
Another implication is that such a policy would immure students into a mould
where they are looked less as individuals and more as products that an
educational institute is mass-producing for the society. It relegates individual
preference to the background and disenfranchises students of their power of
decision-making. This effect becomes so deeply entrenched that individuals
often lose their ability to make decisions and is a phenomenon that is manifested
through ones life and people are often unable to think for themselves or have
the confidence to take a position. An individual who has through his entire initial
period of development has been stopped from taking decisions, cannot be

104

expected as an adult to break out of these shackles and transform into someone
who can cut the Gordian knot.
Longitudinal studies also show that students who were given considerable
freedom in deciding various aspects of their schooling performed better as they
grew older. This is also corroborated by observation of students from Sudbury
Schools, a much higher percentage of who go on to enroll in college than their
counterparts in closely monitored schools.
The problem should also be considered from the aspect that every field differs in
its orientation and straight jacketing education does not yield results. An
individual who wants to go on to join the corporate world in a managerial
capacity would need to have solid grounding in subjects like economics, finance,
leadership and decision making amongst others. An artist on the other hand
need not have an aptitude for economics, but for the different styles and
variations in art, a deep understanding of colors and moods etc. Even within the
same field of study, symmetry cannot be said to exist. An artist might choose to
focus on medieval art while another might be keener on the aspects of modern
art while there would be an overlap of a few core subjects, a large part of the
curriculum would need to be customized. It would be impossible to prefabricate
course structure for each individual preference and would be far more logical
and economical to let students choose their strands of study based on their
preferences.
Opposing views to the above thinking are not completely unfounded. It is likely
that students might not have the experience or vision that adults have and would
need some structure to be able to decide what they want to do or should do
based on their aptitude. The best course would then be to provide guidelines or
framework and also counsel students based on their interests. The debate
however is not whether students should be given completely autonomy but
whether they should be completely devoid of any freedom to take their
decisions. From the points highlighted above, it is clear that depriving students
of this power would have severe implications on their performance both in the
short and the long run and would additionally also be detrimental to the society.


82. No field of study can advance significantly unless it incorporates
knowledge and experience from outside that field.

The claim made in the argument is that a field of study is most benefited when it
draws from interdisciplinary knowledge. I am in agreement with this viewpoint
and would defend my opinion based on three key points people have
contributed significantly to fields in which they had limited exposure, people
working in one field are likely to make mistakes that a person with a fresh
perspective can point out, whole new frontiers could be traversed by crossing

105

two different fields of knowledge which would also have a bearing on solving
social problems.
Let us take the example of Bill Gates, an entrepreneur par excellence who spent a
substantial portion of his life in the IT industry. Post his voluntary retirement he
decided to dedicate his life to philanthropic causes primarily in the area of
health, a field in which he had little exposure or theoretical grounding. But
despite what might popularly be considered a handicap, or perhaps because of
it, the foundation has been able to make significant contribution to the
improvement of health initiatives in third world countries. Bill Gates drew
heavily from this experience as a businessman in creating a new field, that of
philanthrocapitalism, and has been able to alleviate some of the ills that dogged
the philanthropic community scalability, accountability, evaluating success and
raising funds. It was his perspective as an outsider that enabled him to view the
field afresh, to challenge the status quo and look for solutions to problems that
ailed the field.
When one spends a majority of their time working on the same thing, it often
becomes a blind spot and even established experts are unable to subvert this
problem. However when fresh ways of thinking and knowledge from other fields
in imbibed, the problem reveals itself and is followed by a solution that enriches
the field more than before. An example of this is the problem confounding the
scientific community with regards to the molecular structure on an enzyme in a
breed of monkeys that is known to cut proteins from an AIDS like virus. The
puzzle when presented to lay people as an online game was cracked in less than
10 days!
Another defense for the importance of cross disciplinary knowledge is that new
fields of study could be created at the hybrid intersection of existing fields.
Studies on extra terrestrial life forms are one such example, which necessarily
require an understanding of both astronomy and biology. The field would be able
to make little headway in the absence of knowledge of even one of these areas.
Learnings from this field of study could have important implications for
humanity and have the potential to attenuate our problems or compound them.
But in either case an absence of knowledge would be far more harmful.
Opponents however argue that independent fields of study are likely to advance
faster, when an attempt is not being made to solve all problems at one go. They
argue that a possible solution is not presented until it satisfies the beliefs of
experts of all the fields, which could be a daunting task. For instance the
evolutionary science is severely restricted because it contradicts religious
sentiments and beliefs. This behavior however is akin to The Ostrich myth and
one needs to realize that ignoring a problem would not make the problem go
away.
In conclusion there is much to be gained when different fields of knowledge
intersect and work together to solve problems. Significant gains are likely when
an esoteric view is adopted.

106

83. True success can be measured primarily in terms of the goals one sets for
oneself.

The author asserts that true success is not an extrinsic phenomenon but an
intrinsic one and success can be best measured based on the goals one has
defined for oneself. I largely endorse the authors view and believe that the
definition of success cannot be straightjacketed, instead it varies from person to
person depending on what they are striving towards.
Few words are as elusive to a definition as success and while there have been
attempts to define proxies to measure success; they have found little
endorsement from a cross section of people. One of the oft-used proxies for
success is richness and society is quick to label the wealthy as successful ideals
that humanity must aspire towards. However would wealth be a good aspiration
for a majority of the world population that is living below the poverty line?
Would it be good yardstick to measure the grit and determination of people in
war torn countries like Afghanistan, whose biggest achievement could be
characterized as their sense of hope and their ability to move on despite all
odds? Or to the malnourished populations in Somalia who feel a sense of victory
if they can provide four square meals to their family in a day? The definition of
success then changes depending on the circumstances of the individual and only
an internal benchmark could be used to evaluate performance.
While objective externally defined criteria present a metric, they often fail to
motivate people as much as self defined individual goals. Motivation serves as
the basis to impel people towards hard work and persistence, both of which are
essential for true success. In fact drive and doggedness have the power to
subvert all else making people pursue their goals single-mindedly. Even
supposed failures do not present a roadblock then, as proven by personalities
across the spectrum from Henry Ford, to Abraham Lincoln and Oprah Winfrey.
All these people were able to overcome their circumstances because they had an
internal benchmark they had defined for themselves, not because someone
required them to do it.
Another constraint that externally defined measures of success presents is that
they do not take into consideration the ever-evolving nature of internally defined
goals. Goals a person defines for oneself undergo transitions as a person grows
and develops and additionally, past laurels do not immure internal goals. If Bill
Gates believed that being the wealthiest man in the world was the ultimate
manifestation of success, he would not have chosen to start a foundation that
today is making significant contributions in changing health concerns that plague
a majority of the worlds population. Maslows need hierarchy theory states that
self-actualization goals are at the pinnacle driving people to solve previous
conundrums.
From the above it is evidenced that externally defined measures of success pose
more questions than answers and fail on many counts as a means to celebrate
the success of people. It is only when outcomes are measured against individual
determinations, can the true success of an individual be ascertained.

107

84. Governments should offer a free university education to all students



Education plays an important role in the overall development of a child. For the
harmonious transformation of a student into a responsible citizen, education
must be made mandatory at least at the elementary level. To impart education to
all, it should be made free at school level.
However, making college and university education free may yield adverse
results. Students, as they pass high school and reach college are quite grown up
and aware of their aptitudes. There may be many who would rather go in for a
professional or vocational course. Others may be more interested in business.
The ones left out would be genuinely interested in academics. By making
university level education free, more and more students would be tempted to get
enrolled. This can result in unemployment in future as lesser vacancies would be
available for the large number of qualified students. To tackle unemployment,
government would have to adopt various methods like entrance exams etc.,
which are not very reliable in selecting the meritorious and deserving students.
Moreover, due to shortage of appropriate posts, the more qualified ones may
have to settle down for jobs that do not honor their capabilities and hard work
put in for the acquired degrees. This will not only demoralize the overqualified
ones, but also distort the image of university education in the eyes of new
aspirants.
College and university education, if made free, will heavily burden the
government financially. It will be not only difficult to provide the best of facilities
like good libraries, laboratories, scientific and sports equipments etc. but also
challenging to pay the hired staff and faculty. In the wake of lesser salaries, less
and less people would be interested in joining the faculty. If the deserving
candidates would not be paid justifiably, then there will be deterioration in the
quality of education imparted. Making education free at university level may also
reduce the standards of education as students may develop a laid back attitude
and may not feel the necessity to clear the courses at the earliest. This will
further deteriorate the standard of education and create a huge backlog of
students for the government to entertain.
Apart from the financial difficulty in paying the staff and providing appropriate
facilities to the students, there will be yet another challenge for the government
to look after the other employment channels.
Although education is a must, there are other skills required too in different
fields. If all these skills are explored and promoted well, only then can a nation
be more independent financially. The works of handicrafts, interior and fashion
designing, sports, farming, etc. are just a few to mention that need the
government's assistance to flourish to the fullest. To be unbiased towards all
streams of talent and employment, the government will be under pressure to
make all these trainings free as well. This will be a difficult task to accomplish.
Thus, making the university level education free will put a lot of economic strain
on the government.

108

Making university and college level education free will be beneficial neither for
the government nor for the citizens in the long run. Thus, instead of making the
university education free, the government should adopt other ways of
channelizing the talents of the youth and also keep a check on private
institutions overburdening the students by high fee structure.



*** END ***

Вам также может понравиться